ВсС ΠΎ Ρ‚ΡŽΠ½ΠΈΠ½Π³Π΅ Π°Π²Ρ‚ΠΎ

Π₯имичСскиС свойства Π²Ρ‹ΡΡˆΠ΅Π³ΠΎ оксида Π°Π·ΠΎΡ‚Π°. О.Ρ€.валСдинская. Π₯имичСскиС свойства Π°Π·ΠΎΡ‚Π½ΠΎΠΉ кислоты

ΠžΠΊΡΠΈΠ΄Ρ‹ ΠΈ гидроксиды Π°Π·ΠΎΡ‚Π°
ВАЙНА Π‘Π’Π•ΠŸΠ•ΠΠ•Π™ ΠžΠšΠ˜Π‘Π›Π•ΠΠ˜Π―

Азот ΠΎΠ±Ρ€Π°Π·ΡƒΠ΅Ρ‚ ряд оксидов, Ρ„ΠΎΡ€ΠΌΠ°Π»ΡŒΠ½ΠΎ ΠΎΡ‚Π²Π΅Ρ‡Π°ΡŽΡ‰ΠΈΡ… всСм Π²ΠΎΠ·ΠΌΠΎΠΆΠ½Ρ‹ΠΌ стСпСням окислСния ΠΎΡ‚ +1 Π΄ΠΎ +5: N 2 O, NО, N 2 O 3 , NO 2 , N 2 O 5 , ΠΎΠ΄Π½Π°ΠΊΠΎ всСго Π΄Π²Π° ΠΈΠ· Π½ΠΈΡ… - оксид Π°Π·ΠΎΡ‚Π°(II) ΠΈ оксид Π°Π·ΠΎΡ‚Π°(IV) - Π½Π΅ Ρ‚ΠΎΠ»ΡŒΠΊΠΎ устойчивы ΠΏΡ€ΠΈ ΠΎΠ±Ρ‹Ρ‡Π½Ρ‹Ρ… условиях, Π½ΠΎ ΠΈ Π°ΠΊΡ‚ΠΈΠ²Π½ΠΎ задСйствованы Π² ΠΏΡ€ΠΈΡ€ΠΎΠ΄Π½ΠΎΠΌ ΠΈ ΠΏΡ€ΠΎΠΌΡ‹ΡˆΠ»Π΅Π½Π½ΠΎΠΌ ΠΊΡ€ΡƒΠ³ΠΎΠ²ΠΎΡ€ΠΎΡ‚Π°Ρ… Π°Π·ΠΎΡ‚Π°. ΠŸΠΎΡΡ‚ΠΎΠΌΡƒ ΠΌΡ‹ займСмся ΠΈΠ·ΡƒΡ‡Π΅Π½ΠΈΠ΅ΠΌ ΠΈΠΌΠ΅Π½Π½ΠΎ ΠΈΡ… свойств (Π² сравнСнии). НачнСм, ΠΊΠ°ΠΊ ΠΎΠ±Ρ‹Ρ‡Π½ΠΎ, со строСния ΠΌΠΎΠ»Π΅ΠΊΡƒΠ».

CΡ‚Ρ€ΠΎΠ΅Π½ΠΈΠ΅ ΠΌΠΎΠ»Π΅ΠΊΡƒΠ» оксидов Π°Π·ΠΎΡ‚Π°

ΠœΠΎΠ»Π΅ΠΊΡƒΠ»Π° NO. Π‘Ρ‚Ρ€ΠΎΠ΅Π½ΠΈΠ΅ ΠΏΡ€Π΅Π΄ΠΏΠΎΠ»ΠΎΠΆΠΈΡ‚ΡŒ достаточно просто: Ρƒ кислорода Π΄Π²Π° нСспарСнных элСктрона, Ρƒ Π°Π·ΠΎΡ‚Π° Ρ‚Ρ€ΠΈ – образуСтся двойная связь ΠΈ ΠΎΠ΄ΠΈΠ½ нСспарСнный элСктрон Π² остаткС... НСпросто ΠΎΡ‚Π²Π΅Ρ‚ΠΈΡ‚ΡŒ Π½Π° вопрос, ΠΏΠΎΡ‡Π΅ΠΌΡƒ такая «нСстандартная» ΠΌΠΎΠ»Π΅ΠΊΡƒΠ»Π° устойчива. ΠšΡΡ‚Π°Ρ‚ΠΈ, стоит Π·Π°ΠΌΠ΅Ρ‚ΠΈΡ‚ΡŒ, Ρ‡Ρ‚ΠΎ устойчивыС свободныС Ρ€Π°Π΄ΠΈΠΊΠ°Π»Ρ‹ – ΠΌΠΎΠ»Π΅ΠΊΡƒΠ»Ρ‹ с нСспарСнными элСктронами – Π² ΠΏΡ€ΠΈΡ€ΠΎΠ΄Π΅ достаточно Ρ€Π΅Π΄ΠΊΠΈ. МоТно ΠΏΡ€Π΅Π΄ΠΏΠΎΠ»ΠΎΠΆΠΈΡ‚ΡŒ, Ρ‡Ρ‚ΠΎ ΠΌΠΎΠ»Π΅ΠΊΡƒΠ»Ρ‹ NO Π±ΡƒΠ΄ΡƒΡ‚ ΡΠΏΠ°Ρ€ΠΈΠ²Π°Ρ‚ΡŒΡΡ ΠΈ ΠΎΠ±Ρ€Π°Π·ΠΎΠ²Ρ‹Π²Π°Ρ‚ΡŒ ΡƒΠ΄Π²ΠΎΠ΅Π½Π½ΡƒΡŽ, ΠΈΠ»ΠΈ Π΄ΠΈΠΌΠ΅Ρ€Π½ΡƒΡŽ, ΠΌΠΎΠ»Π΅ΠΊΡƒΠ»Ρƒ ONNO. Π’Π°ΠΊΠΈΠΌ ΠΎΠ±Ρ€Π°Π·ΠΎΠΌ удаСтся Ρ€Π΅ΡˆΠΈΡ‚ΡŒ ΠΏΡ€ΠΎΠ±Π»Π΅ΠΌΡƒ нСспарСнного элСктрона.
ΠœΠΎΠ»Π΅ΠΊΡƒΠ»Π° NO 2 . Казалось Π±Ρ‹, Ρ‡Π΅Π³ΠΎ ΠΏΡ€ΠΎΡ‰Π΅ – ΠΊ ΠΌΠΎΠ»Π΅ΠΊΡƒΠ»Π΅ NО ΠΏΠΎ нСспарСнному элСктрону присоСдинился Π°Ρ‚ΠΎΠΌ кислорода. (На самом Π΄Π΅Π»Π΅ присоСдиняСтся Π½Π΅ Π°Ρ‚ΠΎΠΌ, Π° ΠΌΠΎΠ»Π΅ΠΊΡƒΠ»Π°, ΠΈ Π½Π΅ ΠΊ NО, Π° ΠΊ Π΄ΠΈΠΌΠ΅Ρ€Ρƒ ОNNO. ΠŸΠΎΡΡ‚ΠΎΠΌΡƒ-Ρ‚ΠΎ ΠΈ ΡƒΠΌΠ΅Π½ΡŒΡˆΠ°Π΅Ρ‚ΡΡ ΡΠΊΠΎΡ€ΠΎΡΡ‚ΡŒ присоСдинСния с ΠΏΠΎΠ²Ρ‹ΡˆΠ΅Π½ΠΈΠ΅ΠΌ Ρ‚Π΅ΠΌΠΏΠ΅Ρ€Π°Ρ‚ΡƒΡ€Ρ‹ – Π΄ΠΈΠΌΠ΅Ρ€ разваливаСтся Π½Π° ΠΏΠΎΠ»ΠΎΠ²ΠΈΠ½ΠΊΠΈ.) И Ρ‚Π΅ΠΏΠ΅Ρ€ΡŒ ΡƒΠΆΠ΅ Ρƒ кислорода появился нСспарСнный элСктрон – ΠΌΠΎΠ»Π΅ΠΊΡƒΠ»Π° оксида Π°Π·ΠΎΡ‚Π°(IV) Ρ‚ΠΎΠΆΠ΅ свободный Ρ€Π°Π΄ΠΈΠΊΠ°Π». Однако извСстно, Ρ‡Ρ‚ΠΎ ΠΏΡ€ΠΈ соСдинСнии Π΄Π²ΡƒΡ… ΠΌΠΎΠ»Π΅ΠΊΡƒΠ» NО 2 ΠΈ ΠΎΠ±Ρ€Π°Π·ΠΎΠ²Π°Π½ΠΈΠΈ ΠΌΠΎΠ»Π΅ΠΊΡƒΠ»Ρ‹ N 2 O 4 связь осущСствляСтся Ρ‡Π΅Ρ€Π΅Π· Π°Ρ‚ΠΎΠΌΡ‹ Π°Π·ΠΎΡ‚Π°, Π·Π½Π°Ρ‡ΠΈΡ‚, ΠΈΠΌΠ΅Π½Π½ΠΎ Ρƒ Π°Π·ΠΎΡ‚Π° ΠΈ Π΄ΠΎΠ»ΠΆΠ΅Π½ Π±Ρ‹ Π±Ρ‹Ρ‚ΡŒ этот самый нСспарСнный элСктрон. Как это ΠΌΠΎΠΆΠ½ΠΎ ΠΎΡΡƒΡ‰Π΅ΡΡ‚Π²ΠΈΡ‚ΡŒ?
ΠžΡ‚Π²Π΅Ρ‚ Π½Π΅Ρ‚Ρ€Π°Π΄ΠΈΡ†ΠΈΠΎΠ½Π΅Π½, Π½ΠΎ Π²ΠΏΠΎΠ»Π½Π΅ Π² Β«Ρ…Π°Ρ€Π°ΠΊΡ‚Π΅Ρ€Π΅Β» Π°Π·ΠΎΡ‚Π° – Π΄ΠΎΠ½ΠΎΡ€Π½ΠΎ-акцСпторная связь. Π˜ΡΠΏΠΎΠ»ΡŒΠ·ΡƒΡ Π»ΠΎΠ³ΠΈΠΊΡƒ, рассмотрим элСктроны, ΠΊΠΎΡ‚ΠΎΡ€Ρ‹Π΅ Π΅ΡΡ‚ΡŒ Ρƒ Π°Ρ‚ΠΎΠΌΠ° Π°Π·ΠΎΡ‚Π° Π² ΠΌΠΎΠ»Π΅ΠΊΡƒΠ»Π΅ NО. Π­Ρ‚ΠΎ нСспарСнный элСктрон, свободная ΠΏΠ°Ρ€Π° элСктронов ΠΈ Π΅Ρ‰Π΅ Π΄Π²Π° элСктрона Π½Π° связи с кислородом – всСго ΠΏΡΡ‚ΡŒ. А Ρƒ Π°Ρ‚ΠΎΠΌΠ° кислорода, «выходящСго Π½Π° связь», ΡˆΠ΅ΡΡ‚ΡŒ элСктронов Π½Π° Ρ‡Π΅Ρ‚Ρ‹Ρ€Π΅Ρ… орбиталях. Если Ρ€Π°ΡΠΏΠΎΠ»ΠΎΠΆΠΈΡ‚ΡŒ ΠΈΡ… ΠΏΠΎ Π΄Π²Π°, Ρ‚ΠΎ ΠΎΠ΄Π½Π° ΠΎΡ€Π±ΠΈΡ‚Π°Π»ΡŒ останСтся свободной. Π’ΠΎΡ‚ ΠΈΠΌΠ΅Π½Π½ΠΎ Π΅Π΅ ΠΈ Π·Π°Π½ΠΈΠΌΠ°Π΅Ρ‚ ΠΏΠ°Ρ€Π° элСктронов Π°Ρ‚ΠΎΠΌΠ° Π°Π·ΠΎΡ‚Π°, Π° нСспарСнный элСктрон Π² этой связи оказываСтся ΡΠΎΠ²Π΅Ρ€ΡˆΠ΅Π½Π½ΠΎ Π½ΠΈ ΠΏΡ€ΠΈ Ρ‡Π΅ΠΌ (рис. 1, 2).
Π‘Ρ‚ΠΎΠΈΡ‚ ΡƒΠΏΠΎΠΌΡΠ½ΡƒΡ‚ΡŒ Π΅Ρ‰Π΅ ΠΎΠ΄ΠΈΠ½ ΠΌΠΎΠΌΠ΅Π½Ρ‚ – Ρ€Π°Π· ΠΏΠ°Ρ€Π° элСктронов, находящаяся Π½Π° s -ΠΎΡ€Π±ΠΈΡ‚Π°Π»ΠΈ, «пошла Π½Π° связь», ΠΎΠ½Π° Π±Ρ‹Π»Π° просто обязана ΠΏΠΎΠ΄Π²Π΅Ρ€Π³Π½ΡƒΡ‚ΡŒΡΡ Π³ΠΈΠ±Ρ€ΠΈΠ΄ΠΈΠ·Π°Ρ†ΠΈΠΈ – ΠΎΡ‡Π΅Π½ΡŒ слоТно ΠΏΡ€Π΅Π΄Π»ΠΎΠΆΠΈΡ‚ΡŒ Π²Ρ‚ΠΎΡ€ΠΎΠΌΡƒ Π°Ρ‚ΠΎΠΌΡƒ Π² ΠΎΠ±Ρ‰Π΅Π΅ пользованиС ΠΏΠ°Ρ€Ρƒ элСктронов, Ρ€Π°Π²Π½ΠΎΠΌΠ΅Ρ€Π½ΠΎ Ρ€Π°ΡΠΏΡ€Π΅Π΄Π΅Π»Π΅Π½Π½ΡƒΡŽ ΠΏΠΎ повСрхности ΠΏΠ΅Ρ€Π²ΠΎΠ³ΠΎ Π°Ρ‚ΠΎΠΌΠ°. Π’ΠΎΠ·Π½ΠΈΠΊΠ°Π΅Ρ‚ вопрос: ΠΊΠ°ΠΊΠΎΠΉ Ρ‚ΠΈΠΏ Π³ΠΈΠ±Ρ€ΠΈΠ΄ΠΈΠ·Π°Ρ†ΠΈΠΈ ΠΈΡΠΏΠΎΠ»ΡŒΠ·ΡƒΠ΅Ρ‚ Π°Ρ‚ΠΎΠΌ? ΠžΡ‚Π²Π΅Ρ‚: Ρ‚Ρ€ΠΈ элСктронныС ΠΎΡ€Π±ΠΈΡ‚Π°Π»ΠΈ Π°Π·ΠΎΡ‚Π° находятся Π² состоянии sp 2 -Π³ΠΈΠ±Ρ€ΠΈΠ΄ΠΈΠ·Π°Ρ†ΠΈΠΈ. ΠœΠΎΠ»Π΅ΠΊΡƒΠ»Π° NO 2 угловая, ΡƒΠ³ΠΎΠ» 134Β° (ΡƒΠ³ΠΎΠ» большС 120Β° ΠΏΠΎΡ‚ΠΎΠΌΡƒ, Ρ‡Ρ‚ΠΎ ΠΎΠ΄ΠΈΠ½ элСктрон ΠΎΡ‚Ρ‚Π°Π»ΠΊΠΈΠ²Π°Π΅Ρ‚ ΠΎΡ‚ сСбя элСктроны связи слабСС, Ρ‡Π΅ΠΌ ΠΏΠ°Ρ€Π° элСктронов) (рис. 3–5).

ЀизичСскиС свойства оксидов Π°Π·ΠΎΡ‚Π°

Оксид Π°Π·ΠΎΡ‚Π°(II) NO. ΠšΡ€ΠΈΡΡ‚Π°Π»Π»ΠΈΡ‡Π΅ΡΠΊΠ°Ρ Ρ€Π΅ΡˆΠ΅Ρ‚ΠΊΠ° молСкулярная; ΠΌΠΎΠ»Π΅ΠΊΡƒΠ»Π° лСгкая, слабополярная (ΡΠ»Π΅ΠΊΡ‚Ρ€ΠΎΠΎΡ‚Ρ€ΠΈΡ†Π°Ρ‚Π΅Π»ΡŒΠ½ΠΎΡΡ‚ΡŒ кислорода Π²Ρ‹ΡˆΠ΅, Ρ‡Π΅ΠΌ Ρƒ Π°Π·ΠΎΡ‚Π°, Π½ΠΎ Π½Π΅ Π½Π°ΠΌΠ½ΠΎΠ³ΠΎ). МоТно ΠΏΡ€Π΅Π΄ΠΏΠΎΠ»ΠΎΠΆΠΈΡ‚ΡŒ, Ρ‡Ρ‚ΠΎ Ρ‚Π΅ΠΌΠΏΠ΅Ρ€Π°Ρ‚ΡƒΡ€Ρ‹ плавлСния ΠΈ кипСния Π±ΡƒΠ΄ΡƒΡ‚ Π½ΠΈΠ·ΠΊΠΈΠΌΠΈ, Π½ΠΎ Π²Ρ‹ΡˆΠ΅, Ρ‡Π΅ΠΌ Ρƒ Π°Π·ΠΎΡ‚Π°, Ρ‚. ΠΊ. какая-никакая ΠΏΠΎΠ»ΡΡ€Π½ΠΎΡΡ‚ΡŒ ΠΌΠΎΠ»Π΅ΠΊΡƒΠ»Ρ‹ Π΄Π°Π΅Ρ‚ Π²ΠΎΠ·ΠΌΠΎΠΆΠ½ΠΎΡΡ‚ΡŒ ΠΏΠΎΠ΄ΠΊΠ»ΡŽΡ‡Π°Ρ‚ΡŒ элСктростатичСскиС силы притяТСния ΠΊ просто мСТмолСкулярным силам. ΠžΠ±Ρ€Π°Π·ΠΎΠ²Π°Π½ΠΈΠ΅ Π΄ΠΈΠΌΠ΅Ρ€Π° Ρ‚ΠΎΠΆΠ΅ способствуСт ΠΏΠΎΠ²Ρ‹ΡˆΠ΅Π½ΠΈΡŽ Ρ‚Π΅ΠΌΠΏΠ΅Ρ€Π°Ρ‚ΡƒΡ€Ρ‹ кипСния, утяТСляя ΠΌΠΎΠ»Π΅ΠΊΡƒΠ»Ρƒ. Π‘Ρ‚Ρ€ΠΎΠ΅Π½ΠΈΠ΅ ΠΌΠΎΠ»Π΅ΠΊΡƒΠ»Ρ‹ позволяСт ΠΏΡ€Π΅Π΄ΠΏΠΎΠ»ΠΎΠΆΠΈΡ‚ΡŒ ΠΈ Π½Π΅Π²Ρ‹ΡΠΎΠΊΡƒΡŽ Ρ€Π°ΡΡ‚Π²ΠΎΡ€ΠΈΠΌΠΎΡΡ‚ΡŒ Π² Π²ΠΎΠ΄Π΅ – растворитСлС Π·Π°ΠΌΠ΅Ρ‚Π½ΠΎ Π±ΠΎΠ»Π΅Π΅ полярном, Ρ‡Π΅ΠΌ NO. ΠžΡ‚Π΄Π΅Π»ΡŒΠ½ΠΎ стоит ΠΏΠΎΠ΄Ρ‡Π΅Ρ€ΠΊΠ½ΡƒΡ‚ΡŒ, Ρ‡Ρ‚ΠΎ оксид Π°Π·ΠΎΡ‚Π°(II) Π½Π΅ ΠΈΠΌΠ΅Π΅Ρ‚ Π½ΠΈ Ρ†Π²Π΅Ρ‚Π°, Π½ΠΈ Π·Π°ΠΏΠ°Ρ…Π°.

Оксид Π°Π·ΠΎΡ‚Π°(IV) NO 2 . ΠšΡ€ΠΈΡΡ‚Π°Π»Π»ΠΈΡ‡Π΅ΡΠΊΠ°Ρ Ρ€Π΅ΡˆΠ΅Ρ‚ΠΊΠ° Ρ‚ΠΎΠΆΠ΅ молСкулярная, ΠΎΠ΄Π½Π°ΠΊΠΎ ΠΏΠΎΡΠΊΠΎΠ»ΡŒΠΊΡƒ сама ΠΌΠΎΠ»Π΅ΠΊΡƒΠ»Π° тяТСлСС NO ΠΈ ΡΠΊΠ»ΠΎΠ½Π½ΠΎΡΡ‚ΡŒ ΠΊ Π΄ΠΈΠΌΠ΅Ρ€ΠΈΠ·Π°Ρ†ΠΈΠΈ Ρƒ Π½Π΅Π΅ Π·Π°ΠΌΠ΅Ρ‚Π½ΠΎ Π²Ρ‹ΡˆΠ΅, Ρ‚ΠΎ ΠΏΠ»Π°Π²ΠΈΡ‚ΡŒΡΡ ΠΈ ΠΊΠΈΠΏΠ΅Ρ‚ΡŒ это вСщСство Π΄ΠΎΠ»ΠΆΠ½ΠΎ ΠΏΡ€ΠΈ Π·Π°ΠΌΠ΅Ρ‚Π½ΠΎ Π±ΠΎΠ»Π΅Π΅ высоких Ρ‚Π΅ΠΌΠΏΠ΅Ρ€Π°Ρ‚ΡƒΡ€Π°Ρ…. Π’Π΅ΠΌΠΏΠ΅Ρ€Π°Ρ‚ΡƒΡ€Π° кипСния составляСт 21 Β°Π‘, поэтому ΠΏΡ€ΠΈ ΠΎΠ±Ρ‹Ρ‡Π½Ρ‹Ρ… условиях – 20 Β°Π‘ ΠΈ 760 ΠΌΠΌ Ρ€Ρ‚. ст. – оксид Π°Π·ΠΎΡ‚Π°(IV) ΠΆΠΈΠ΄ΠΊΠΈΠΉ.
Π’Π΅ΠΏΠ΅Ρ€ΡŒ займСмся Ρ€Π°ΡΡ‚Π²ΠΎΡ€ΠΈΠΌΠΎΡΡ‚ΡŒΡŽ. Вспомним, Ρ‡Ρ‚ΠΎ ΠΏΠΎΠ΄ словом Β«Ρ€Π°ΡΡ‚Π²ΠΎΡ€ΠΈΠΌΠΎΡΡ‚ΡŒΒ» ΠΌΠΎΠ³ΡƒΡ‚ ΠΏΠΎΠ΄Ρ€Π°Π·ΡƒΠΌΠ΅Π²Π°Ρ‚ΡŒΡΡ ΠΈ химичСскиС Ρ€Π΅Π°ΠΊΡ†ΠΈΠΈ с Π²ΠΎΠ΄ΠΎΠΉ; Π³Π»Π°Π²Π½ΠΎΠ΅, Ρ‡Ρ‚ΠΎΠ±Ρ‹ Ρ€Π°ΡΡ‚Π²ΠΎΡ€ΠΈΡ‚Π΅Π»ΡŒ ΠΏΠΎΠ³Π»ΠΎΡ‚ΠΈΠ» растворяСмоС. ΠŸΡ€ΠΈ Ρ€Π΅Π°ΠΊΡ†ΠΈΠΈ оксидов с Π²ΠΎΠ΄ΠΎΠΉ, ΠΊΠ°ΠΊ извСстно, ΠΏΠΎΠ»ΡƒΡ‡Π°ΡŽΡ‚ΡΡ гидроксиды – Ρ„ΠΎΡ€ΠΌΠ°Π»ΡŒΠ½ΠΎ это просто Π³ΠΈΠ΄Ρ€Π°Ρ‚ΠΈΡ€ΠΎΠ²Π°Π½Π½Ρ‹Π΅ оксиды, Π½ΠΎ Ρ€Π΅Π°Π»ΡŒΠ½ΠΎΡΡ‚ΡŒ Π·Π°Ρ‡Π°ΡΡ‚ΡƒΡŽ прСподносит массу интСрСсного ΠΈ совсСм Π½Π΅Ρ„ΠΎΡ€ΠΌΠ°Π»ΡŒΠ½ΠΎΠ³ΠΎ. Π’ΠΎΡ‚ ΠΈ этот оксид Π°Π·ΠΎΡ‚Π° Π² Π²ΠΎΠ΄Π΅ растворяСтся, ΠΎΠ΄Π½ΠΎΠ²Ρ€Π΅ΠΌΠ΅Π½Π½ΠΎ с Π½Π΅ΠΉ рСагируя, ΠΈ получаСтся ΠΏΡ€ΠΈ этом сразу Π΄Π²Π΅ кислоты!

ΠžΡ‚ΠΌΠ΅Ρ‚ΠΈΠΌ, Ρ‡Ρ‚ΠΎ оксид Π°Π·ΠΎΡ‚Π°(IV) ΠΈΠΌΠ΅Π΅Ρ‚ ΠΈ Ρ…Π°Ρ€Π°ΠΊΡ‚Π΅Ρ€Π½Ρ‹ΠΉ Ρ€Π΅Π·ΠΊΠΈΠΉ Π·Π°ΠΏΠ°Ρ…, ΠΈ Ρ€Ρ‹ΠΆΠ΅Π²Π°Ρ‚ΠΎ-Π±ΡƒΡ€Ρ‹ΠΉ Ρ†Π²Π΅Ρ‚, ΠΎΡ‚Ρ‚Π΅Π½ΠΊΠΈ ΠΊΠΎΡ‚ΠΎΡ€ΠΎΠ³ΠΎ ΠΎΡ‚Π»ΠΈΡ‡Π°ΡŽΡ‚ΡΡ Π΄Ρ€ΡƒΠ³ ΠΎΡ‚ Π΄Ρ€ΡƒΠ³Π° Π² зависимости ΠΎΡ‚ ΠΊΠΎΠ½Ρ†Π΅Π½Ρ‚Ρ€Π°Ρ†ΠΈΠΈ. ИмСнно Π·Π° этот Ρ†Π²Π΅Ρ‚ выбросы оксидов Π°Π·ΠΎΡ‚Π° Π² атмосфСру Π½Π°Π·Ρ‹Π²Π°ΡŽΡ‚ «лисьими хвостами».
Π’Ρ‹ спроситС: Π° Π³Π΄Π΅ ΠΆΠ΅ Ρ‚Π°ΠΉΠ½Π°? ΠŸΠ΅Ρ€Π²Π°Ρ Ρ‡Π°ΡΡ‚ΡŒ Ρ‚Π°ΠΉΠ½Ρ‹ стСпСнСй окислСния ΠΏΠ΅Ρ€Π΅Π΄ Π²Π°ΠΌΠΈ: ΠΏΠΎΡ‡Π΅ΠΌΡƒ Ρƒ элСмСнта пятой (Π½Π΅Ρ‡Π΅Ρ‚Π½ΠΎΠΉ) Π³Ρ€ΡƒΠΏΠΏΡ‹ устойчивы оксиды с Ρ‡Π΅Ρ‚Π½Ρ‹ΠΌΠΈ стСпСнями окислСния? (ΠŸΡ€ΠΈ этом Π΅Ρ‰Π΅ ΠΈ свободныС Ρ€Π°Π΄ΠΈΠΊΠ°Π»Ρ‹!) Π’ Π½Π°ΠΈΠ±ΠΎΠ»Π΅Π΅ ΠΎΠ±Ρ‰Π΅ΠΌ смыслС ΠΎΡ‚Π²Π΅Ρ‚ ΠΎΡ‡Π΅Π²ΠΈΠ΄Π΅Π½ – Ρ€Π°Π· устойчивы, Π·Π½Π°Ρ‡ΠΈΡ‚, ΠΈΠΌ Ρ‚Π°ΠΊ Π²Ρ‹Π³ΠΎΠ΄Π½ΠΎ. ЭнСргСтичСски. А ΠΏΠΎΡ‡Π΅ΠΌΡƒ? Π’ΠΈΠ΄ΠΈΠΌΠΎ, Π΄Π΅Π»ΠΎ Π² спСцификС строСния Π°Ρ‚ΠΎΠΌΠΎΠ² Π°Π·ΠΎΡ‚Π° ΠΈ кислорода – Ρƒ Π½ΠΈΡ… слишком ΠΌΠ½ΠΎΠ³ΠΎ элСктронов ΠΈ слишком ΠΌΠ°Π»ΠΎ ΠΎΡ€Π±ΠΈΡ‚Π°Π»Π΅ΠΉ. ИмСнно Β«ΠΎΡ€Π±ΠΈΡ‚Π°Π»ΡŒΠ½Ρ‹Π΅ возмоТности» Π΄ΠΈΠΊΡ‚ΡƒΡŽΡ‚ свои ΠΏΡ€Π°Π²ΠΈΠ»Π°, ΡƒΡΡ‚Π°Π½Π°Π²Π»ΠΈΠ²Π°ΡŽΡ‚ Ρ‚Π°ΠΊΠΈΠ΅ «энСргСтичСскиС Π²Ρ‹Π³ΠΎΠ΄Ρ‹Β». Π’ΠΎΠ³Π΄Π° Ρ†ΠΈΡ„Ρ€Ρ‹ Β«Π΄Π²Π°Β» ΠΈ Β«Ρ‡Π΅Ρ‚Ρ‹Ρ€Π΅Β» становятся понятными: Π΄Π²ΡƒΡ… элСктронов Π½Π΅ Ρ…Π²Π°Ρ‚Π°Π΅Ρ‚ кислороду Π΄ΠΎ восьми ΠΈ Ρƒ ΠΎΠ±ΠΎΠΈΡ… Π°Ρ‚ΠΎΠΌΠΎΠ² Ρ‚ΠΎΠ»ΡŒΠΊΠΎ ΠΏΠΎ Ρ‡Π΅Ρ‚Ρ‹Ρ€Π΅ ΠΎΡ€Π±ΠΈΡ‚Π°Π»ΠΈ.
А Π΅Ρ‰Π΅ ΠΌΠΎΠΆΠ½ΠΎ ΡΠΊΠ°Π·Π°Ρ‚ΡŒ, Ρ‡Ρ‚ΠΎ NO просто... ΠΆΠ΄Π΅Ρ‚ ΠΌΠΎΠ»Π΅ΠΊΡƒΠ»Ρƒ кислорода, Ρ‡Ρ‚ΠΎΠ±Ρ‹ ΠΏΡ€Π΅Π²Ρ€Π°Ρ‚ΠΈΡ‚ΡŒΡΡ Π² NО 2 . Π˜ΡΠΏΠΎΠ»ΡŒΠ·ΡƒΡ ΠΌΠ΅Ρ‚Π°Ρ„ΠΎΡ€Ρƒ, ΠΎΡ‚ΠΌΠ΅Ρ‚ΠΈΠΌ, Ρ‡Ρ‚ΠΎ «смысл ΠΆΠΈΠ·Π½ΠΈΒ» ΠΌΠ½ΠΎΠ³ΠΈΡ… Π°Ρ‚ΠΎΠΌΠΎΠ² – стрСмлСниС Π½Π°ΠΉΡ‚ΠΈ сСбС Β«ΠΏΠ°Ρ€Ρ‚Π½Π΅Ρ€Π° ΠΏΠΎ ΠΆΠΈΠ·Π½ΠΈΒ» – Π°Ρ‚ΠΎΠΌ ΠΈΠ»ΠΈ Π°Ρ‚ΠΎΠΌΡ‹ Π΄Ρ€ΡƒΠ³ΠΎΠ³ΠΎ элСмСнта. Π₯отя Π΅ΡΡ‚ΡŒ, ΠΊΠΎΠ½Π΅Ρ‡Π½ΠΎ, ΠΈ Β«ΡƒΠ±Π΅ΠΆΠ΄Π΅Π½Π½Ρ‹Π΅ холостяки» Ρ‚ΠΈΠΏΠ° Π·ΠΎΠ»ΠΎΡ‚Π°.

Π₯имичСскиС свойства оксидов Π°Π·ΠΎΡ‚Π°

1. Π Π΅Π°ΠΊΡ†ΠΈΠΈ с ΠΌΠ΅Ρ‚Π°Π»Π»Π°ΠΌΠΈ. ΠŸΠΎΡΠΊΠΎΠ»ΡŒΠΊΡƒ Π°Ρ‚ΠΎΠΌ Π°Π·ΠΎΡ‚Π° Π² ΠΏΠΎΠ»ΠΎΠΆΠΈΡ‚Π΅Π»ΡŒΠ½Ρ‹Ρ… стСпСнях окислСния являСтся окислитСлСм, ΠΏΡ€ΠΈΡ‡Π΅ΠΌ Ρ‡Π΅ΠΌ Π²Ρ‹ΡˆΠ΅ ΡΡ‚Π΅ΠΏΠ΅Π½ΡŒ окислСния, Ρ‚Π΅ΠΌ сильнСС ΡΠΏΠΎΡΠΎΠ±Π½ΠΎΡΡ‚ΡŒ ΠΎΡ‚Π±ΠΈΡ€Π°Ρ‚ΡŒ элСктроны Ρƒ Π΄Ρ€ΡƒΠ³ΠΈΡ… Π°Ρ‚ΠΎΠΌΠΎΠ², Ρ‚ΠΎ с ΠΌΠ΅Ρ‚Π°Π»Π»Π°ΠΌΠΈ – ΠΏΠΎ сути своСй восстановитСлями – оксиды Π°Π·ΠΎΡ‚Π° Π±ΡƒΠ΄ΡƒΡ‚ Ρ€Π΅Π°Π³ΠΈΡ€ΠΎΠ²Π°Ρ‚ΡŒ. ΠŸΡ€ΠΎΠ΄ΡƒΠΊΡ‚Ρ‹ ΠΏΡ€ΠΈ этом ΠΌΠΎΠ³ΡƒΡ‚ ΠΏΠΎΠ»ΡƒΡ‡Π°Ρ‚ΡŒΡΡ совсСм Ρ€Π°Π·Π½Ρ‹Π΅, Π² зависимости ΠΎΡ‚ условий Ρ€Π΅Π°ΠΊΡ†ΠΈΠΈ ΠΈ самого ΠΌΠ΅Ρ‚Π°Π»Π»Π°. НапримСр, раскалСнной ΠΌΠ΅Π΄ΠΈ всС оксиды Π°Π·ΠΎΡ‚Π° ΠΎΡ‚Π΄Π°ΡŽΡ‚ кислород, Π° сами ΠΏΡ€Π΅Π²Ρ€Π°Ρ‰Π°ΡŽΡ‚ΡΡ Π² простоС вСщСство Π°Π·ΠΎΡ‚:

По количСству ΠΎΠ±Ρ€Π°Π·ΠΎΠ²Π°Π²ΡˆΠΈΡ…ΡΡ оксида ΠΌΠ΅Π΄ΠΈ ΠΈ Π°Π·ΠΎΡ‚Π° ΠΌΠΎΠΆΠ½ΠΎ ΡƒΡΡ‚Π°Π½ΠΎΠ²ΠΈΡ‚ΡŒ, ΠΊΠ°ΠΊΠΎΠΉ ΠΈΠΌΠ΅Π½Π½ΠΎ ΠΈΠ· оксидов Π°Π·ΠΎΡ‚Π° Ρ€Π΅Π°Π³ΠΈΡ€ΠΎΠ²Π°Π» с мСдью.

2. Π Π΅Π°ΠΊΡ†ΠΈΠΈ с Π½Π΅ΠΌΠ΅Ρ‚Π°Π»Π»Π°ΠΌΠΈ. Π’ ΠΏΠ΅Ρ€Π²ΡƒΡŽ ΠΎΡ‡Π΅Ρ€Π΅Π΄ΡŒ рассмотрим Ρ€Π΅Π°ΠΊΡ†ΠΈΠΈ с кислородом. Π—Π΄Π΅ΡΡŒ ΠΌΠ΅ΠΆΠ΄Ρƒ оксидами Π½Π°Π±Π»ΡŽΠ΄Π°Π΅Ρ‚ΡΡ Ρ€Π°Π·Π»ΠΈΡ‡ΠΈΠ΅, ΠΈ вСсьма сущСствСнноС.
Оксид NO Ρ€Π΅Π°Π³ΠΈΡ€ΡƒΠ΅Ρ‚ с кислородом с ΠΎΠ±Ρ€Π°Π·ΠΎΠ²Π°Π½ΠΈΠ΅ΠΌ оксида Π°Π·ΠΎΡ‚Π°(IV). РСакция обратимая. ΠŸΡ€ΠΈΡ‡Π΅ΠΌ с ΠΏΠΎΠ²Ρ‹ΡˆΠ΅Π½ΠΈΠ΅ΠΌ Ρ‚Π΅ΠΌΠΏΠ΅Ρ€Π°Ρ‚ΡƒΡ€Ρ‹ ΡΠΊΠΎΡ€ΠΎΡΡ‚ΡŒ этой Ρ€Π΅Π°ΠΊΡ†ΠΈΠΈ ΡƒΠΌΠ΅Π½ΡŒΡˆΠ°Π΅Ρ‚ΡΡ:

2NO + O 2 = 2NO 2 .

Оксид NO 2 с кислородом Π²ΠΎΠΎΠ±Ρ‰Π΅ Π½Π΅ Ρ€Π΅Π°Π³ΠΈΡ€ΡƒΠ΅Ρ‚.
Озон ΠΏΠ΅Ρ€Π΅Π²ΠΎΠ΄ΠΈΡ‚ ΠΎΠ±Π° оксида Π² оксид Π°Π·ΠΎΡ‚Π°(V).
Оксид Π°Π·ΠΎΡ‚Π°(II) NO присоСдиняСт ΠΎΠ·ΠΎΠ½ ΠΏΠΎΠ»Π½ΠΎΡΡ‚ΡŒΡŽ:

2NO + O 3 = N 2 O 5 .

Оксид Π°Π·ΠΎΡ‚Π°(IV) NO 2 Π² Ρ€Π΅Π°ΠΊΡ†ΠΈΠΈ с ΠΎΠ·ΠΎΠ½ΠΎΠΌ выдСляСт Π΅Ρ‰Π΅ ΠΈ кислород:

2NO 2 + O 3 = N 2 O 5 + O 2 .

3. Π Π΅Π°ΠΊΡ†ΠΈΠΈ с Π²ΠΎΠ΄ΠΎΠΉ . Оксид NO с Π²ΠΎΠ΄ΠΎΠΉ Π½Π΅ Ρ€Π΅Π°Π³ΠΈΡ€ΡƒΠ΅Ρ‚. Оксид NO 2 с Π²ΠΎΠ΄ΠΎΠΉ ΠΎΠ±Ρ€Π°Π·ΡƒΠ΅Ρ‚ Π΄Π²Π΅ кислоты – Π°Π·ΠΎΡ‚Π½ΡƒΡŽ (ΡΡ‚Π΅ΠΏΠ΅Π½ΡŒ окислСния Π°Π·ΠΎΡ‚Π° +5) ΠΈ Π°Π·ΠΎΡ‚ΠΈΡΡ‚ΡƒΡŽ (ΡΡ‚Π΅ΠΏΠ΅Π½ΡŒ окислСния Π°Π·ΠΎΡ‚Π° +3). Π’ присутствии кислорода оксид NO 2 Ρ†Π΅Π»ΠΈΠΊΠΎΠΌ ΠΏΠ΅Ρ€Π΅Ρ…ΠΎΠ΄ΠΈΡ‚ Π² Π°Π·ΠΎΡ‚Π½ΡƒΡŽ кислоту:

2NO 2 + Н 2 O = НNО 3 + НNO 2 ,
4NO 2 + O 2 + 2Н 2 О = 4НNО 3 .

4. Π Π΅Π°ΠΊΡ†ΠΈΠΈ c кислотами. Ни ΠΎΠ΄ΠΈΠ½ ΠΈΠ· оксидов – NO ΠΈΠ»ΠΈ NO 2 – Π½Π΅ Ρ€Π΅Π°Π³ΠΈΡ€ΡƒΠ΅Ρ‚ с кислотами.
5. Π Π΅Π°ΠΊΡ†ΠΈΠΈ со Ρ‰Π΅Π»ΠΎΡ‡Π°ΠΌΠΈ . Оба оксида Π°Π·ΠΎΡ‚Π° Ρ€Π΅Π°Π³ΠΈΡ€ΡƒΡŽΡ‚ со Ρ‰Π΅Π»ΠΎΡ‡Π°ΠΌΠΈ.

Оксид NO ΠΎΠ±Ρ€Π°Π·ΡƒΠ΅Ρ‚ с Ρ‰Π΅Π»ΠΎΡ‡ΡŒΡŽ соль азотистой кислоты, оксид Π°Π·ΠΎΡ‚Π°(I) ΠΈ Π°Π·ΠΎΡ‚:

10NO + 6NaOH = 6NaNO 2 + N 2 O + N 2 + 3H 2 O.

Оксид NO 2 ΠΎΠ±Ρ€Π°Π·ΡƒΠ΅Ρ‚ с Ρ‰Π΅Π»ΠΎΡ‡ΡŒΡŽ соли Π΄Π²ΡƒΡ… кислот – Π°Π·ΠΎΡ‚Π½ΠΎΠΉ ΠΈ азотистой:

2NO 2 + 2NaOH = NaNO 3 + NaNO 2 + H 2 O.

ВСрнСмся ΠΊ нашСй Ρ‚Π°ΠΉΠ½Π΅ стСпСнСй окислСния. ΠŸΡ€ΠΈ ΠΏΠ΅Ρ€Π΅Ρ…ΠΎΠ΄Π΅ кислородных соСдинСний Π°Π·ΠΎΡ‚Π° ΠΈΠ· состояния Β«Π³Π°Π·Β», Π³Π΄Π΅ свободно ΠΌΠΎΠΆΠ½ΠΎ Π΄Π²ΠΈΠ³Π°Ρ‚ΡŒΡΡ, Π² состояниС Β«Π²ΠΎΠ΄Π½Ρ‹ΠΉ раствор», Π³Π΄Π΅ большС Ρ‚ΠΎΠ»ΠΊΠΎΡ‚Π½ΠΈ, Π³Π΄Π΅ ΠΏΡ€ΠΎΡ†Π²Π΅Ρ‚Π°Π΅Ρ‚ ΠΊΠΎΠ»Π»Π΅ΠΊΡ‚ΠΈΠ²ΠΈΠ·ΠΌ, Π³Π΄Π΅ ΡΡƒΡ‰Π΅ΡΡ‚Π²ΡƒΡŽΡ‚ ΠΈ Π°ΠΊΡ‚ΠΈΠ²Π½ΠΎ Π΄Π΅ΠΉΡΡ‚Π²ΡƒΡŽΡ‚ полярныС ΠΌΠΎΠ»Π΅ΠΊΡƒΠ»Ρ‹ Π²ΠΎΠ΄Ρ‹, Π½ΠΈΠΊΡ‚ΠΎ ΡƒΠΆΠ΅ Π½Π΅ ΠΏΠΎΠ·Π²ΠΎΠ»ΠΈΡ‚ ΠΌΠΎΠ»Π΅ΠΊΡƒΠ»Π΅, Π°Ρ‚ΠΎΠΌΡƒ ΠΈΠ»ΠΈ ΠΈΠΎΠ½Ρƒ Π±Ρ‹Ρ‚ΡŒ Π² одиночСствС, происходит «смСна ΠΎΡ€ΠΈΠ΅Π½Ρ‚Π°Ρ†ΠΈΠΈΒ». Устойчивыми становятся ΠΊΠ°ΠΊ Ρ€Π°Π· Π½Π΅Ρ‡Π΅Ρ‚Π½Ρ‹Π΅ стСпСни окислСния, ΠΊΠ°ΠΊ ΠΈ ΠΏΠΎΠ»ΠΎΠΆΠ΅Π½ΠΎ элСмСнту ΠΈΠ· Π½Π΅Ρ‡Π΅Ρ‚Π½ΠΎΠΉ Π³Ρ€ΡƒΠΏΠΏΡ‹. (Устойчивыми, ΠΏΡ€Π°Π²Π΄Π°, ΠΎΡ‚Π½ΠΎΡΠΈΡ‚Π΅Π»ΡŒΠ½ΠΎ. Азотистая кислота, Π½Π°ΠΏΡ€ΠΈΠΌΠ΅Ρ€, ΠΌΠΎΠΆΠ΅Ρ‚ ΡΡƒΡ‰Π΅ΡΡ‚Π²ΠΎΠ²Π°Ρ‚ΡŒ Ρ‚ΠΎΠ»ΡŒΠΊΠΎ Π² растворС, ΠΈΠ½Π°Ρ‡Π΅ ΠΎΠ½Π° разлагаСтся. Но кислот, Ρ„ΠΎΡ€ΠΌΠ°Π»ΡŒΠ½ΠΎ ΡΠΎΠΎΡ‚Π²Π΅Ρ‚ΡΡ‚Π²ΡƒΡŽΡ‰ΠΈΡ… оксидам Π°Π·ΠΎΡ‚Π°(II) ΠΈ (IV), Π²ΠΎΠΎΠ±Ρ‰Π΅ Π½Π΅ сущСствуСт. ВсС познаСтся Π² сравнСнии.)
Π˜Π½Ρ‚Π΅Ρ€Π΅ΡΠ½ΠΎ, Ρ‡Ρ‚ΠΎ со Ρ‰Π΅Π»ΠΎΡ‡Π°ΠΌΠΈ Ρ€Π΅Π°Π³ΠΈΡ€ΡƒΠ΅Ρ‚ Π½Π΅ Ρ‚ΠΎΠ»ΡŒΠΊΠΎ явно кислотный оксид NO 2 , Π½ΠΎ ΠΈ NО – ΠΏΠΎ свойствам ΠΈ стСпСни окислСния нСкислотный, ΠΏΡ€ΠΈ этом ΠΏΠΎΠ»ΡƒΡ‡Π°ΡŽΡ‚ΡΡ соСдинСния Π΄Ρ€ΡƒΠ³ΠΈΡ… стСпСнСй окислСния – Π½Π΅Ρ‡Π΅Ρ‚Π½Ρ‹Ρ…! Π’Π°ΠΉΠ½Π°? Π’ΠΏΠΎΠ»Π½Π΅!

Π‘Ρ‚Ρ€ΠΎΠ΅Π½ΠΈΠ΅ ΠΌΠΎΠ»Π΅ΠΊΡƒΠ»Ρ‹ гидроксида Π°Π·ΠΎΡ‚Π°(V) – Π°Π·ΠΎΡ‚Π½ΠΎΠΉ кислоты

Из гидроксидов Π°Π·ΠΎΡ‚Π° ΠΌΡ‹ рассмотрим ΠΎΠ΄ΠΈΠ½, Π½ΠΎ Π½Π°ΠΈΠ±ΠΎΠ»Π΅Π΅ ΠΌΠ½ΠΎΠ³ΠΎΡ‚ΠΎΠ½Π½Π°ΠΆΠ½Ρ‹ΠΉ – Π°Π·ΠΎΡ‚Π½ΡƒΡŽ кислоту.
ΠœΠΎΠ»Π΅ΠΊΡƒΠ»Π° Π°Π·ΠΎΡ‚Π½ΠΎΠΉ кислоты полярна (Π² ΠΏΠ΅Ρ€Π²ΡƒΡŽ ΠΎΡ‡Π΅Ρ€Π΅Π΄ΡŒ ΠΈΠ·-Π·Π° Ρ€Π°Π·Π½ΠΎΠΉ ΡΠ»Π΅ΠΊΡ‚Ρ€ΠΎΠΎΡ‚Ρ€ΠΈΡ†Π°Ρ‚Π΅Π»ΡŒΠ½ΠΎΡΡ‚ΠΈ кислорода ΠΈ Π²ΠΎΠ΄ΠΎΡ€ΠΎΠ΄Π°, ΠΏΠΎΡ‚ΠΎΠΌΡƒ Ρ‡Ρ‚ΠΎ Π°Π·ΠΎΡ‚ ΠΊΠ°ΠΊ Π±Ρ‹ скрыт Π²Π½ΡƒΡ‚Ρ€ΠΈ ΠΌΠΎΠ»Π΅ΠΊΡƒΠ»Ρ‹) ΠΈ асиммСтрична. ВсС Ρ‚Ρ€ΠΈ ΠΈΠΌΠ΅ΡŽΡ‰ΠΈΡ…ΡΡ Π² Π½Π΅ΠΉ ΡƒΠ³Π»Π° ΠΌΠ΅ΠΆΠ΄Ρƒ связями Π°Π·ΠΎΡ‚Π° с кислородом Ρ€Π°Π·Π½Ρ‹Π΅. Π€ΠΎΡ€ΠΌΠ°Π»ΡŒΠ½Π°Ρ ΡΡ‚Π΅ΠΏΠ΅Π½ΡŒ окислСния Π°Π·ΠΎΡ‚Π° Π²Ρ‹ΡΡˆΠ°Ρ, Ρ‚. Π΅. +5. Но ΠΏΡ€ΠΈ этом Ρ‚ΠΎΠ»ΡŒΠΊΠΎ Ρ‡Π΅Ρ‚Ρ‹Ρ€Π΅ связи Ρƒ Π°Ρ‚ΠΎΠΌΠ° Π°Π·ΠΎΡ‚Π° с Π΄Ρ€ΡƒΠ³ΠΈΠΌΠΈ Π°Ρ‚ΠΎΠΌΠ°ΠΌΠΈ – Π²Π°Π»Π΅Π½Ρ‚Π½ΠΎΡΡ‚ΡŒ Π°Π·ΠΎΡ‚Π° Ρ€Π°Π²Π½Π° Ρ‡Π΅Ρ‚Ρ‹Ρ€Π΅ΠΌ. Π•Ρ‰Π΅ ΠΎΠ΄Π½Π° Ρ‚Π°ΠΉΠ½Π°.
ΠŸΠΎΠ½ΡΡ‚Π½ΠΎ, ΠΊΠ°ΠΊ ΠΌΠΎΠΆΠ΅Ρ‚ ΠΏΠΎΠ»ΡƒΡ‡ΠΈΡ‚ΡŒΡΡ Ρ‚Π°ΠΊ, Ρ‡Ρ‚ΠΎ Π²Π°Π»Π΅Π½Ρ‚Π½ΠΎΡΡ‚ΡŒ Π°Ρ‚ΠΎΠΌΠ° числСнно большС Π΅Π³ΠΎ стСпСни окислСния . Для этого достаточно ΠΎΠ±Ρ€Π°Π·ΠΎΠ²Π°Ρ‚ΡŒ Π² ΠΌΠΎΠ»Π΅ΠΊΡƒΠ»Π΅ связь ΠΌΠ΅ΠΆΠ΄Ρƒ ΠΎΠ΄ΠΈΠ½Π°ΠΊΠΎΠ²Ρ‹ΠΌΠΈ Π°Ρ‚ΠΎΠΌΠ°ΠΌΠΈ. НапримСр, Π² пСроксидС Π²ΠΎΠ΄ΠΎΡ€ΠΎΠ΄Π° Ρƒ кислорода Π²Π°Π»Π΅Π½Ρ‚Π½ΠΎΡΡ‚ΡŒ Π΄Π²Π°, Π° ΡΡ‚Π΅ΠΏΠ΅Π½ΡŒ окислСния Ρ‚ΠΎΠ»ΡŒΠΊΠΎ –1. ΠžΠ±Ρ‰ΡƒΡŽ ΡΠ»Π΅ΠΊΡ‚Ρ€ΠΎΠ½Π½ΡƒΡŽ ΠΏΠ°Ρ€Ρƒ связи с Π²ΠΎΠ΄ΠΎΡ€ΠΎΠ΄ΠΎΠΌ кислороду ΡƒΠ΄Π°Π»ΠΎΡΡŒ ΠΏΠΎΠ΄Ρ‚ΡΠ½ΡƒΡ‚ΡŒ ΠΊ сСбС ΠΏΠΎΠ±Π»ΠΈΠΆΠ΅, Π° ΠΏΠ°Ρ€Π° связи Π΄Π²ΡƒΡ… Π°Ρ‚ΠΎΠΌΠΎΠ² кислорода Ρ‚Π°ΠΊ ΠΈ находится строго посСрСдинС. Но ΠΊΠ°ΠΊ ΡΠ΄Π΅Π»Π°Ρ‚ΡŒ Ρ‚Π°ΠΊ, Ρ‡Ρ‚ΠΎΠ±Ρ‹ Π²Π°Π»Π΅Π½Ρ‚Π½ΠΎΡΡ‚ΡŒ Π°Ρ‚ΠΎΠΌΠ° Π±Ρ‹Π»Π° мСньшС стСпСни окислСния ?
Π”Π°Π²Π°ΠΉΡ‚Π΅ ΠΏΠΎΠ΄ΡƒΠΌΠ°Π΅ΠΌ: Π° ΠΊΠ°ΠΊ Π²ΠΎΠΎΠ±Ρ‰Π΅ устроСна ΠΌΠΎΠ»Π΅ΠΊΡƒΠ»Π° Π°Π·ΠΎΡ‚Π½ΠΎΠΉ кислоты? Π‘Ρ‚Ρ€ΠΎΠ΅Π½ΠΈΠ΅ ΠΌΠΎΠ»Π΅ΠΊΡƒΠ»Ρ‹ Π»Π΅Π³Ρ‡Π΅ ΠΏΠΎΠ½ΡΡ‚ΡŒ, Ссли Ρ€Π°ΡΡΠΌΠΎΡ‚Ρ€Π΅Ρ‚ΡŒ процСсс Π΅Π΅ получСния. Азотная кислота получаСтся ΠΏΡ€ΠΈ Ρ€Π΅Π°ΠΊΡ†ΠΈΠΈ оксида Π°Π·ΠΎΡ‚Π°(IV) с Π²ΠΎΠ΄ΠΎΠΉ (Π² присутствии кислорода): Π΄Π²Π΅ ΠΌΠΎΠ»Π΅ΠΊΡƒΠ»Ρ‹ NO 2 ΠΎΠ΄Π½ΠΎΠ²Ρ€Π΅ΠΌΠ΅Π½Π½ΠΎ Β«Π°Ρ‚Π°ΠΊΡƒΡŽΡ‚Β» ΠΌΠΎΠ»Π΅ΠΊΡƒΠ»Ρƒ Π²ΠΎΠ΄Ρ‹ своими нСспарСнными элСктронами, Π² Ρ€Π΅Π·ΡƒΠ»ΡŒΡ‚Π°Ρ‚Π΅ связь Π²ΠΎΠ΄ΠΎΡ€ΠΎΠ΄Π° с кислородом разрываСтся Π½Π΅ ΠΊΠ°ΠΊ ΠΎΠ±Ρ‹Ρ‡Π½ΠΎ (ΠΏΠ°Ρ€Π° элСктронов Ρƒ кислорода ΠΈ Β«Π³ΠΎΠ»Ρ‹ΠΉ ΠΏΡ€ΠΎΡ‚ΠΎΠ½Β»), Π° Β«ΠΏΠΎ-чСстному» – ΠΎΠ΄Π½ΠΎΠΉ ΠΌΠΎΠ»Π΅ΠΊΡƒΠ»Π΅ NO 2 достаСтся Π²ΠΎΠ΄ΠΎΡ€ΠΎΠ΄ со своим элСктроном, Π΄Ρ€ΡƒΠ³ΠΎΠΉ – Ρ€Π°Π΄ΠΈΠΊΠ°Π» ОН (рис. 6). ΠžΠ±Ρ€Π°Π·ΡƒΡŽΡ‚ΡΡ Π΄Π²Π΅ кислоты: ΠΎΠ±Π΅ кислоты ΡΠΈΠ»ΡŒΠ½Ρ‹Π΅, ΠΎΠ±Π΅ быстро ΠΎΡ‚Π΄Π°ΡŽΡ‚ свой ΠΏΡ€ΠΎΡ‚ΠΎΠ½ блиТайшим ΠΌΠΎΠ»Π΅ΠΊΡƒΠ»Π°ΠΌ Π²ΠΎΠ΄Ρ‹ ΠΈ ΠΎΡΡ‚Π°ΡŽΡ‚ΡΡ Π² ΠΈΡ‚ΠΎΠ³Π΅ Π² Π²ΠΈΠ΄Π΅ ΠΈΠΎΠ½ΠΎΠ² ΠΈ . Ион нСстоСк, Π΄Π²Π΅ ΠΌΠΎΠ»Π΅ΠΊΡƒΠ»Ρ‹ НNО 2 Ρ€Π°Π·Π»Π°Π³Π°ΡŽΡ‚ΡΡ Π½Π° Π²ΠΎΠ΄Ρƒ, NО 2 ΠΈ NО. Оксид NO Ρ€Π΅Π°Π³ΠΈΡ€ΡƒΠ΅Ρ‚ с кислородом, ΠΏΡ€Π΅Π²Ρ€Π°Ρ‰Π°ΡΡΡŒ Π² NО 2 , ΠΈ Ρ‚Π°ΠΊ Π΄ΠΎ Ρ‚Π΅Ρ… ΠΏΠΎΡ€, ΠΏΠΎΠΊΠ° Π½Π΅ получится ΠΎΠ΄Π½Π° Ρ‚ΠΎΠ»ΡŒΠΊΠΎ азотная кислота.

Π€ΠΎΡ€ΠΌΠ°Π»ΡŒΠ½ΠΎ Π²Ρ‹Ρ…ΠΎΠ΄ΠΈΡ‚, Ρ‡Ρ‚ΠΎ с ΠΎΠ΄Π½ΠΈΠΌ Π°Ρ‚ΠΎΠΌΠΎΠΌ кислорода Π°Ρ‚ΠΎΠΌ Π°Π·ΠΎΡ‚Π° связан Π΄Π²ΠΎΠΉΠ½ΠΎΠΉ связью, Π° с Π΄Ρ€ΡƒΠ³ΠΈΠΌ – ΠΎΠ±Ρ‹Ρ‡Π½ΠΎΠΉ ΠΎΠ΄ΠΈΠ½Π°Ρ€Π½ΠΎΠΉ связью (этот Π°Ρ‚ΠΎΠΌ кислорода связан Π΅Ρ‰Π΅ ΠΈ с Π°Ρ‚ΠΎΠΌΠΎΠΌ Π²ΠΎΠ΄ΠΎΡ€ΠΎΠ΄Π°). Π‘ Ρ‚Ρ€Π΅Ρ‚ΡŒΠΈΠΌ Π°Ρ‚ΠΎΠΌΠΎΠΌ кислорода Π°Π·ΠΎΡ‚ Π² HNO 3 связан Π΄ΠΎΠ½ΠΎΡ€Π½ΠΎ-Π°ΠΊΡ†Π΅ΠΏΡ‚ΠΎΡ€Π½ΠΎΠΉ связью, ΠΏΡ€ΠΈΡ‡Π΅ΠΌ Π² качСствС Π΄ΠΎΠ½ΠΎΡ€Π° выступаСт Π°Ρ‚ΠΎΠΌ Π°Π·ΠΎΡ‚Π°. Гибридизация Π°Ρ‚ΠΎΠΌΠ° Π°Π·ΠΎΡ‚Π° ΠΏΡ€ΠΈ этом Π΄ΠΎΠ»ΠΆΠ½Π° Π±Ρ‹Ρ‚ΡŒ sΡ€ 2 ΠΈΠ·-Π·Π° наличия Π΄Π²ΠΎΠΉΠ½ΠΎΠΉ связи, Ρ‡Ρ‚ΠΎ опрСдСляСт структуру – плоский Ρ‚Ρ€Π΅ΡƒΠ³ΠΎΠ»ΡŒΠ½ΠΈΠΊ. РСально получаСтся, Ρ‡Ρ‚ΠΎ Π΄Π΅ΠΉΡΡ‚Π²ΠΈΡ‚Π΅Π»ΡŒΠ½ΠΎ Ρ„Ρ€Π°Π³ΠΌΠ΅Π½Ρ‚ ΠΈΠ· Π°Ρ‚ΠΎΠΌΠ° Π°Π·ΠΎΡ‚Π° ΠΈ Ρ‚Ρ€Π΅Ρ… Π°Ρ‚ΠΎΠΌΠΎΠ² кислорода – плоский Ρ‚Ρ€Π΅ΡƒΠ³ΠΎΠ»ΡŒΠ½ΠΈΠΊ, Ρ‚ΠΎΠ»ΡŒΠΊΠΎ Π² ΠΌΠΎΠ»Π΅ΠΊΡƒΠ»Π΅ Π°Π·ΠΎΡ‚Π½ΠΎΠΉ кислоты этот Ρ‚Ρ€Π΅ΡƒΠ³ΠΎΠ»ΡŒΠ½ΠΈΠΊ Π½Π΅ΠΏΡ€Π°Π²ΠΈΠ»ΡŒΠ½Ρ‹ΠΉ – всС Ρ‚Ρ€ΠΈ ΡƒΠ³Π»Π° ОNО Ρ€Π°Π·Π½Ρ‹Π΅, ΡΠ»Π΅Π΄ΠΎΠ²Π°Ρ‚Π΅Π»ΡŒΠ½ΠΎ, ΠΈ Ρ€Π°Π·Π½Ρ‹Π΅ стороны Ρ‚Ρ€Π΅ΡƒΠ³ΠΎΠ»ΡŒΠ½ΠΈΠΊΠ°. Когда ΠΆΠ΅ ΠΌΠΎΠ»Π΅ΠΊΡƒΠ»Π° диссоциируСт, Ρ‚Ρ€Π΅ΡƒΠ³ΠΎΠ»ΡŒΠ½ΠΈΠΊ становится ΠΏΡ€Π°Π²ΠΈΠ»ΡŒΠ½Ρ‹ΠΌ, равносторонним. Π—Π½Π°Ρ‡ΠΈΡ‚, ΠΈ Π°Ρ‚ΠΎΠΌΡ‹ кислорода Π² Π½Π΅ΠΌ становятся Ρ€Π°Π²Π½ΠΎΡ†Π΅Π½Π½Ρ‹ΠΌΠΈ! ΠžΠ΄ΠΈΠ½Π°ΠΊΠΎΠ²Ρ‹ΠΌΠΈ становятся ΠΈ всС связи (двойная связь ΠΊΠΎΡ€ΠΎΡ‡Π΅ ΠΎΠ΄ΠΈΠ½Π°Ρ€Π½ΠΎΠΉ). Каким ΠΎΠ±Ρ€Π°Π·ΠΎΠΌ?
Π”Π°Π²Π°ΠΉΡ‚Π΅ Ρ€Π°ΡΡΡƒΠΆΠ΄Π°Ρ‚ΡŒ. sΡ€ 2 -Гибридизация Π°Ρ‚ΠΎΠΌΠ° Π°Π·ΠΎΡ‚Π° ΠΏΡ€ΠΈΠ½ΡƒΠΆΠ΄Π°Π΅Ρ‚ ΠΊ Ρ‚Π°ΠΊΠΎΠΌΡƒ ΠΆΠ΅ Ρ‚ΠΈΠΏΡƒ Π³ΠΈΠ±Ρ€ΠΈΠ΄ΠΈΠ·Π°Ρ†ΠΈΠΈ ΠΈ Π°Ρ‚ΠΎΠΌΡ‹ кислорода. ΠŸΠΎΠ»ΡƒΡ‡Π°Π΅Ρ‚ΡΡ плоская структура, ΠΏΠΎΠΏΠ΅Ρ€Π΅ΠΊ ΠΊΠΎΡ‚ΠΎΡ€ΠΎΠΉ располоТСны Π½Π΅ ΡƒΡ‡Π°ΡΡ‚Π²ΡƒΡŽΡ‰ΠΈΠ΅ Π² Π³ΠΈΠ±Ρ€ΠΈΠ΄ΠΈΠ·Π°Ρ†ΠΈΠΈ Ρ€-ΠΎΡ€Π±ΠΈΡ‚Π°Π»ΠΈ, ΠΈΠΌΠ΅ΡŽΡ‰ΠΈΠ΅ΡΡ Ρƒ всСх Ρ‡Π΅Ρ‚Ρ‹Ρ€Π΅Ρ… Π°Ρ‚ΠΎΠΌΠΎΠ².
Π’Π΅ΠΏΠ΅Ρ€ΡŒ разбСрСмся с ΠΎΠ±Ρ‰ΠΈΠΌ количСством Π²Π°Π»Π΅Π½Ρ‚Π½Ρ‹Ρ… элСктронов: ΠΈΠΎΠ½ содСрТит ΠΏΡΡ‚ΡŒ элСктронов Π°Π·ΠΎΡ‚Π°, ΠΏΠΎ ΡˆΠ΅ΡΡ‚ΡŒ Ρƒ Ρ‚Ρ€Π΅Ρ… Π°Ρ‚ΠΎΠΌΠΎΠ² кислорода ΠΈ Π΅Ρ‰Π΅ ΠΎΠ΄ΠΈΠ½, ΠΏΡ€ΠΈΠ΄Π°ΡŽΡ‰ΠΈΠΉ заряд ΠΈΠΎΠ½Ρƒ Π² Ρ†Π΅Π»ΠΎΠΌ, ΠΈΡ‚ΠΎΠ³ΠΎ Π΄Π²Π°Π΄Ρ†Π°Ρ‚ΡŒ Ρ‡Π΅Ρ‚Ρ‹Ρ€Π΅. Из Π½ΠΈΡ… ΡˆΠ΅ΡΡ‚ΡŒ элСктронов трСбуСтся для образования Ρ‚Ρ€Π΅Ρ… ΠΎΠ΄ΠΈΠ½Π°Ρ€Π½Ρ‹Ρ… связСй, Π΄Π²Π΅Π½Π°Π΄Ρ†Π°Ρ‚ΡŒ элСктронов располоТСны ΠΏΠΎ ΠΏΠ΅Ρ€ΠΈΠΌΠ΅Ρ‚Ρ€Ρƒ ΠΌΠΎΠ»Π΅ΠΊΡƒΠ»Ρ‹ Π½Π° Π³ΠΈΠ±Ρ€ΠΈΠ΄Π½Ρ‹Ρ… орбиталях (ΠΏΠΎ Π΄Π²Π΅ элСктронныС ΠΏΠ°Ρ€Ρ‹ Ρƒ ΠΊΠ°ΠΆΠ΄ΠΎΠ³ΠΎ Π°Ρ‚ΠΎΠΌΠ° кислорода), остаСтся ΡˆΠ΅ΡΡ‚ΡŒ элСктронов Π½Π° Ρ‡Π΅Ρ‚Ρ‹Ρ€Π΅ Ρ‚Π΅ самыС Ρ€ -ΠΎΡ€Π±ΠΈΡ‚Π°Π»ΠΈ, Π½Π΅ ΡƒΡ‡Π°ΡΡ‚Π²ΡƒΡŽΡ‰ΠΈΠ΅ Π² Π³ΠΈΠ±Ρ€ΠΈΠ΄ΠΈΠ·Π°Ρ†ΠΈΠΈ. ЕдинствСнноС Π²ΠΎΠ·ΠΌΠΎΠΆΠ½ΠΎΠ΅ Π² Π΄Π°Π½Π½ΠΎΠΌ случаС Ρ€Π°Π·ΡƒΠΌΠ½ΠΎΠ΅ объяснСниС – это обобщСствлСниС всСми Π°Ρ‚ΠΎΠΌΠ°ΠΌΠΈ своих элСктронов Π² Π΅Π΄ΠΈΠ½ΠΎΠ΅ элСктронноС ΠΎΠ±Π»Π°ΠΊΠΎ (рис. 7). Π­Ρ‚ΠΎΠΌΡƒ ΡΠΏΠΎΡΠΎΠ±ΡΡ‚Π²ΡƒΡŽΡ‚ ΠΌΠ°Π»Ρ‹Π΅ радиусы Π°Ρ‚ΠΎΠΌΠΎΠ² ΠΈ ΠΌΠ°Π»Ρ‹Π΅ ΠΌΠ΅ΠΆΠ°Ρ‚ΠΎΠΌΠ½Ρ‹Π΅ расстояния. А симмСтрия ΠΎΠ±Ρ‹Ρ‡Π½ΠΎ энСргСтичСски Π²Ρ‹Π³ΠΎΠ΄Π½Π° ΠΈ поэтому ΠΏΠΎΠ²Ρ‹ΡˆΠ°Π΅Ρ‚ ΡƒΡΡ‚ΠΎΠΉΡ‡ΠΈΠ²ΠΎΡΡ‚ΡŒ конструкции Π² Ρ†Π΅Π»ΠΎΠΌ. Π­Ρ‚ΠΎ Π½Π΅ СдинствСнный случай обобщСствлСния элСктронов нСсколькими Π°Ρ‚ΠΎΠΌΠ°ΠΌΠΈ, ΠΏΠΎΡ…ΠΎΠΆΠ΅Π΅ Β«ΠΊΠΎΠ»Π»Π΅ΠΊΡ‚ΠΈΠ²Π½ΠΎΠ΅ элСктронноС хозяйство» встрСчаСтся Π² органичСской Ρ…ΠΈΠΌΠΈΠΈ, Π½Π°ΠΏΡ€ΠΈΠΌΠ΅Ρ€ Π² ароматичСских соСдинСниях.

ВСрнСмся, ΠΎΠ΄Π½Π°ΠΊΠΎ, ΠΊ прСдсказаниям свойств Π°Π·ΠΎΡ‚Π½ΠΎΠΉ кислоты, исходя ΠΈΠ· прСдставлСний ΠΎ структурС ΠΌΠΎΠ»Π΅ΠΊΡƒΠ»Ρ‹. ΠžΡ‡Π΅Π²ΠΈΠ΄Π½Π°Ρ Π²Ρ‹Π³ΠΎΠ΄Π½ΠΎΡΡ‚ΡŒ прСбывания Π² Π²ΠΈΠ΄Π΅ ΠΈΠΎΠ½Π° ΠΎΠ±ΡŠΡΡΠ½ΡΠ΅Ρ‚ Π½Π΅ Ρ‚ΠΎΠ»ΡŒΠΊΠΎ Π²Ρ‹ΡΠΎΠΊΡƒΡŽ ΡΡ‚Π΅ΠΏΠ΅Π½ΡŒ диссоциации кислоты Π² Π²ΠΎΠ΄Π½ΠΎΠΌ растворС, Π½ΠΎ ΠΈ Π²ΠΎΠ·ΠΌΠΎΠΆΠ½ΠΎΡΡ‚ΡŒ диссоциации Π±Π΅Π·Π²ΠΎΠ΄Π½ΠΎΠΉ кислоты. И ΠΈΠΌΠ΅Π½Π½ΠΎ диссоциация опрСдСляСт физичСскиС свойства этого вСщСства.

ЀизичСскиС свойства Π°Π·ΠΎΡ‚Π½ΠΎΠΉ кислоты

Π‘ΠΎΠ΅Π΄ΠΈΠ½Π΅Π½ΠΈΠ΅ ΠΈΠΎΠ½ΠΈΠ·ΠΈΡ€ΠΎΠ²Π°Π½Π½ΠΎΠ΅, ΠΏΡƒΡΡ‚ΡŒ Π΄Π°ΠΆΠ΅ ΠΈ частично, слоТно пСрСвСсти Π² Π³Π°Π·. Π’Π°ΠΊΠΈΠΌ ΠΎΠ±Ρ€Π°Π·ΠΎΠΌ, Ρ‚Π΅ΠΌΠΏΠ΅Ρ€Π°Ρ‚ΡƒΡ€Π° кипСния Π΄ΠΎΠ»ΠΆΠ½Π° Π±Ρ‹ Π±Ρ‹Ρ‚ΡŒ достаточно высокой, ΠΎΠ΄Π½Π°ΠΊΠΎ ΠΏΡ€ΠΈ Ρ‚Π°ΠΊΠΎΠΉ нСбольшой молСкулярной массС (ΠΈ ΠΈΠ·-Π·Π° высокой подвиТности) Ρ‚Π΅ΠΌΠΏΠ΅Ρ€Π°Ρ‚ΡƒΡ€Π° плавлСния высокой Π±Ρ‹Ρ‚ΡŒ Π½Π΅ Π΄ΠΎΠ»ΠΆΠ½Π°. Π‘Π»Π΅Π΄ΠΎΠ²Π°Ρ‚Π΅Π»ΡŒΠ½ΠΎ, Π°Π³Ρ€Π΅Π³Π°Ρ‚Π½ΠΎΠ΅ состояниС ΠΏΡ€ΠΈ 20 Β°Π‘ – ΠΆΠΈΠ΄ΠΊΠΎΠ΅.
Π§Ρ‚ΠΎ касаСтся растворимости Π² Π²ΠΎΠ΄Π΅, Ρ‚ΠΎ, ΠΊΠ°ΠΊ ΠΈ ΠΌΠ½ΠΎΠ³ΠΈΠ΅ Π΄Ρ€ΡƒΠ³ΠΈΠ΅ полярныС Тидкости, азотная кислота Π»Π΅Π³ΠΊΠΎ ΡΠΌΠ΅ΡˆΠΈΠ²Π°Π΅Ρ‚ΡΡ с Π²ΠΎΠ΄ΠΎΠΉ Π² Π»ΡŽΠ±Ρ‹Ρ… ΡΠΎΠΎΡ‚Π½ΠΎΡˆΠ΅Π½ΠΈΡΡ….
Чистая азотная кислота бСсцвСтна ΠΈ Π½Π΅ ΠΈΠΌΠ΅Π΅Ρ‚ Π·Π°ΠΏΠ°Ρ…Π°. Однако ΠΈΠ·-Π·Π° разлоТСния Π½Π° кислород ΠΈ оксид Π°Π·ΠΎΡ‚Π°(IV), ΠΊΠΎΡ‚ΠΎΡ€Ρ‹ΠΉ Π² Π½Π΅ΠΉ ΠΆΠ΅ ΠΈ растворяСтся, ΠΌΠΎΠΆΠ½ΠΎ ΡΠΊΠ°Π·Π°Ρ‚ΡŒ, Ρ‡Ρ‚ΠΎ обычная концСнтрированная азотная кислота ΠΈΠΌΠ΅Π΅Ρ‚ ΠΆΠ΅Π»Ρ‚ΠΎΠ±ΡƒΡ€Ρ‹ΠΉ Ρ†Π²Π΅Ρ‚ ΠΈ Ρ…Π°Ρ€Π°ΠΊΡ‚Π΅Ρ€Π½Ρ‹ΠΉ для NO 2 Ρ€Π΅Π·ΠΊΠΈΠΉ Π·Π°ΠΏΠ°Ρ….
ΠŸΠΎΡΠΌΠΎΡ‚Ρ€ΠΈΠΌ, ΠΊΠ°ΠΊ влияСт строСниС ΠΌΠΎΠ»Π΅ΠΊΡƒΠ»Ρ‹ Π°Π·ΠΎΡ‚Π½ΠΎΠΉ кислоты Π½Π° Π΅Π΅ химичСскиС свойства.

Π₯имичСскиС свойства Π°Π·ΠΎΡ‚Π½ΠΎΠΉ кислоты

Π“Π»Π°Π²Π½ΠΎΠ΅, Ρ‡Ρ‚ΠΎ ΠΌΡ‹ Π΄ΠΎΠ»ΠΆΠ½Ρ‹ ΠΎΡ‚ΠΌΠ΅Ρ‚ΠΈΡ‚ΡŒ, – Π½Π°Π»ΠΈΡ‡ΠΈΠ΅ Π²Ρ‹ΡΡˆΠ΅ΠΉ стСпСни окислСния Π°Ρ‚ΠΎΠΌΠ° Π°Π·ΠΎΡ‚Π° ΠΎΠ³Ρ€Π°Π½ΠΈΡ‡ΠΈΠ²Π°Π΅Ρ‚ свойства Π°Π·ΠΎΡ‚Π½ΠΎΠΉ кислоты, с окислитСлями ΠΎΠ½Π° Π½Π΅ Ρ€Π΅Π°Π³ΠΈΡ€ΡƒΠ΅Ρ‚. Π—Π°Ρ‚ΠΎ с восстановитСлями, Π² ΠΏΠ΅Ρ€Π²ΡƒΡŽ ΠΎΡ‡Π΅Ρ€Π΅Π΄ΡŒ с ΠΌΠ΅Ρ‚Π°Π»Π»Π°ΠΌΠΈ, ΠΎΠ½Π° Ρ€Π΅Π°Π³ΠΈΡ€ΡƒΠ΅Ρ‚ Π½Π΅Ρ‚Ρ€Π°Π΄ΠΈΡ†ΠΈΠΎΠ½Π½ΠΎ ΠΈ Ρ€Π°Π·Π½ΠΎΠΎΠ±Ρ€Π°Π·Π½ΠΎ.
1. Π Π΅Π°ΠΊΡ†ΠΈΠΈ с ΠΌΠ΅Ρ‚Π°Π»Π»Π°ΠΌΠΈ . Π‘ ΠΌΠ΅Ρ‚Π°Π»Π»Π°ΠΌΠΈ азотная кислота Ρ€Π΅Π°Π³ΠΈΡ€ΡƒΠ΅Ρ‚ ΠΊΠ°ΠΊ ΡΠΈΠ»ΡŒΠ½Ρ‹ΠΉ ΠΎΠΊΠΈΡΠ»ΠΈΡ‚Π΅Π»ΡŒ Π΄Π°ΠΆΠ΅ Π² Ρ€Π°Π·Π±Π°Π²Π»Π΅Π½Π½Ρ‹Ρ… растворах (Π² ΠΎΡ‚Π»ΠΈΡ‡ΠΈΠ΅ ΠΎΡ‚ сСрной кислоты, ΠΏΡ€ΠΎΡΠ²Π»ΡΡŽΡ‰Π΅ΠΉ свои ΠΎΠΊΠΈΡΠ»ΠΈΡ‚Π΅Π»ΡŒΠ½Ρ‹Π΅ свойства Ρ‚ΠΎΠ»ΡŒΠΊΠΎ Π² ΠΊΠΎΠ½Ρ†Π΅Π½Ρ‚Ρ€ΠΈΡ€ΠΎΠ²Π°Π½Π½ΠΎΠΌ Π²ΠΈΠ΄Π΅). ΠžΠ±Ρ€Π°Π·ΡƒΠ΅Ρ‚ΡΡ ΠΎΠ±Ρ‹Ρ‡Π½ΠΎ Π½ΠΈΡ‚Ρ€Π°Ρ‚ ΠΌΠ΅Ρ‚Π°Π»Π»Π°, Π½ΠΎ вмСсто Π²ΠΎΠ΄ΠΎΡ€ΠΎΠ΄Π° Π²Ρ‹Π΄Π΅Π»ΡΡŽΡ‚ΡΡ Π³Π°Π·ΠΎΠΎΠ±Ρ€Π°Π·Π½Ρ‹Π΅ соСдинСния Π°Π·ΠΎΡ‚Π°: NО 2 , NО, N 2 O, N 2 ΠΈΠ»ΠΈ Π°ΠΌΠΌΠΈΠ°ΠΊ, ΠΊΠΎΡ‚ΠΎΡ€Ρ‹ΠΉ Π² кислой срСдС сразу ΠΏΠ΅Ρ€Π΅Ρ…ΠΎΠ΄ΠΈΡ‚ Π² ΠΈΠΎΠ½ аммония. Π’ ΠΏΡ€ΠΈΠ½Ρ†ΠΈΠΏΠ΅ ΠΏΡ€ΠΈ Ρ€Π΅Π°ΠΊΡ†ΠΈΠΈ ΠΌΠ΅Ρ‚Π°Π»Π»Π° с Π°Π·ΠΎΡ‚Π½ΠΎΠΉ кислотой образуСтся вСсь этот Β«Π±ΡƒΠΊΠ΅Ρ‚Β» Π³Π°Π·ΠΎΠ², Π½ΠΎ Π² зависимости ΠΎΡ‚ ΠΌΠ΅Ρ‚Π°Π»Π»Π° ΠΈ ΠΊΠΎΠ½Ρ†Π΅Π½Ρ‚Ρ€Π°Ρ†ΠΈΠΈ кислоты Ρ‚Π΅ ΠΈΠ»ΠΈ ΠΈΠ½Ρ‹Π΅ ΠΊΠΎΠΌΠΏΠΎΠ½Π΅Π½Ρ‚Ρ‹ Π±ΡƒΠ΄ΡƒΡ‚ ΠΏΡ€Π΅Π²Π°Π»ΠΈΡ€ΠΎΠ²Π°Ρ‚ΡŒ.
Π’Π°ΠΊ, Π² Π»Π°Π±ΠΎΡ€Π°Ρ‚ΠΎΡ€Π½Ρ‹Ρ… условиях оксид Π°Π·ΠΎΡ‚Π°(II) ΠΎΠ±Ρ‹Ρ‡Π½ΠΎ ΠΏΠΎΠ»ΡƒΡ‡Π°ΡŽΡ‚ ΠΏΡ€ΠΈ взаимодСйствии ΠΌΠ΅Π΄Π½Ρ‹Ρ… струТСк с Π°Π·ΠΎΡ‚Π½ΠΎΠΉ кислотой ΠΏΠ»ΠΎΡ‚Π½ΠΎΡΡ‚ΡŒΡŽ 1,2 Π³/см 3 , Ρ‚. Π΅. ΠΏΡ€ΠΈ ΠΎΠ±Ρ€Π°Π±ΠΎΡ‚ΠΊΠ΅ ΠΌΠ΅Π΄ΠΈ Ρ€Π°Π·Π±Π°Π²Π»Π΅Π½Π½ΠΎΠΉ кислотой Π² Π³Π°Π·ΠΎΠΎΠ±Ρ€Π°Π·Π½Ρ‹Ρ… ΠΏΡ€ΠΎΠ΄ΡƒΠΊΡ‚Π°Ρ… Ρ€Π΅Π°ΠΊΡ†ΠΈΠΈ явно ΠΏΡ€Π΅Π²Π°Π»ΠΈΡ€ΡƒΠ΅Ρ‚ ΠΈΠΌΠ΅Π½Π½ΠΎ этот оксид:

А Π²ΠΎΡ‚ ΠΏΡ€ΠΈ Ρ€Π΅Π°ΠΊΡ†ΠΈΠΈ Π°Π·ΠΎΡ‚Π½ΠΎΠΉ кислоты Ρ‚ΠΎΠΉ ΠΆΠ΅ плотности (Π° ΡΠ»Π΅Π΄ΠΎΠ²Π°Ρ‚Π΅Π»ΡŒΠ½ΠΎ, ΠΈ ΠΊΠΎΠ½Ρ†Π΅Π½Ρ‚Ρ€Π°Ρ†ΠΈΠΈ) с ΠΆΠ΅Π»Π΅Π·ΠΎΠΌ содСрТаниС оксида Π°Π·ΠΎΡ‚Π°(II) Π² смСси составляСт Ρ‚ΠΎΠ»ΡŒΠΊΠΎ 40% – мСньшС ΠΏΠΎΠ»ΠΎΠ²ΠΈΠ½Ρ‹, Π° ΠΎΡΡ‚Π°Π»ΡŒΠ½Ρ‹Π΅ 60% Ρ€Π°Π²Π½ΠΎΠΌΠ΅Ρ€Π½ΠΎ распрСдСлСны ΠΌΠ΅ΠΆΠ΄Ρƒ Π½ΠΈΡ‚Ρ€Π°Ρ‚ΠΎΠΌ аммония, Π°Π·ΠΎΡ‚ΠΎΠΌ, оксидом Π°Π·ΠΎΡ‚Π°(I) ΠΈ оксидом Π°Π·ΠΎΡ‚Π°(IV) (рис. 8).

Π‘Π»Π΅Π΄ΡƒΠ΅Ρ‚ ΠΎΡ‚ΠΌΠ΅Ρ‚ΠΈΡ‚ΡŒ Ρ‚Π°ΠΊΠΎΠΉ интСрСсный ΠΈ ΠΆΠΈΠ·Π½Π΅Π½Π½ΠΎ Π²Π°ΠΆΠ½Ρ‹ΠΉ Ρ„Π°ΠΊΡ‚, Ρ‡Ρ‚ΠΎ со 100%-ΠΉ Π°Π·ΠΎΡ‚Π½ΠΎΠΉ кислотой Π½ΠΈ ΠΆΠ΅Π»Π΅Π·ΠΎ, Π½ΠΈ алюминий Π½Π΅ Ρ€Π΅Π°Π³ΠΈΡ€ΡƒΡŽΡ‚ (ΡΠ»Π΅Π΄ΠΎΠ²Π°Ρ‚Π΅Π»ΡŒΠ½ΠΎ, Π΅Π΅ ΠΌΠΎΠΆΠ½ΠΎ Ρ…Ρ€Π°Π½ΠΈΡ‚ΡŒ ΠΈ ΠΏΠ΅Ρ€Π΅Π²ΠΎΠ·ΠΈΡ‚ΡŒ Π² цистСрнах ΠΈ Π΄Ρ€ΡƒΠ³ΠΈΡ… Смкостях ΠΈΠ· этих ΠΌΠ΅Ρ‚Π°Π»Π»ΠΎΠ²). Π”Π΅Π»ΠΎ Π² Ρ‚ΠΎΠΌ, Ρ‡Ρ‚ΠΎ эти ΠΌΠ΅Ρ‚Π°Π»Π»Ρ‹ ΠΏΠΎΠΊΡ€Ρ‹Ρ‚Ρ‹ ΠΏΡ€ΠΎΡ‡Π½Ρ‹ΠΌΠΈ ΠΏΠ»Π΅Π½ΠΊΠ°ΠΌΠΈ оксидов, нСрастворимых Π² чистой кислотС. Для проявлСния кислотных свойств Π½ΡƒΠΆΠ½ΠΎ, Ρ‡Ρ‚ΠΎΠ±Ρ‹ кислота Π·Π°ΠΌΠ΅Ρ‚Π½ΠΎ диссоциировала, Π° для этого Π² свою ΠΎΡ‡Π΅Ρ€Π΅Π΄ΡŒ трСбуСтся Π²ΠΎΠ΄Π°.
2. Π Π΅Π°ΠΊΡ†ΠΈΠΈ с Π½Π΅ΠΌΠ΅Ρ‚Π°Π»Π»Π°ΠΌΠΈ. Π‘ кислородом ΠΈ ΠΎΠ·ΠΎΠ½ΠΎΠΌ азотная кислота Π½Π΅ Ρ€Π΅Π°Π³ΠΈΡ€ΡƒΠ΅Ρ‚.
3. РСакция с Π²ΠΎΠ΄ΠΎΠΉ Π½Π΅ происходит. Π’ΠΎΠ΄Π° лишь способствуСт диссоциации кислоты.
4. Π Π΅Π°ΠΊΡ†ΠΈΠΈ с кислотами. Азотная кислота Π½Π΅ Ρ€Π΅Π°Π³ΠΈΡ€ΡƒΠ΅Ρ‚ с Π΄Ρ€ΡƒΠ³ΠΈΠΌΠΈ кислотами ΠΏΠΎ Ρ‚ΠΈΠΏΡƒ Ρ€Π΅Π°ΠΊΡ†ΠΈΠΉ ΠΎΠ±ΠΌΠ΅Π½Π° ΠΈΠ»ΠΈ соСдинСния. Однако Π²ΠΏΠΎΠ»Π½Π΅ способна Ρ€Π΅Π°Π³ΠΈΡ€ΠΎΠ²Π°Ρ‚ΡŒ ΠΊΠ°ΠΊ ΡΠΈΠ»ΡŒΠ½Ρ‹ΠΉ ΠΎΠΊΠΈΡΠ»ΠΈΡ‚Π΅Π»ΡŒ. Π’ смСси ΠΊΠΎΠ½Ρ†Π΅Π½Ρ‚Ρ€ΠΈΡ€ΠΎΠ²Π°Π½Π½Ρ‹Ρ… Π°Π·ΠΎΡ‚Π½ΠΎΠΉ ΠΈ соляной кислот ΠΏΡ€ΠΎΡ‚Π΅ΠΊΠ°ΡŽΡ‚ ΠΎΠ±Ρ€Π°Ρ‚ΠΈΠΌΡ‹Π΅ Ρ€Π΅Π°ΠΊΡ†ΠΈΠΈ, ΡΡƒΡ‚ΡŒ ΠΊΠΎΡ‚ΠΎΡ€Ρ‹Ρ… ΠΌΠΎΠΆΠ½ΠΎ ΠΎΠ±ΠΎΠ±Ρ‰ΠΈΡ‚ΡŒ ΡƒΡ€Π°Π²Π½Π΅Π½ΠΈΠ΅ΠΌ:

ΠžΠ±Ρ€Π°Π·ΡƒΡŽΡ‰ΠΈΠΉΡΡ Π°Ρ‚ΠΎΠΌΠ°Ρ€Π½Ρ‹ΠΉ Ρ…Π»ΠΎΡ€ ΠΎΡ‡Π΅Π½ΡŒ Π°ΠΊΡ‚ΠΈΠ²Π΅Π½ ΠΈ Π»Π΅Π³ΠΊΠΎ ΠΎΡ‚Π±ΠΈΡ€Π°Π΅Ρ‚ элСктроны Ρƒ Π°Ρ‚ΠΎΠΌΠΎΠ² ΠΌΠ΅Ρ‚Π°Π»Π»ΠΎΠ², Π° ΠΈΠΌΠ΅ΡŽΡ‰ΠΈΠΉΡΡ Β«ΠΏΠΎΠ΄ Π±ΠΎΠΊΠΎΠΌΒ» Ρ…Π»ΠΎΡ€ΠΈΠ΄-ΠΈΠΎΠ½ ΠΎΠ±Ρ€Π°Π·ΡƒΠ΅Ρ‚ устойчивыС комплСксныС ΠΈΠΎΠ½Ρ‹ с ΠΏΠΎΠ»ΡƒΡ‡Π°ΡŽΡ‰ΠΈΠΌΠΈΡΡ ΠΈΠΎΠ½Π°ΠΌΠΈ ΠΌΠ΅Ρ‚Π°Π»Π»ΠΎΠ². ВсС это позволяСт пСрСвСсти Π² раствор Π΄Π°ΠΆΠ΅ Π·ΠΎΠ»ΠΎΡ‚ΠΎ. По Ρ‚ΠΎΠΉ ΠΏΡ€ΠΈΡ‡ΠΈΠ½Π΅, Ρ‡Ρ‚ΠΎ Π·ΠΎΠ»ΠΎΡ‚ΠΎ – Β«Ρ†Π°Ρ€ΡŒ ΠΌΠ΅Ρ‚Π°Π»Π»ΠΎΠ²Β», смСсь ΠΊΠΎΠ½Ρ†Π΅Π½Ρ‚Ρ€ΠΈΡ€ΠΎΠ²Π°Π½Π½Ρ‹Ρ… Π°Π·ΠΎΡ‚Π½ΠΎΠΉ ΠΈ соляной кислот Π½Π°Π·Ρ‹Π²Π°ΡŽΡ‚ царской Π²ΠΎΠ΄ΠΊΠΎΠΉ.
ΠšΠΎΠ½Ρ†Π΅Π½Ρ‚Ρ€ΠΈΡ€ΠΎΠ²Π°Π½Π½Π°Ρ сСрная кислота ΠΊΠ°ΠΊ сильноС Π²ΠΎΠ΄ΠΎΠΎΡ‚Π½ΠΈΠΌΠ°ΡŽΡ‰Π΅Π΅ срСдство способствуСт Ρ€Π΅Π°ΠΊΡ†ΠΈΠΈ разлоТСния Π°Π·ΠΎΡ‚Π½ΠΎΠΉ кислоты Π½Π° оксид Π°Π·ΠΎΡ‚Π°(IV) ΠΈ кислород.
5. Π Π΅Π°ΠΊΡ†ΠΈΠΈ с основаниями ΠΈ с основными оксидами . Азотная кислота – ΠΎΠ΄Π½Π° ΠΈΠ· ΡΠΈΠ»ΡŒΠ½Ρ‹Ρ… нСорганичСских кислот ΠΈ, СстСствСнно, со Ρ‰Π΅Π»ΠΎΡ‡Π°ΠΌΠΈ Ρ€Π΅Π°Π³ΠΈΡ€ΡƒΠ΅Ρ‚. Π Π΅Π°Π³ΠΈΡ€ΡƒΠ΅Ρ‚ ΠΎΠ½Π° Ρ‚Π°ΠΊΠΆΠ΅ ΠΈ с нСрастворимыми гидроксидами, ΠΈ с основными оксидами. Π­Ρ‚ΠΈΠΌ рСакциям способствуСт Π΅Ρ‰Π΅ ΠΈ Ρ‚ΠΎ ΠΎΠ±ΡΡ‚ΠΎΡΡ‚Π΅Π»ΡŒΡΡ‚Π²ΠΎ, Ρ‡Ρ‚ΠΎ всС соли Π°Π·ΠΎΡ‚Π½ΠΎΠΉ кислоты ΠΈΠΌΠ΅ΡŽΡ‚ Ρ…ΠΎΡ€ΠΎΡˆΡƒΡŽ Ρ€Π°ΡΡ‚Π²ΠΎΡ€ΠΈΠΌΠΎΡΡ‚ΡŒ Π² Π²ΠΎΠ΄Π΅, ΡΠ»Π΅Π΄ΠΎΠ²Π°Ρ‚Π΅Π»ΡŒΠ½ΠΎ, ΠΏΡ€ΠΎΠ΄ΡƒΠΊΡ‚Ρ‹ Ρ€Π΅Π°ΠΊΡ†ΠΈΠΈ Π½Π΅ Π±ΡƒΠ΄ΡƒΡ‚ ΠΌΠ΅ΡˆΠ°Ρ‚ΡŒ Π΅Π΅ ΠΏΡ€ΠΎΡ‚Π΅ΠΊΠ°Π½ΠΈΡŽ.

ЀизичСскиС свойства соСдинСний NO, NO 2 ΠΈ HNO 3 Π² Ρ†ΠΈΡ„Ρ€Π°Ρ…

Оксид Π°Π·ΠΎΡ‚Π°(II ) NO. ΠœΠΎΠ»ΡΡ€Π½Π°Ρ масса 30 Π³/моль. Π’Π΅ΠΌΠΏΠ΅Ρ€Π°Ρ‚ΡƒΡ€Π° плавлСния Ρ€Π°Π²Π½Π° –164 Β°Π‘, Ρ‚Π΅ΠΌΠΏΠ΅Ρ€Π°Ρ‚ΡƒΡ€Π° кипСния –154 Β°Π‘. ΠŸΠ»ΠΎΡ‚Π½ΠΎΡΡ‚ΡŒ Π³Π°Π·ΠΎΠΎΠ±Ρ€Π°Π·Π½ΠΎΠ³ΠΎ NО ΠΏΡ€ΠΈ Π½ΠΎΡ€ΠΌΠ°Π»ΡŒΠ½Ρ‹Ρ… условиях (0 Β°Π‘, 1 Π°Ρ‚ΠΌ) 1,3402 Π³/Π». Π Π°ΡΡ‚Π²ΠΎΡ€ΠΈΠΌΠΎΡΡ‚ΡŒ ΠΏΡ€ΠΈ атмосфСрном Π΄Π°Π²Π»Π΅Π½ΠΈΠΈ ΠΈ 20 Β°Π‘ Ρ€Π°Π²Π½Π° 4,7 ΠΌΠ» Π³Π°Π·Π° NO Π½Π° 100 Π³ Π²ΠΎΠ΄Ρ‹.
Оксид Π°Π·ΠΎΡ‚Π°(IV ) NO 2 . ΠœΠΎΠ»ΡΡ€Π½Π°Ρ масса 46 Π³/моль. Π’Π΅ΠΌΠΏΠ΅Ρ€Π°Ρ‚ΡƒΡ€Π° плавлСния –11 Β°Π‘, Ρ‚Π΅ΠΌΠΏΠ΅Ρ€Π°Ρ‚ΡƒΡ€Π° кипСния 21 Β°Π‘. ΠŸΠ»ΠΎΡ‚Π½ΠΎΡΡ‚ΡŒ Π³Π°Π·ΠΎΠΎΠ±Ρ€Π°Π·Π½ΠΎΠ³ΠΎ NO 2 ΠΏΡ€ΠΈ Π½. Ρƒ. 1,491 Π³/Π». Π Π°ΡΡ‚Π²ΠΎΡ€ΠΈΠΌΠΎΡΡ‚ΡŒ – ΠΏΡ€ΠΈ условии, Ρ‡Ρ‚ΠΎ этот оксид сначала Ρ€Π΅Π°Π³ΠΈΡ€ΡƒΠ΅Ρ‚ с Π²ΠΎΠ΄ΠΎΠΉ Π½Π° Π²ΠΎΠ·Π΄ΡƒΡ…Π΅, Π° ΠΏΠΎΡ‚ΠΎΠΌ Π΅Ρ‰Π΅ ΠΈ растворяСтся Π² ΠΏΠΎΠ»ΡƒΡ‡ΠΈΠ²ΡˆΠ΅ΠΉΡΡ Π°Π·ΠΎΡ‚Π½ΠΎΠΉ кислотС, – ΠΌΠΎΠΆΠ½ΠΎ ΡΡ‡ΠΈΡ‚Π°Ρ‚ΡŒ Π½Π΅ΠΎΠ³Ρ€Π°Π½ΠΈΡ‡Π΅Π½Π½ΠΎΠΉ (Π²ΠΏΠ»ΠΎΡ‚ΡŒ Π΄ΠΎ образования 60%-ΠΉ HNO 3).
ΠŸΠΎΡΠΊΠΎΠ»ΡŒΠΊΡƒ оксид Π°Π·ΠΎΡ‚Π°(IV) Π°ΠΊΡ‚ΠΈΠ²Π½ΠΎ димСризуСтся (ΠΏΡ€ΠΈ 140 Β°Π‘ ΠΎΠ½ Ρ†Π΅Π»ΠΈΠΊΠΎΠΌ находится Π² Π²ΠΈΠ΄Π΅ ΠΌΠΎΠ½ΠΎΠΌΠ΅Ρ€Π° NO 2 , ΠΎΠ΄Π½Π°ΠΊΠΎ ΠΏΡ€ΠΈ 40 Β°Π‘ ΠΌΠΎΠ½ΠΎΠΌΠ΅Ρ€Π° остаСтся ΡƒΠΆΠ΅ ΠΎΠΊΠΎΠ»ΠΎ 30%, Π° ΠΏΡ€ΠΈ 20 Β°Π‘ ΠΎΠ½ практичСски вСсь ΠΏΠ΅Ρ€Π΅Ρ…ΠΎΠ΄ΠΈΡ‚ Π² Π΄ΠΈΠΌΠ΅Ρ€ N 2 O 4), Ρ‚ΠΎ физичСскиС свойства относятся скорСС ΠΊ Π΄ΠΈΠΌΠ΅Ρ€Ρƒ, Ρ‡Π΅ΠΌ ΠΊ ΠΌΠΎΠ½ΠΎΠΌΠ΅Ρ€Ρƒ. ИмСнно этим ΠΌΠΎΠΆΠ½ΠΎ ΠΎΠ±ΡŠΡΡΠ½ΠΈΡ‚ΡŒ достаточно Π²Ρ‹ΡΠΎΠΊΡƒΡŽ Ρ‚Π΅ΠΌΠΏΠ΅Ρ€Π°Ρ‚ΡƒΡ€Ρƒ кипСния (N 2 O 4 достаточно тяТСлая ΠΌΠΎΠ»Π΅ΠΊΡƒΠ»Π°). О стСпСни Π΄ΠΈΠΌΠ΅Ρ€ΠΈΠ·Π°Ρ†ΠΈΠΈ ΠΌΠΎΠΆΠ½ΠΎ ΡΡƒΠ΄ΠΈΡ‚ΡŒ ΠΏΠΎ Ρ†Π²Π΅Ρ‚Ρƒ: ΠΌΠΎΠ½ΠΎΠΌΠ΅Ρ€ интСнсивно ΠΎΠΊΡ€Π°ΡˆΠ΅Π½, Π° Π΄ΠΈΠΌΠ΅Ρ€ бСсцвСтСн.
Азотная кислота HNO 3 . ΠœΠΎΠ»ΡΡ€Π½Π°Ρ масса 63 Π³/моль. Π’Π΅ΠΌΠΏΠ΅Ρ€Π°Ρ‚ΡƒΡ€Π° плавлСния –41,6 Β°Π‘, Ρ‚Π΅ΠΌΠΏΠ΅Ρ€Π°Ρ‚ΡƒΡ€Π° кипСния 83 Β°Π‘. ΠŸΠ»ΠΎΡ‚Π½ΠΎΡΡ‚ΡŒ ΠΆΠΈΠ΄ΠΊΠΎΠΉ 100%-ΠΉ кислоты 1,513 Π³/см 3 . Π Π°ΡΡ‚Π²ΠΎΡ€ΠΈΠΌΠΎΡΡ‚ΡŒ нСограничСнная, ΠΈΠ½Ρ‹ΠΌΠΈ словами, кислота ΠΈ Π²ΠΎΠ΄Π° ΡΠΌΠ΅ΡˆΠΈΠ²Π°ΡŽΡ‚ΡΡ Π² Π»ΡŽΠ±Ρ‹Ρ… ΡΠΎΠΎΡ‚Π½ΠΎΡˆΠ΅Π½ΠΈΡΡ…. Π‘Ρ‚ΠΎΠΈΡ‚ ΠΎΡ‚ΠΌΠ΅Ρ‚ΠΈΡ‚ΡŒ, Ρ‡Ρ‚ΠΎ растворы Π°Π·ΠΎΡ‚Π½ΠΎΠΉ кислоты кипят ΠΏΡ€ΠΈ Ρ‚Π΅ΠΌΠΏΠ΅Ρ€Π°Ρ‚ΡƒΡ€Π°Ρ… Π²Ρ‹ΡˆΠ΅ Ρ‚Π΅ΠΌΠΏΠ΅Ρ€Π°Ρ‚ΡƒΡ€ кипСния чистых Π²ΠΎΠ΄Ρ‹ ΠΈ кислоты. ΠŸΡ€ΠΈ максимальной Ρ‚Π΅ΠΌΠΏΠ΅Ρ€Π°Ρ‚ΡƒΡ€Π΅ (122 Β°Π‘) ΠΊΠΈΠΏΠΈΡ‚ 68,4%-ΠΉ раствор, ΠΏΡ€ΠΈ этом ΠΏΡ€ΠΎΡ†Π΅Π½Ρ‚Π½Ρ‹ΠΉ состав раствора ΠΈ ΠΏΠ°Ρ€Π° ΠΎΠ΄ΠΈΠ½Π°ΠΊΠΎΠ².
БмСси вСщСств, для ΠΊΠΎΡ‚ΠΎΡ€Ρ‹Ρ… состав ΠΏΠ°Ρ€Π° ΠΏΡ€ΠΈ кипячСнии соотвСтствуСт составу Тидкости, Π½Π°Π·Ρ‹Π²Π°ΡŽΡ‚ Π°Π·Π΅ΠΎΡ‚Ρ€ΠΎΠΏΠ½Ρ‹ΠΌΠΈ ΠΈΠ»ΠΈ Π½Π΅Ρ€Π°Π·Π΄Π΅Π»ΡŒΠ½ΠΎΠΊΠΈΠΏΡΡ‰ΠΈΠΌΠΈ. (Π‘Π»ΠΎΠ²ΠΎ Β«Π°Π·Π΅ΠΎΡ‚Ρ€ΠΎΠΏΒ» ΠΏΡ€ΠΎΠΈΠ·ΠΎΡˆΠ»ΠΎ ΠΎΡ‚ Π³Ρ€Π΅Ρ‡. – киплю, – ΠΈΠ·ΠΌΠ΅Π½Π΅Π½ΠΈΠ΅, – ΠΎΡ‚Ρ€ΠΈΡ†Π°Ρ‚Π΅Π»ΡŒΠ½Π°Ρ приставка.) Для Π±ΠΎΠ»Π΅Π΅ Π½ΠΈΠ·ΠΊΠΈΡ… ΠΊΠΎΠ½Ρ†Π΅Π½Ρ‚Ρ€Π°Ρ†ΠΈΠΉ кислоты Ρ…Π°Ρ€Π°ΠΊΡ‚Π΅Ρ€Π½ΠΎ ΡƒΠ²Π΅Π»ΠΈΡ‡Π΅Π½ΠΈΠ΅ количСства Π²ΠΎΠ΄Ρ‹ Π² ΠΏΠ°Ρ€Π΅ ΠΏΠΎ ΡΡ€Π°Π²Π½Π΅Π½ΠΈΡŽ с раствором, Ρ‡Ρ‚ΠΎ ΠΏΡ€ΠΈΠ²ΠΎΠ΄ΠΈΡ‚ ΠΊ ΠΊΠΎΠ½Ρ†Π΅Π½Ρ‚Ρ€ΠΈΡ€ΠΎΠ²Π°Π½ΠΈΡŽ раствора. ΠŸΡ€ΠΈ Π±ΠΎΠ»Π΅Π΅ высоких концСнтрациях, Π½Π°ΠΎΠ±ΠΎΡ€ΠΎΡ‚, состав ΠΏΠ°Ρ€Π° ΠΎΠ±ΠΎΠ³Π°Ρ‰Π΅Π½ кислотой.

Π₯имичСскиС свойства соСдинСний Π°Π·ΠΎΡ‚Π° (Π΄ΠΎΠΏΠΎΠ»Π½Π΅Π½ΠΈΠ΅)

Как ΠΈ всякиС Π΄Ρ€ΡƒΠ³ΠΈΠ΅ вСщСства, содСрТащиС Π°Ρ‚ΠΎΠΌ с ΠΏΡ€ΠΎΠΌΠ΅ΠΆΡƒΡ‚ΠΎΡ‡Π½ΠΎΠΉ ΡΡ‚Π΅ΠΏΠ΅Π½ΡŒΡŽ окислСния, оксиды Π°Π·ΠΎΡ‚Π°(II) ΠΈ (IV) Π² ΠΎΡ‚Π»ΠΈΡ‡ΠΈΠ΅ ΠΎΡ‚ Π°Π·ΠΎΡ‚Π½ΠΎΠΉ кислоты ΠΌΠΎΠ³ΡƒΡ‚ Π²Ρ‹ΡΡ‚ΡƒΠΏΠ°Ρ‚ΡŒ ΠΈ ΠΊΠ°ΠΊ окислитСли, ΠΈ ΠΊΠ°ΠΊ восстановитСли, Π² зависимости ΠΎΡ‚ ΠΏΠ°Ρ€Ρ‚Π½Π΅Ρ€Π° ΠΏΠΎ Ρ€Π΅Π°ΠΊΡ†ΠΈΠΈ. Однако ΠΌΠ½ΠΎΠ³ΠΈΠ΅ ΠΈΠ· этих Ρ€Π΅Π°ΠΊΡ†ΠΈΠΉ Β«Π½Π΅Π°ΠΊΡ‚ΡƒΠ°Π»ΡŒΠ½Ρ‹Β» ΠΈ соотвСтствСнно слабо ΠΈΠ·ΡƒΡ‡Π΅Π½Ρ‹.
Из Β«Π°ΠΊΡ‚ΡƒΠ°Π»ΡŒΠ½Ρ‹Ρ…Β» Ρ€Π΅Π°ΠΊΡ†ΠΈΠΉ стоит Π½Π°Π·Π²Π°Ρ‚ΡŒ Ρ€Π΅Π°ΠΊΡ†ΠΈΡŽ оксида Π°Π·ΠΎΡ‚Π°(IV) с оксидом сСры(IV) Π² присутствии Π²ΠΎΠ΄Ρ‹:

ΠΠΊΡ‚ΡƒΠ°Π»ΡŒΠ½Π° эта рСакция ΠΏΠΎΡ‚ΠΎΠΌΡƒ, Ρ‡Ρ‚ΠΎ присоСдинСниС кислорода ΠΊ оксиду сСры(IV) ΠΏΡ€ΠΎΡ‚Π΅ΠΊΠ°Π΅Ρ‚ Ρ‚ΠΎΠ»ΡŒΠΊΠΎ ΠΏΡ€ΠΈ высоких Ρ‚Π΅ΠΌΠΏΠ΅Ρ€Π°Ρ‚ΡƒΡ€Π°Ρ… ΠΈ Π² присутствии ΠΊΠ°Ρ‚Π°Π»ΠΈΠ·Π°Ρ‚ΠΎΡ€Π°, Π² Ρ‚ΠΎ врСмя ΠΊΠ°ΠΊ присоСдинСниС кислорода ΠΊ оксиду Π°Π·ΠΎΡ‚Π°(II) происходит ΠΏΡ€ΠΈ ΠΎΠ±Ρ‹Ρ‡Π½Ρ‹Ρ… условиях. Π’Π°ΠΊΠΈΠΌ ΠΎΠ±Ρ€Π°Π·ΠΎΠΌ, оксид Π°Π·ΠΎΡ‚Π°(IV) ΠΊΠ°ΠΊ Π±Ρ‹ просто ΠΏΠΎΠΌΠΎΠ³Π°Π΅Ρ‚ оксиду сСры ΠΏΡ€ΠΈΡΠΎΠ΅Π΄ΠΈΠ½ΠΈΡ‚ΡŒ кислород. ΠŸΡ€ΠΎΡ‚Π΅ΠΊΠ°Π΅Ρ‚ эта рСакция ΠΏΡ€ΠΈ ΠΎΠ±Ρ‹Ρ‡Π½Ρ‹Ρ… условиях (Π΄ΠΎΠΏΠΎΠ»Π½ΠΈΡ‚Π΅Π»ΡŒΠ½ΠΎΠ³ΠΎ давлСния Π² смСси ΠΈ нагрСвания Π½Π΅ трСбуСтся).
Оксид Π°Π·ΠΎΡ‚Π°(II ) Ρ‚ΠΎΠΆΠ΅ Ρ€Π΅Π°Π³ΠΈΡ€ΡƒΠ΅Ρ‚ с оксидом сСры(IV), ΠΎΠ΄Π½Π°ΠΊΠΎ ΠΏΡ€ΠΈ совсСм Π΄Ρ€ΡƒΠ³ΠΈΡ… условиях: ΠΈΠ»ΠΈ ΠΏΡ€ΠΈ Π΄Π°Π²Π»Π΅Π½ΠΈΠΈ Π² 500 атмосфСр (!), Ρ‚ΠΎΠ³Π΄Π° ΠΏΠΎΠ»ΡƒΡ‡Π°ΡŽΡ‚ΡΡ оксид сСры(VI) ΠΈ Π°Π·ΠΎΡ‚, ΠΈΠ»ΠΈ Π² Π²ΠΎΠ΄Π½ΠΎΠΌ растворС, Ρ‚ΠΎΠ³Π΄Π° ΠΏΠΎΠ»ΡƒΡ‡Π°ΡŽΡ‚ΡΡ сСрная кислота ΠΈ оксид Π°Π·ΠΎΡ‚Π°(I).
Оксид Π°Π·ΠΎΡ‚Π°(I ). Π˜ΠΌΠ΅Π΅Ρ‚ слабый приятный Π·Π°ΠΏΠ°Ρ… ΠΈ сладковатый вкус. Π‘ кислородом, Π²ΠΎΠ΄ΠΎΠΉ, растворами кислот ΠΈ Ρ‰Π΅Π»ΠΎΡ‡Π΅ΠΉ Π½Π΅ Ρ€Π΅Π°Π³ΠΈΡ€ΡƒΠ΅Ρ‚. РазлагаСтся Π½Π° элСмСнты ΠΏΡ€ΠΈ Ρ‚Π΅ΠΌΠΏΠ΅Ρ€Π°Ρ‚ΡƒΡ€Π΅ Π²Ρ‹ΡˆΠ΅ 500 Β°Π‘, ΠΈΠ½Ρ‹ΠΌΠΈ словами, достаточно устойчив.
Π‘Ρ‚Ρ€ΠΎΠ΅Π½ΠΈΠ΅ ΠΌΠΎΠ»Π΅ΠΊΡƒΠ»Ρ‹ интСрСсноС: линСйная ΠΌΠΎΠ»Π΅ΠΊΡƒΠ»Π° О=N=N, Π² ΠΊΠΎΡ‚ΠΎΡ€ΠΎΠΉ Ρ†Π΅Π½Ρ‚Ρ€Π°Π»ΡŒΠ½Ρ‹ΠΉ Π°Ρ‚ΠΎΠΌ Π°Π·ΠΎΡ‚Π° Ρ‡Π΅Ρ‚Ρ‹Ρ€Π΅Ρ…Π²Π°Π»Π΅Π½Ρ‚Π΅Π½. Он ΠΎΠ±Ρ€Π°Π·ΡƒΠ΅Ρ‚ Π΄Π²Π΅ Π΄Π²ΠΎΠΉΠ½Ρ‹Π΅ связи: ΠΎΠ΄Π½Ρƒ – с кислородом ΠΏΠΎ Ρ‚ΠΈΠΏΠΈΡ‡Π½ΠΎΠΉ схСмС создания ΠΊΠΎΠ²Π°Π»Π΅Π½Ρ‚Π½ΠΎΠΉ связи (Π΄Π²Π° элСктрона Π°Π·ΠΎΡ‚Π°, Π΄Π²Π° элСктрона кислорода), Π΄Ρ€ΡƒΠ³ΡƒΡŽ – с Π°Ρ‚ΠΎΠΌΠΎΠΌ Π°Π·ΠΎΡ‚Π° (ΠΊΠΎΡ‚ΠΎΡ€Ρ‹ΠΉ Π΄Π²Π° ΠΈΠ· своих Ρ‚Ρ€Π΅Ρ… нСспарСнных элСктронов спариваСт ΠΈ ΠΎΠ±Ρ€Π°Π·ΡƒΠ΅Ρ‚ Π·Π° счСт этого ΠΏΡƒΡΡ‚ΡƒΡŽ ΠΎΡ€Π±ΠΈΡ‚Π°Π»ΡŒ), ΠΎΠ΄Π½Π° ΠΈΠ· связСй – ковалСнтная, вторая – Π΄ΠΎΠ½ΠΎΡ€Π½ΠΎ-акцСпторная (рис. 9).

Рис. 9.
ΠœΠΎΠ»Π΅ΠΊΡƒΠ»Π° оксида Π°Π·ΠΎΡ‚Π°(I) – N 2 O.
(Π¦Π΅Π½Ρ‚Ρ€Π°Π»ΡŒΠ½Ρ‹ΠΉ Π°Ρ‚ΠΎΠΌ Π°Π·ΠΎΡ‚Π° Ρ‡Π΅Ρ‚Ρ‹Ρ€Π΅Ρ…Π²Π°Π»Π΅Π½Ρ‚Π΅Π½:
Π΄Π²Π΅ связи с Π°Ρ‚ΠΎΠΌΠ°ΠΌΠΈ Π°Π·ΠΎΡ‚Π°
ΠΈ кислорода ΠΎΠ½ ΠΎΠ±Ρ€Π°Π·ΡƒΠ΅Ρ‚
с ΠΏΠΎΠΌΠΎΡ‰ΡŒΡŽ Π³ΠΈΠ±Ρ€ΠΈΠ΄ΠΈΠ·ΠΎΠ²Π°Π½Π½Ρ‹Ρ…
Π½Π°ΠΏΡ€Π°Π²Π»Π΅Π½Π½Ρ‹Ρ… ΠΎΡ€Π±ΠΈΡ‚Π°Π»Π΅ΠΉ,
Π΄Π²Π΅ Π΄Ρ€ΡƒΠ³ΠΈΠ΅ связи с Ρ‚Π΅ΠΌΠΈ ΠΆΠ΅ Π°Ρ‚ΠΎΠΌΠ°ΠΌΠΈ –
с ΠΏΠΎΠΌΠΎΡ‰ΡŒΡŽ Π½Π΅Π³ΠΈΠ±Ρ€ΠΈΠ΄ΠΈΠ·ΠΎΠ²Π°Π½Π½Ρ‹Ρ… Ρ€-ΠΎΡ€Π±ΠΈΡ‚Π°Π»Π΅ΠΉ,
ΠΏΡ€ΠΈΡ‡Π΅ΠΌ эти связи находятся
Π² Π΄Π²ΡƒΡ… Π²Π·Π°ΠΈΠΌΠ½ΠΎ пСрпСндикулярных плоскостях.
ΠŸΠΎΡΡ‚ΠΎΠΌΡƒ ΠΎΠ΄Π½Π° ΠΈΠ·ΠΎΠ±Ρ€Π°ΠΆΠ΅Π½Π° ΠΊΠ°ΠΊ Β«Π²ΠΈΠ΄ сбоку» (1) –
связь с Π°Ρ‚ΠΎΠΌΠΎΠΌ кислорода,
Π° другая – ΠΊΠ°ΠΊ Β«Π²ΠΈΠ΄ свСрху» (2) –
связь ΠΌΠ΅ΠΆΠ΄Ρƒ двумя Π°Ρ‚ΠΎΠΌΠ°ΠΌΠΈ Π°Π·ΠΎΡ‚Π°.)

Оксид Π°Π·ΠΎΡ‚Π°(III). Бостоит ΠΈΠ· NО ΠΈ NO 2 , ΡΠΏΠ°Ρ€ΠΈΠ²ΡˆΠΈΡ… свои нСспарСнныС элСктроны. НачинаСт Ρ€Π°Π·Π»Π°Π³Π°Ρ‚ΡŒΡΡ Π½Π° ΡΠΎΠΎΡ‚Π²Π΅Ρ‚ΡΡ‚Π²ΡƒΡŽΡ‰ΠΈΠ΅ Π³Π°Π·Ρ‹ ΡƒΠΆΠ΅ ΠΏΡ€ΠΈ ΠΏΠ»Π°Π²Π»Π΅Π½ΠΈΠΈ (–101 Β°Π‘).
Оксид Π°Π·ΠΎΡ‚Π°(V). Бостоит ΠΈΠ· Π΄Π²ΡƒΡ… Π³Ρ€ΡƒΠΏΠΏ NO 2 , соСдинСнных Ρ‡Π΅Ρ€Π΅Π· кислород. НСсколько Π±ΠΎΠ»Π΅Π΅ устойчивоС соСдинСниС, Ρ‡Π΅ΠΌ оксид Π°Π·ΠΎΡ‚Π°(III), Π½Π°Ρ‡ΠΈΠ½Π°Π΅Ρ‚ Ρ€Π°Π·Π»Π°Π³Π°Ρ‚ΡŒΡΡ ΠΏΡ€ΠΈ ΠΊΠΎΠΌΠ½Π°Ρ‚Π½ΠΎΠΉ Ρ‚Π΅ΠΌΠΏΠ΅Ρ€Π°Ρ‚ΡƒΡ€Π΅. Π§Π°ΡΡ‚ΡŒ связСй Π² Π½Π΅ΠΌ, СстСствСнно, Π΄ΠΎΠ½ΠΎΡ€Π½ΠΎ-Π°ΠΊΡ†Π΅ΠΏΡ‚ΠΎΡ€Π½Ρ‹Π΅. И Π½ΠΈΠΊΠ°ΠΊΠΈΡ… «пятивалСнтных Π°Π·ΠΎΡ‚ΠΎΠ²Β».
К химичСским свойствам Π°Π·ΠΎΡ‚Π½ΠΎΠΉ кислоты слСдуСт Π΄ΠΎΠ±Π°Π²ΠΈΡ‚ΡŒ, Ρ‡Ρ‚ΠΎ ΠΎΠ½Π° прСкрасно Ρ€Π΅Π°Π³ΠΈΡ€ΡƒΠ΅Ρ‚ с Π½Π΅ΠΌΠ΅Ρ‚Π°Π»Π»Π°ΠΌΠΈ, ΠΊΠΎΡ‚ΠΎΡ€Ρ‹Π΅ ΠΌΠΎΠΆΠ΅Ρ‚ ΠΎΠΊΠΈΡΠ»ΠΈΡ‚ΡŒ. Π’Π°ΠΊ, концСнтрированная азотная кислота Ρ€Π΅Π°Π³ΠΈΡ€ΡƒΠ΅Ρ‚ ΠΈ с сСрой, ΠΈ с фосфором, ΠΈ с ΡƒΠ³Π»Π΅ΠΌ, образуя ΠΏΡ€ΠΈ этом ΡΠ΅Ρ€Π½ΡƒΡŽ, Ρ„ΠΎΡΡ„ΠΎΡ€Π½ΡƒΡŽ ΠΈ ΡƒΠ³ΠΎΠ»ΡŒΠ½ΡƒΡŽ кислоты соотвСтствСнно.
Π˜Π½Ρ‚Π΅Ρ€Π΅ΡΠ½Ρ‹ ΠΈ Π²Π°ΠΆΠ½Ρ‹ Ρ€Π΅Π°ΠΊΡ†ΠΈΠΈ Π°Π·ΠΎΡ‚Π½ΠΎΠΉ кислоты с органичСскими вСщСствами. НапримСр, ΠΏΡ€ΠΈ Π·Π°ΠΌΠ΅Ρ‰Π΅Π½ΠΈΠΈ Ρ‚Ρ€Π΅Ρ… Π°Ρ‚ΠΎΠΌΠΎΠ² Π²ΠΎΠ΄ΠΎΡ€ΠΎΠ΄Π° Π² Ρ‚ΠΎΠ»ΡƒΠΎΠ»Π΅ Π½Π° Π³Ρ€ΡƒΠΏΠΏΡ‹ NO 2 образуСтся Ρ‚Ρ€ΠΈΠ½ΠΈΡ‚Ρ€ΠΎΡ‚ΠΎΠ»ΡƒΠΎΠ» (ΠΈΠ»ΠΈ просто Ρ‚ΠΎΠ») – Π²Π·Ρ€Ρ‹Π²Ρ‡Π°Ρ‚ΠΎΠ΅ вСщСство.

ЭкологичСскиС свойства оксидов Π°Π·ΠΎΡ‚Π° ΠΈ Π°Π·ΠΎΡ‚Π½ΠΎΠΉ кислоты

Оксид Π°Π·ΠΎΡ‚Π°(I) ΠΎΡ‚Π½ΠΎΡΠΈΡ‚Π΅Π»ΡŒΠ½ΠΎ ΠΈΠ½Π΅Ρ€Ρ‚Π΅Π½, Π° ΠΏΠΎΡ‚ΠΎΠΌΡƒ «экологичСски Π½Π΅ΠΉΡ‚Ρ€Π°Π»Π΅Π½Β». Однако Π½Π° Ρ‡Π΅Π»ΠΎΠ²Π΅ΠΊΠ° ΠΎΠ½ ΠΎΠΊΠ°Π·Ρ‹Π²Π°Π΅Ρ‚ наркотичСскоС дСйствиС, начиная ΠΎΡ‚ просто вСсСлья (Π·Π° Ρ‡Ρ‚ΠΎ ΠΎΠ½ ΠΈ Π±Ρ‹Π» ΠΏΡ€ΠΎΠ·Π²Π°Π½ «вСсСлящим Π³Π°Π·ΠΎΠΌΒ») ΠΈ заканчивая Π³Π»ΡƒΠ±ΠΎΠΊΠΈΠΌ сном, Ρ‡Ρ‚ΠΎ нашло своС ΠΏΡ€ΠΈΠΌΠ΅Π½Π΅Π½ΠΈΠ΅ Π² ΠΌΠ΅Π΄ΠΈΡ†ΠΈΠ½Π΅. Π˜Π½Ρ‚Π΅Ρ€Π΅ΡΠ½ΠΎ, Ρ‡Ρ‚ΠΎ ΠΎΠ½ Π±Π΅Π·Π²Ρ€Π΅Π΄Π΅Π½, ΠΈ для мСдицинского Π½Π°Ρ€ΠΊΠΎΠ·Π° ΠΏΡ€ΠΈΠΌΠ΅Π½ΡΡŽΡ‚ смСсь оксида Π°Π·ΠΎΡ‚Π°(I) с кислородом Π² Ρ‚Π°ΠΊΠΎΠΌ ΠΆΠ΅ ΡΠΎΠΎΡ‚Π½ΠΎΡˆΠ΅Π½ΠΈΠΈ, Ρ‡Ρ‚ΠΎ ΠΈ ΡΠΎΠΎΡ‚Π½ΠΎΡˆΠ΅Π½ΠΈΠ΅ Π°Π·ΠΎΡ‚Π° ΠΈ кислорода Π² Π²ΠΎΠ·Π΄ΡƒΡ…Π΅. НаркотичСскоС дСйствиС снимаСтся сразу послС прСкращСния вдыхания этого Π³Π°Π·Π°.
Π”Π²Π° Π΄Ρ€ΡƒΠ³ΠΈΡ… устойчивых оксида Π°Π·ΠΎΡ‚Π° Π»Π΅Π³ΠΊΠΎ пСрСходят ΠΎΠ΄ΠΈΠ½ Π² Π΄Ρ€ΡƒΠ³ΠΎΠΉ, Π·Π°Ρ‚Π΅ΠΌ Π² кислоты, Π° Π·Π°Ρ‚Π΅ΠΌ Π² Π°Π½ΠΈΠΎΠ½Ρ‹ ΠΈ. Π’Π°ΠΊΠΈΠΌ ΠΎΠ±Ρ€Π°Π·ΠΎΠΌ, эти вСщСства ΠΏΡ€Π΅Π΄ΡΡ‚Π°Π²Π»ΡΡŽΡ‚ собой СстСствСнныС ΠΌΠΈΠ½Π΅Ρ€Π°Π»ΡŒΠ½Ρ‹Π΅ удобрСния, ΠΏΡ€Π°Π²Π΄Π°, Ссли находятся Π² СстСствСнных количСствах. Π’ «нССстСствСнных» количСствах эти Π³Π°Π·Ρ‹ Ρ€Π΅Π΄ΠΊΠΎ ΠΏΠΎΠΏΠ°Π΄Π°ΡŽΡ‚ Π² атмосфСру Π² одиночСствС. Как ΠΏΡ€Π°Π²ΠΈΠ»ΠΎ, образуСтся Ρ†Π΅Π»Ρ‹ΠΉ Β«Π±ΡƒΠΊΠ΅Ρ‚Β» ядовитых соСдинСний, ΠΊΠΎΡ‚ΠΎΡ€Ρ‹Π΅ Π΄Π΅ΠΉΡΡ‚Π²ΡƒΡŽΡ‚ комплСксно.
НапримСр, всСго ΠΎΠ΄ΠΈΠ½ Π·Π°Π²ΠΎΠ΄ Π°Π·ΠΎΡ‚Π½Ρ‹Ρ… ΡƒΠ΄ΠΎΠ±Ρ€Π΅Π½ΠΈΠΉ выбрасываСт Π² Π²ΠΎΠ·Π΄ΡƒΡ… ΠΊΡ€ΠΎΠΌΠ΅ оксидов Π°Π·ΠΎΡ‚Π°, Π°Π·ΠΎΡ‚Π½ΠΎΠΉ кислоты, Π°ΠΌΠΌΠΈΠ°ΠΊΠ° ΠΈ ΠΏΡ‹Π»ΠΈ ΠΎΡ‚ ΡƒΠ΄ΠΎΠ±Ρ€Π΅Π½ΠΈΠΉ Π΅Ρ‰Π΅ ΠΈ оксиды сСры, соСдинСния Ρ„Ρ‚ΠΎΡ€Π°, Π½Π΅ΠΊΠΎΡ‚ΠΎΡ€Ρ‹Π΅ органичСскиС соСдинСния. Π£Ρ‡Π΅Π½Ρ‹Π΅ Π²Ρ‹ΡΡΠ½ΡΡŽΡ‚ ΡƒΡΡ‚ΠΎΠΉΡ‡ΠΈΠ²ΠΎΡΡ‚ΡŒ Ρ€Π°Π·Π»ΠΈΡ‡Π½Ρ‹Ρ… Ρ‚Ρ€Π°Π², кустов ΠΈ Π΄Π΅Ρ€Π΅Π²ΡŒΠ΅Π² ΠΊ ΠΏΠΎΠ΄ΠΎΠ±Π½Ρ‹ΠΌ Β«Π±ΡƒΠΊΠ΅Ρ‚Π°ΠΌΒ». Π£ΠΆΠ΅ извСстно, Ρ‡Ρ‚ΠΎ, ΠΊ соТалСнию, Сль ΠΈ сосна нСустойчивы ΠΈ быстро ΠΏΠΎΠ³ΠΈΠ±Π°ΡŽΡ‚, ΠΎΠ΄Π½Π°ΠΊΠΎ бСлая акация, канадский Ρ‚ΠΎΠΏΠΎΠ»ΡŒ, ΠΈΠ²Ρ‹ ΠΈ Π½Π΅ΠΊΠΎΡ‚ΠΎΡ€Ρ‹Π΅ Π΄Ρ€ΡƒΠ³ΠΈΠ΅ растСния ΠΌΠΎΠ³ΡƒΡ‚ ΡΡƒΡ‰Π΅ΡΡ‚Π²ΠΎΠ²Π°Ρ‚ΡŒ Π² Ρ‚Π°ΠΊΠΈΡ… условиях, Π±ΠΎΠ»Π΅Π΅ Ρ‚ΠΎΠ³ΠΎ, ΠΎΠ½ΠΈ ΡΠΏΠΎΡΠΎΠ±ΡΡ‚Π²ΡƒΡŽΡ‚ ΡƒΠ΄Π°Π»Π΅Π½ΠΈΡŽ ΠΈΠ· Π²ΠΎΠ·Π΄ΡƒΡ…Π° этих вСщСств.
БильноС ΠΎΡ‚Ρ€Π°Π²Π»Π΅Π½ΠΈΠ΅ оксидами Π°Π·ΠΎΡ‚Π° ΠΌΠΎΠΆΠ½ΠΎ ΠΏΠΎΠ»ΡƒΡ‡ΠΈΡ‚ΡŒ Π² основном ΠΏΡ€ΠΈ авариях Π½Π° ΡΠΎΠΎΡ‚Π²Π΅Ρ‚ΡΡ‚Π²ΡƒΡŽΡ‰ΠΈΡ… производствах. ΠžΡ‚Π²Π΅Ρ‚Π½Π°Ρ рСакция ΠΎΡ€Π³Π°Π½ΠΈΠ·ΠΌΠ° Π±ΡƒΠ΄Π΅Ρ‚ Ρ€Π°Π·Π½ΠΎΠΉ ΠΈΠ·-Π·Π° различия Π² свойствах этих Π³Π°Π·ΠΎΠ². Β«Π•Π΄ΠΊΠΈΠΉΒ» NО 2 Π² ΠΏΠ΅Ρ€Π²ΡƒΡŽ ΠΎΡ‡Π΅Ρ€Π΅Π΄ΡŒ дСйствуСт Π½Π° слизистыС ΠΎΠ±ΠΎΠ»ΠΎΡ‡ΠΊΠΈ носоглотки, Π³Π»Π°Π·, Π²Ρ‹Π·Ρ‹Π²Π°Π΅Ρ‚ ΠΎΡ‚Π΅ΠΊ Π»Π΅Π³ΠΊΠΈΡ…; NО, ΠΊΠ°ΠΊ малорастворимоС Π² Π²ΠΎΠ΄Π΅ ΠΈ Π½Π΅ Π΅Π΄ΠΊΠΎΠ΅ вСщСство, ΠΏΡ€ΠΎΡ…ΠΎΠ΄ΠΈΡ‚ Ρ‡Π΅Ρ€Π΅Π· Π»Π΅Π³ΠΊΠΈΠ΅ ΠΈ ΠΏΠΎΠΏΠ°Π΄Π°Π΅Ρ‚ Π² ΠΊΡ€ΠΎΠ²ΡŒ, вызывая Π½Π°Ρ€ΡƒΡˆΠ΅Π½ΠΈΡ Π² Ρ†Π΅Π½Ρ‚Ρ€Π°Π»ΡŒΠ½ΠΎΠΉ ΠΈ ΠΏΠ΅Ρ€ΠΈΡ„Π΅Ρ€ΠΈΠΉΠ½ΠΎΠΉ Π½Π΅Ρ€Π²Π½Ρ‹Ρ… систСмах. Оба оксида Ρ€Π΅Π°Π³ΠΈΡ€ΡƒΡŽΡ‚ с Π³Π΅ΠΌΠΎΠ³Π»ΠΎΠ±ΠΈΠ½ΠΎΠΌ ΠΊΡ€ΠΎΠ²ΠΈ, хотя ΠΈ ΠΏΠΎ-Ρ€Π°Π·Π½ΠΎΠΌΡƒ, Π½ΠΎ с ΠΎΠ΄ΠΈΠ½Π°ΠΊΠΎΠ²Ρ‹ΠΌ Ρ€Π΅Π·ΡƒΠ»ΡŒΡ‚Π°Ρ‚ΠΎΠΌ – Π³Π΅ΠΌΠΎΠ³Π»ΠΎΠ±ΠΈΠ½ пСрСстаСт ΠΏΠ΅Ρ€Π΅Π½ΠΎΡΠΈΡ‚ΡŒ кислород.
ЭкологичСскиС свойства Π°Π·ΠΎΡ‚Π½ΠΎΠΉ кислоты ΡΠΊΠ»Π°Π΄Ρ‹Π²Π°ΡŽΡ‚ΡΡ ΠΈΠ· Π΄Π²ΡƒΡ… Β«ΠΏΠΎΠ»ΠΎΠ²ΠΈΠ½ΠΎΠΊΒ». Как сильная кислота, ΠΎΠ½Π° Ρ€Π°Π·Ρ€ΡƒΡˆΠ°ΡŽΡ‰Π΅ дСйствуСт Π½Π΅ Ρ‚ΠΎΠ»ΡŒΠΊΠΎ Π½Π° ΠΆΠΈΠ²Ρ‹Π΅ Ρ‚ΠΊΠ°Π½ΠΈ (ΠΊΠΎΠΆΡƒ Ρ‡Π΅Π»ΠΎΠ²Π΅ΠΊΠ°, лист растСния), Π½ΠΎ ΠΈ Π½Π° ΠΏΠΎΡ‡Π²Ρƒ, Ρ‡Ρ‚ΠΎ достаточно Π°ΠΊΡ‚ΡƒΠ°Π»ΡŒΠ½ΠΎ – кислотныС (ΠΈΠ·-Π·Π° присутствия оксидов Π°Π·ΠΎΡ‚Π° ΠΈ сСры) Π΄ΠΎΠΆΠ΄ΠΈ, ΡƒΠ²Ρ‹, Π½Π΅ Ρ€Π΅Π΄ΠΊΠΎΡΡ‚ΡŒ. ΠŸΡ€ΠΈ ΠΏΠΎΠΏΠ°Π΄Π°Π½ΠΈΠΈ кислоты Π½Π° ΠΊΠΎΠΆΡƒ Π²ΠΎΠ·Π½ΠΈΠΊΠ°Π΅Ρ‚ химичСский ΠΎΠΆΠΎΠ³, ΠΊΠΎΡ‚ΠΎΡ€Ρ‹ΠΉ Π±ΠΎΠ»Π΅Π·Π½Π΅Π½Π½Π΅Π΅ ΠΈ Π·Π°ΠΆΠΈΠ²Π°Π΅Ρ‚ Π·Π½Π°Ρ‡ΠΈΡ‚Π΅Π»ΡŒΠ½ΠΎ дольшС, Ρ‡Π΅ΠΌ тСрмичСский. Π­Ρ‚ΠΎ Π±Ρ‹Π»ΠΈ основныС экологичСскиС свойства ΠΊΠ°Ρ‚ΠΈΠΎΠ½Π° Π²ΠΎΠ΄ΠΎΡ€ΠΎΠ΄Π°.
ΠŸΠ΅Ρ€Π΅ΠΉΠ΄Π΅ΠΌ ΠΊ ΠΈΠ·ΡƒΡ‡Π΅Π½ΠΈΡŽ Π°Π½ΠΈΠΎΠ½Π° . ΠŸΡ€ΠΈ дСйствии сильной кислоты Π½Π° ΠΏΠ΅Ρ€Π΅Π΄Π½ΠΈΠΉ ΠΏΠ»Π°Π½ Π²Ρ‹Π΄Π²ΠΈΠ³Π°ΡŽΡ‚ΡΡ ΠΈΠΌΠ΅Π½Π½ΠΎ кислотныС свойства, поэтому свойства Π°Π½ΠΈΠΎΠ½Π° Π»ΡƒΡ‡ΡˆΠ΅ Ρ€Π°ΡΡΠΌΠΎΡ‚Ρ€Π΅Ρ‚ΡŒ Π½Π° ΠΏΡ€ΠΈΠΌΠ΅Ρ€Π΅ солСй.
ВзаимодСйствиС Π½ΠΈΡ‚Ρ€Π°Ρ‚-ΠΈΠΎΠ½Π° с Ρ„Π°ΡƒΠ½ΠΎΠΉ ΠΈ Ρ„Π»ΠΎΡ€ΠΎΠΉ. Π”Π΅Π»ΠΎ Π² Ρ‚ΠΎΠΌ, Ρ‡Ρ‚ΠΎ Π½ΠΈΡ‚Ρ€Π°Ρ‚-ΠΈΠΎΠ½ – Π½Π΅ΠΎΡ‚ΡŠΠ΅ΠΌΠ»Π΅ΠΌΠ°Ρ Ρ‡Π°ΡΡ‚ΡŒ ΠΊΡ€ΡƒΠ³ΠΎΠ²ΠΎΡ€ΠΎΡ‚Π° Π°Π·ΠΎΡ‚Π° Π² ΠΏΡ€ΠΈΡ€ΠΎΠ΄Π΅, ΠΎΠ½ Π² Π½Π΅ΠΉ всСгда присутствуСт. Π’ ΠΎΠ±Ρ‹Ρ‡Π½Ρ‹Ρ… условиях ΠΈ Π² Ρ€Π°Π·Π±Π°Π²Π»Π΅Π½Π½Ρ‹Ρ… растворах устойчив, слабо проявляСт ΠΎΠΊΠΈΡΠ»ΠΈΡ‚Π΅Π»ΡŒΠ½Ρ‹Π΅ свойства, Π½Π΅ осаТдаСт ΠΊΠ°Ρ‚ΠΈΠΎΠ½Ρ‹ ΠΌΠ΅Ρ‚Π°Π»Π»ΠΎΠ², Ρ‚Π΅ΠΌ самым способствуя транспортировкС этих ΠΈΠΎΠ½ΠΎΠ² с раствором Π² ΠΏΠΎΡ‡Π²Π΅, растСниях ΠΈ Ρ‚. ΠΏ.
Нитрат-ΠΈΠΎΠ½ становится ядовитым Ρ‚ΠΎΠ»ΡŒΠΊΠΎ Π² Π±ΠΎΠ»ΡŒΡˆΠΈΡ… количСствах, Π½Π°Ρ€ΡƒΡˆΠ°ΡŽΡ‰ΠΈΡ… баланс Π΄Ρ€ΡƒΠ³ΠΈΡ… вСщСств. НапримСр, ΠΏΡ€ΠΈ ΠΈΠ·Π±Ρ‹Ρ‚ΠΊΠ΅ Π½ΠΈΡ‚Ρ€Π°Ρ‚ΠΎΠ² Π² растСниях ΡƒΠΌΠ΅Π½ΡŒΡˆΠ°Π΅Ρ‚ΡΡ количСство аскорбиновой кислоты. (Π‘Ρ‚ΠΎΠΈΡ‚ Π½Π°ΠΏΠΎΠΌΠ½ΠΈΡ‚ΡŒ, Ρ‡Ρ‚ΠΎ ΠΆΠΈΠ²ΠΎΠΉ ΠΎΡ€Π³Π°Π½ΠΈΠ·ΠΌ Π½Π°ΡΡ‚ΠΎΠ»ΡŒΠΊΠΎ Ρ‚ΠΎΠ½ΠΊΠΎ ΠΎΡ€Π³Π°Π½ΠΈΠ·ΠΎΠ²Π°Π½, Ρ‡Ρ‚ΠΎ любоС вСщСство Π² Π±ΠΎΠ»ΡŒΡˆΠΈΡ… количСствах Π½Π°Ρ€ΡƒΡˆΠ°Π΅Ρ‚ равновСсиС ΠΈ, ΡΠ»Π΅Π΄ΠΎΠ²Π°Ρ‚Π΅Π»ΡŒΠ½ΠΎ, становится ядовитым.)
РастСния ΠΈ Π±Π°ΠΊΡ‚Π΅Ρ€ΠΈΠΈ ΠΈΡΠΏΠΎΠ»ΡŒΠ·ΡƒΡŽΡ‚ Π½ΠΈΡ‚Ρ€Π°Ρ‚Ρ‹ для построСния Π±Π΅Π»ΠΊΠΎΠ² ΠΈ Π΄Ρ€ΡƒΠ³ΠΈΡ… Π½Π΅ΠΎΠ±Ρ…ΠΎΠ΄ΠΈΠΌΡ‹Ρ… органичСских соСдинСний. Для этого Π½Π°Π΄ΠΎ пСрСвСсти Π½ΠΈΡ‚Ρ€Π°Ρ‚-ΠΈΠΎΠ½ Π² ΠΈΠΎΠ½ аммония. Π­Ρ‚Π° рСакция катализируСтся Ρ„Π΅Ρ€ΠΌΠ΅Π½Ρ‚Π°ΠΌΠΈ, содСрТащими ΠΈΠΎΠ½Ρ‹ ΠΌΠ΅Ρ‚Π°Π»Π»ΠΎΠ² (ΠΌΠ΅Π΄ΠΈ, ΠΆΠ΅Π»Π΅Π·Π°, ΠΌΠ°Ρ€Π³Π°Π½Ρ†Π° ΠΈ Π΄Ρ€.). Из-Π·Π° Π³ΠΎΡ€Π°Π·Π΄ΠΎ большСй ядовитости Π°ΠΌΠΌΠΈΠ°ΠΊΠ° ΠΈ ΠΈΠΎΠ½Π° аммония Π² растСниях Ρ…ΠΎΡ€ΠΎΡˆΠΎ ΠΎΡ‚Ρ€Π°Π±ΠΎΡ‚Π°Π½Π° ΠΈ обратная рСакция ΠΏΠ΅Ρ€Π΅Π²ΠΎΠ΄Π° ΠΈΠΎΠ½Π° аммония Π² Π½ΠΈΡ‚Ρ€Π°Ρ‚.
Π–ΠΈΠ²ΠΎΡ‚Π½Ρ‹Π΅ Π½Π΅ ΡƒΠΌΠ΅ΡŽΡ‚ ΡΡ‚Ρ€ΠΎΠΈΡ‚ΡŒ всС Π½Π΅ΠΎΠ±Ρ…ΠΎΠ΄ΠΈΠΌΡ‹Π΅ ΠΈΠΌ органичСскиС соСдинСния ΠΈΠ· нСорганичСских – ΠΎΡ‚ΡΡƒΡ‚ΡΡ‚Π²ΡƒΡŽΡ‚ ΡΠΎΠΎΡ‚Π²Π΅Ρ‚ΡΡ‚Π²ΡƒΡŽΡ‰ΠΈΠ΅ Ρ„Π΅Ρ€ΠΌΠ΅Π½Ρ‚Ρ‹. Однако ΠΌΠΈΠΊΡ€ΠΎΠΎΡ€Π³Π°Π½ΠΈΠ·ΠΌΡ‹, ΠΆΠΈΠ²ΡƒΡ‰ΠΈΠ΅ Π² ΠΆΠ΅Π»ΡƒΠ΄ΠΊΠ΅ ΠΈ ΠΊΠΈΡˆΠ΅Ρ‡Π½ΠΈΠΊΠ΅, этими Ρ„Π΅Ρ€ΠΌΠ΅Π½Ρ‚Π°ΠΌΠΈ ΠΎΠ±Π»Π°Π΄Π°ΡŽΡ‚ ΠΈ ΠΌΠΎΠ³ΡƒΡ‚ ΠΏΠ΅Ρ€Π΅Π²ΠΎΠ΄ΠΈΡ‚ΡŒ Π½ΠΈΡ‚Ρ€Π°Ρ‚-ΠΈΠΎΠ½ Π² Π½ΠΈΡ‚Ρ€ΠΈΡ‚-ΠΈΠΎΠ½. ИмСнно Π½ΠΈΡ‚Ρ€ΠΈΡ‚-ΠΈΠΎΠ½ ΠΈ дСйствуСт ΠΊΠ°ΠΊ ΠΎΡ‚Ρ€Π°Π²ΠΈΡ‚Π΅Π»ΡŒ, пСрСводя ΠΆΠ΅Π»Π΅Π·ΠΎ Π² Π³Π΅ΠΌΠΎΠ³Π»ΠΎΠ±ΠΈΠ½Π΅ ΠΈΠ· Fe 2+ Π² Fe 3+ .
Π‘ΠΎΠ΅Π΄ΠΈΠ½Π΅Π½ΠΈΠ΅, содСрТащСС Fe 3+ ΠΈ Π½Π°Π·Ρ‹Π²Π°Π΅ΠΌΠΎΠ΅ ΠΌΠ΅Ρ‚Π³Π΅ΠΌΠΎΠ³Π»ΠΎΠ±ΠΈΠ½ΠΎΠΌ, слишком ΠΏΡ€ΠΎΡ‡Π½ΠΎ связываСт кислород Π²ΠΎΠ·Π΄ΡƒΡ…Π°, ΡΠ»Π΅Π΄ΠΎΠ²Π°Ρ‚Π΅Π»ΡŒΠ½ΠΎ, Π½Π΅ ΠΌΠΎΠΆΠ΅Ρ‚ ΠΎΡ‚Π΄Π°Π²Π°Ρ‚ΡŒ Π΅Π³ΠΎ тканям. Π’ Ρ€Π΅Π·ΡƒΠ»ΡŒΡ‚Π°Ρ‚Π΅ ΠΎΡ€Π³Π°Π½ΠΈΠ·ΠΌ страдаСт ΠΎΡ‚ нСдостатка кислорода, ΠΏΡ€ΠΈ этом происходят Π½Π°Ρ€ΡƒΡˆΠ΅Π½ΠΈΡ Π² Ρ€Π°Π±ΠΎΡ‚Π΅ ΠΌΠΎΠ·Π³Π°, сСрдца ΠΈ Π΄Ρ€ΡƒΠ³ΠΈΡ… ΠΎΡ€Π³Π°Π½ΠΎΠ².
ΠžΠ±Ρ‹Ρ‡Π½ΠΎ Π½ΠΈΡ‚Ρ€ΠΈΡ‚-ΠΈΠΎΠ½ образуСтся Π½Π΅ Π² ΠΆΠ΅Π»ΡƒΠ΄ΠΊΠ΅, Π° Π² ΠΊΠΈΡˆΠ΅Ρ‡Π½ΠΈΠΊΠ΅ ΠΈ Π½Π΅ успСваСт ΠΏΠ΅Ρ€Π΅ΠΉΡ‚ΠΈ Π² ΠΊΡ€ΠΎΠ²ΡŒ ΠΈ произвСсти всС эти Ρ€Π°Π·Ρ€ΡƒΡˆΠ΅Π½ΠΈΡ. ΠŸΠΎΡΡ‚ΠΎΠΌΡƒ отравлСния Π½ΠΈΡ‚Ρ€Π°Ρ‚Π°ΠΌΠΈ достаточно Ρ€Π΅Π΄ΠΊΠΈ. БущСствуСт, ΠΏΡ€Π°Π²Π΄Π°, ΠΈ другая ΠΎΠΏΠ°ΡΠ½ΠΎΡΡ‚ΡŒ: Π² нашСм ΠΎΡ€Π³Π°Π½ΠΈΠ·ΠΌΠ΅ Π΅ΡΡ‚ΡŒ ΠΌΠ½ΠΎΠ³ΠΎ вСщСств, Π² ΠΊΠΎΡ‚ΠΎΡ€Ρ‹Ρ… Π°Ρ‚ΠΎΠΌΡ‹ Π²ΠΎΠ΄ΠΎΡ€ΠΎΠ΄Π° Π°ΠΌΠΌΠΈΠ°ΠΊΠ° Π·Π°ΠΌΠ΅Ρ‰Π΅Π½Ρ‹ Π½Π° органичСскиС Ρ€Π°Π΄ΠΈΠΊΠ°Π»Ρ‹. Π’Π°ΠΊΠΈΠ΅ соСдинСния Π½Π°Π·Ρ‹Π²Π°ΡŽΡ‚ Π°ΠΌΠΈΠ½Π°ΠΌΠΈ. ΠŸΡ€ΠΈ Ρ€Π΅Π°ΠΊΡ†ΠΈΠΈ Π°ΠΌΠΈΠ½ΠΎΠ² с Π½ΠΈΡ‚Ρ€ΠΈΡ‚-ΠΈΠΎΠ½Π°ΠΌΠΈ ΠΎΠ±Ρ€Π°Π·ΡƒΡŽΡ‚ΡΡ Π½ΠΈΡ‚Ρ€ΠΎΠ·Π°ΠΌΠΈΠ½Ρ‹ – ΠΊΠ°Π½Ρ†Π΅Ρ€ΠΎΠ³Π΅Π½Π½Ρ‹Π΅ вСщСства:

Они Π΄Π΅ΠΉΡΡ‚Π²ΡƒΡŽΡ‚ Π½Π° ΠΏΠ΅Ρ‡Π΅Π½ΡŒ, ΡΠΏΠΎΡΠΎΠ±ΡΡ‚Π²ΡƒΡŽΡ‚ ΠΎΠ±Ρ€Π°Π·ΠΎΠ²Π°Π½ΠΈΡŽ ΠΎΠΏΡƒΡ…ΠΎΠ»Π΅ΠΉ Π² Π»Π΅Π³ΠΊΠΈΡ… ΠΈ ΠΏΠΎΡ‡ΠΊΠ°Ρ…. Π˜Π½Ρ‚Π΅Ρ€Π΅ΡΠ½ΠΎ, Ρ‡Ρ‚ΠΎ Π°ΠΊΡ‚ΠΈΠ²Π½Ρ‹ΠΌ Π·Π°ΠΌΠ΅Π΄Π»ΠΈΡ‚Π΅Π»Π΅ΠΌ Ρ€Π΅Π°ΠΊΡ†ΠΈΠΈ образования Π½ΠΈΡ‚Ρ€ΠΎΠ·Π°ΠΌΠΈΠ½ΠΎΠ² являСтся Π΄Π°Π²Π½ΠΎ Π½Π°ΠΌ знакомая аскорбиновая кислота.

О.Π .Π’ΠΠ›Π•Π”Π˜ΠΠ‘ΠšΠΠ―
(ΠœΠ“Π£, Москва)

Об Π°Π²Ρ‚ΠΎΡ€Π°Ρ…

Π”ΠΈΠ½Π° КамилСвна Π“Π°ΠΉΠ½ΡƒΠ»Π»ΠΈΠ½Π° β€” ΠΊΠ°Π½Π΄ΠΈΠ΄Π°Ρ‚ биологичСских Π½Π°ΡƒΠΊ, Π½Π°ΡƒΡ‡Π½Ρ‹ΠΉ сотрудник ΠΊΠ°Ρ„Π΅Π΄Ρ€Ρ‹ Ρ„ΠΈΠ·ΠΈΠΎΠ»ΠΎΠ³ΠΈΠΈ Ρ‡Π΅Π»ΠΎΠ²Π΅ΠΊΠ° ΠΈ ΠΆΠΈΠ²ΠΎΡ‚Π½Ρ‹Ρ… биологичСского Ρ„Π°ΠΊΡƒΠ»ΡŒΡ‚Π΅Ρ‚Π° Московского государствСнного унивСрситСта ΠΈΠΌ. М. Π’. Ломоносова, спСциалист Π² области Ρ„ΠΈΠ·ΠΈΠΎΠ»ΠΎΠ³ΠΈΠΈ кровообращСния. ΠžΠ±Π»Π°ΡΡ‚ΡŒ Π½Π°ΡƒΡ‡Π½Ρ‹Ρ… интСрСсов β€” особСнности рСгуляции сосудистой систСмы Π² Ρ€Π°Π½Π½Π΅ΠΌ ΠΏΠΎΡΡ‚Π½Π°Ρ‚Π°Π»ΡŒΠ½ΠΎΠΌ ΠΎΠ½Ρ‚ΠΎΠ³Π΅Π½Π΅Π·Π΅.

Π‘Π²Π΅Ρ‚Π»Π°Π½Π° Ивановна Π‘ΠΎΡ„Ρ€ΠΎΠ½ΠΎΠ²Π° β€” аспирант Ρ‚ΠΎΠΉ ΠΆΠ΅ ΠΊΠ°Ρ„Π΅Π΄Ρ€Ρ‹, занимаСтся ΠΏΡ€ΠΎΠ±Π»Π΅ΠΌΠ°ΠΌΠΈ Π³ΠΎΡ€ΠΌΠΎΠ½Π°Π»ΡŒΠ½ΠΎΠΉ рСгуляции синтСза ΡΠ½Π΄ΠΎΡ‚Π΅Π»ΠΈΠ°Π»ΡŒΠ½ΠΎΠ³ΠΎ оксида Π°Π·ΠΎΡ‚Π°.

Ольга Π‘Π΅Ρ€Π³Π΅Π΅Π²Π½Π° Варасова β€” Π΄ΠΎΠΊΡ‚ΠΎΡ€ биологичСских Π½Π°ΡƒΠΊ, профСссор Ρ‚ΠΎΠΉ ΠΆΠ΅ ΠΊΠ°Ρ„Π΅Π΄Ρ€Ρ‹ ΠΈ Π²Π΅Π΄ΡƒΡ‰ΠΈΠΉ Π½Π°ΡƒΡ‡Π½Ρ‹ΠΉ сотрудник Π»Π°Π±ΠΎΡ€Π°Ρ‚ΠΎΡ€ΠΈΠΈ Ρ„ΠΈΠ·ΠΈΠΎΠ»ΠΎΠ³ΠΈΠΈ ΠΌΡ‹ΡˆΠ΅Ρ‡Π½ΠΎΠΉ Π΄Π΅ΡΡ‚Π΅Π»ΡŒΠ½ΠΎΡΡ‚ΠΈ ГНЦ Π Π€ Β«Π˜Π½ΡΡ‚ΠΈΡ‚ΡƒΡ‚ ΠΌΠ΅Π΄ΠΈΠΊΠΎ-биологичСских ΠΏΡ€ΠΎΠ±Π»Π΅ΠΌ РАН», спСциалист Π² области кровообращСния ΠΈ Π°Π²Ρ‚ΠΎΠ½ΠΎΠΌΠ½ΠΎΠΉ Π½Π΅Ρ€Π²Π½ΠΎΠΉ систСмы. ΠžΠ±Π»Π°ΡΡ‚ΡŒ Π½Π°ΡƒΡ‡Π½Ρ‹Ρ… интСрСсов β€” взаимодСйствиС систСмных ΠΈ Π»ΠΎΠΊΠ°Π»ΡŒΠ½Ρ‹Ρ… ΠΌΠ΅Ρ…Π°Π½ΠΈΠ·ΠΌΠΎΠ² рСгуляции сСрдСчно-сосудистой систСмы.

Вонус кровСносных сосудов ΠΈ ΡƒΡ€ΠΎΠ²Π΅Π½ΡŒ Π°Ρ€Ρ‚Π΅Ρ€ΠΈΠ°Π»ΡŒΠ½ΠΎΠ³ΠΎ давлСния Π² ΠΎΡ€Π³Π°Π½ΠΈΠ·ΠΌΠ΅ Ρ€Π΅Π³ΡƒΠ»ΠΈΡ€ΡƒΡŽΡ‚ΡΡ слаТСнной Ρ€Π°Π±ΠΎΡ‚ΠΎΠΉ ΠΌΠ½ΠΎΠ³ΠΈΡ… систСм ΠΈ ΠΌΠ΅Ρ…Π°Π½ΠΈΠ·ΠΌΠΎΠ², срСди ΠΊΠΎΡ‚ΠΎΡ€Ρ‹Ρ… Π²Π°ΠΆΠ½ΡƒΡŽ Ρ€ΠΎΠ»ΡŒ ΠΈΠ³Ρ€Π°Π΅Ρ‚ эндотСлий сосудов. БСкрСция оксида Π°Π·ΠΎΡ‚Π° (NO) - ΠΎΠ΄Π½Π° ΠΈΠ· ΠΊΠ»ΡŽΡ‡Π΅Π²Ρ‹Ρ… Ρ„ΡƒΠ½ΠΊΡ†ΠΈΠΉ ΡΠ½Π΄ΠΎΡ‚Π΅Π»ΠΈΠ°Π»ΡŒΠ½Ρ‹Ρ… ΠΊΠ»Π΅Ρ‚ΠΎΠΊ, Π° ΠΈΡ… Π΄ΠΈΡΡ„ΡƒΠ½ΠΊΡ†ΠΈΡŽ ΠΏΡ€ΠΈ Ρ€Π°Π·Π»ΠΈΡ‡Π½Ρ‹Ρ… заболСваниях Π²Ρ€Π°Ρ‡ΠΈ часто ΡΠ²ΡΠ·Ρ‹Π²Π°ΡŽΡ‚ с ΡƒΠΌΠ΅Π½ΡŒΡˆΠ΅Π½ΠΈΠ΅ΠΌ ΠΏΡ€ΠΎΠ΄ΡƒΠΊΡ†ΠΈΠΈ NO. ΠšΠ°ΠΊΠΎΠ²Ρ‹ ΠΆΠ΅ соврСмСнныС прСдставлСния ΠΎ Ρ€Π°Π±ΠΎΡ‚Π΅ этой систСмы? ΠŸΠΎΠΏΡ‹Ρ‚Π°Π΅ΠΌΡΡ ΠΎΡ‚Π²Π΅Ρ‚ΠΈΡ‚ΡŒ Π½Π° этот вопрос Π² нашСй ΡΡ‚Π°Ρ‚ΡŒΠ΅.

Π˜ΡΡ‚ΠΎΡ€ΠΈΡ вопроса

Π‘Π»ΠΎΠΉ ΠΊΠ»Π΅Ρ‚ΠΎΠΊ, Π²Ρ‹ΡΡ‚ΠΈΠ»Π°ΡŽΡ‰ΠΈΠΉ всС кровСносныС ΠΈ лимфатичСскиС сосуды, Π° Ρ‚Π°ΠΊΠΆΠ΅ сСрдСчныС полости, Π²ΠΏΠ΅Ρ€Π²Ρ‹Π΅ описал Π² 1847 Π³. Π’. Π¨Π²Π°Π½Π½ ΠΊΠ°ΠΊ Β«ΠΎΡ‚Ρ‡Π΅Ρ‚Π»ΠΈΠ²ΠΎ Ρ€Π°Π·Π»ΠΈΡ‡ΠΈΠΌΡƒΡŽ ΠΌΠ΅ΠΌΠ±Ρ€Π°Π½ΡƒΒ», ΠΊΠΎΡ‚ΠΎΡ€ΡƒΡŽ спустя 18 Π»Π΅Ρ‚ Π’. Гис Π½Π°Π·Π²Π°Π» эндотСлиСм. Π’ ΡΡ€Π°Π²Π½ΠΈΡ‚Π΅Π»ΡŒΠ½ΠΎ ΠΊΡ€ΡƒΠΏΠ½Ρ‹Ρ… сосудах (артСриях ΠΈ Π²Π΅Π½Π°Ρ…) этот слой слуТит Π±Π°Ρ€ΡŒΠ΅Ρ€ΠΎΠΌ ΠΌΠ΅ΠΆΠ΄Ρƒ ΠΊΡ€ΠΎΠ²ΡŒΡŽ ΠΈ Π³Π»Π°Π΄ΠΊΠΎΠΌΡ‹ΡˆΠ΅Ρ‡Π½Ρ‹ΠΌΠΈ ΠΊΠ»Π΅Ρ‚ΠΊΠ°ΠΌΠΈ, Π° стСнки ΠΌΠ΅Π»ΡŒΡ‡Π°ΠΉΡˆΠΈΡ… сосудов, капилляров, Ρ†Π΅Π»ΠΈΠΊΠΎΠΌ построСны ΠΈΠ· ΡΠ½Π΄ΠΎΡ‚Π΅Π»ΠΈΠ°Π»ΡŒΠ½Ρ‹Ρ… ΠΊΠ»Π΅Ρ‚ΠΎΠΊ. Π˜Ρ… ΠΎΠ±Ρ‰Π΅Π΅ количСство ΠΎΡ‡Π΅Π½ΡŒ Π²Π΅Π»ΠΈΠΊΠΎ: Π² Ρ‚Π΅Π»Π΅ взрослого Ρ‡Π΅Π»ΠΎΠ²Π΅ΠΊΠ° суммарная масса ΠΏΡ€Π΅Π²Ρ‹ΡˆΠ°Π΅Ρ‚ 1 ΠΊΠ³!

Π’ 50–60-Ρ… Π³ΠΎΠ΄Π°Ρ… XX Π². ΡƒΡ‡Π΅Π½Ρ‹Π΅, Π²ΠΎΠΎΡ€ΡƒΠΆΠΈΠ²ΡˆΠΈΡΡŒ элСктронным микроскопом, Π² дСталях описали строСниС эндотСлия, ΠΎΠ΄Π½Π°ΠΊΠΎ Π΅Π³ΠΎ Ρ€ΠΎΠ»ΡŒ Π² рСгуляции Ρ„ΡƒΠ½ΠΊΡ†ΠΈΠΉ сСрдСчно-сосудистой систСмы ΠΎΡΡ‚Π°Π²Π°Π»Π°ΡΡŒ нСясной. Π’ΠΏΠ»ΠΎΡ‚ΡŒ Π΄ΠΎ 1980 Π³. эндотСлий считался лишь сСлСктивно ΠΏΡ€ΠΎΠ½ΠΈΡ†Π°Π΅ΠΌΡ‹ΠΌ Π±Π°Ρ€ΡŒΠ΅Ρ€ΠΎΠΌ ΠΌΠ΅ΠΆΠ΄Ρƒ ΠΊΡ€ΠΎΠ²ΡŒΡŽ ΠΈ сосудистой стСнкой, хотя ΡƒΠΆΠ΅ Π² Ρ‚ΠΎ врСмя Π±Ρ‹Π»ΠΎ извСстно, Ρ‡Ρ‚ΠΎ ΠΎΠ½ способСн Π²Ρ‹Π΄Π΅Π»ΡΡ‚ΡŒ вСщСства, ΠΏΡ€Π΅ΠΏΡΡ‚ΡΡ‚Π²ΡƒΡŽΡ‰ΠΈΠ΅ ΡΠ²Π΅Ρ€Ρ‚Ρ‹Π²Π°Π½ΠΈΡŽ ΠΊΡ€ΠΎΠ²ΠΈ.

Начало соврСмСнным прСдставлСниям ΠΎ функциях эндотСлия Π±Ρ‹Π»ΠΎ ΠΏΠΎΠ»ΠΎΠΆΠ΅Π½ΠΎ Π² 1980 Π³., ΠΊΠΎΠ³Π΄Π° Π . Π€Π°Ρ€Ρ‡Π³ΠΎΡ‚Ρ‚ ΠΈ Π”ΠΆ. Завадски ΠΎΠ±Ρ€Π°Ρ‚ΠΈΠ»ΠΈ Π²Π½ΠΈΠΌΠ°Π½ΠΈΠ΅ Π½Π° Π΅Π³ΠΎ Ρ€ΠΎΠ»ΡŒ Π² рСгуляции тонуса сосудов . Π’ элСгантных экспСримСнтах исслСдоватСли Π΄ΠΎΠΊΠ°Π·Π°Π»ΠΈ, Ρ‡Ρ‚ΠΎ Ρ‚Π°ΠΊΠΎΠ΅ вСщСство, ΠΊΠ°ΠΊ Π°Ρ†Π΅Ρ‚ΠΈΠ»Ρ…ΠΎΠ»ΠΈΠ½, Π²Ρ‹Π·Ρ‹Π²Π°Π΅Ρ‚ расслаблСниС ΠΏΡ€Π΅ΠΏΠ°Ρ€Π°Ρ‚ΠΎΠ² Π°ΠΎΡ€Ρ‚Ρ‹, ΠΈΠ·ΠΎΠ»ΠΈΡ€ΠΎΠ²Π°Π½Π½Ρ‹Ρ… ΠΈΠ· ΠΎΡ€Π³Π°Π½ΠΈΠ·ΠΌΠ° ΠΊΡ€ΠΎΠ»ΠΈΠΊΠ°, Ρ‚ΠΎΠ»ΡŒΠΊΠΎ ΠΏΡ€ΠΈ Π½Π°Π»ΠΈΡ‡ΠΈΠΈ эндотСлия. Π­Ρ‚ΠΎ наблюдСниС оказалось ΡΡ‚ΠΎΠ»ΡŒ Π²Π°ΠΆΠ½Ρ‹ΠΌ, Ρ‡Ρ‚ΠΎ впослСдствии Π€Π°Ρ€Ρ‡Π³ΠΎΡ‚Ρ‚ стал ΠΎΠ΄Π½ΠΈΠΌ ΠΈΠ· Π»Π°ΡƒΡ€Π΅Π°Ρ‚ΠΎΠ² НобСлСвской ΠΏΡ€Π΅ΠΌΠΈΠΈ (1998). Π’ нашС врСмя зависимая ΠΎΡ‚ эндотСлия рСакция сосудов Π² ΠΎΡ‚Π²Π΅Ρ‚ Π½Π° Π°Ρ†Π΅Ρ‚ΠΈΠ»Ρ…ΠΎΠ»ΠΈΠ½ ΠΈ Π΄Ρ€ΡƒΠ³ΠΈΠ΅ вСщСства описана Π² ΠΎΠ³Ρ€ΠΎΠΌΠ½ΠΎΠΌ количСствС Π½Π°ΡƒΡ‡Π½Ρ‹Ρ… Ρ€Π°Π±ΠΎΡ‚, Π²Ρ‹ΠΏΠΎΠ»Π½Π΅Π½Π½Ρ‹Ρ… Π½Π° самых Ρ€Π°Π·Π»ΠΈΡ‡Π½Ρ‹Ρ… Π°Ρ€Ρ‚Π΅Ρ€ΠΈΠ°Π»ΡŒΠ½Ρ‹Ρ… сосудах - Π½Π΅ Ρ‚ΠΎΠ»ΡŒΠΊΠΎ ΠΊΡ€ΡƒΠΏΠ½Ρ‹Ρ…, Π½ΠΎ ΠΈ ΠΌΠ΅Π»ΠΊΠΈΡ…, Ρ€Π΅Π³ΡƒΠ»ΠΈΡ€ΡƒΡŽΡ‰ΠΈΡ… кровоснабТСниС ΠΎΡ€Π³Π°Π½ΠΎΠ² (рис. 1).

К 1986 Π³. Π²Ρ‹ΡΡΠ½ΠΈΠ»ΠΎΡΡŒ, Ρ‡Ρ‚ΠΎ расслаблСниС Π³Π»Π°Π΄ΠΊΠΎΠΉ ΠΌΡ‹ΡˆΡ†Ρ‹ сосудов Π²Ρ‹Π·Ρ‹Π²Π°Π΅Ρ‚ ΠΈΠΌΠ΅Π½Π½ΠΎ оксид Π°Π·ΠΎΡ‚Π° (NO), ΠΊΠΎΡ‚ΠΎΡ€Ρ‹ΠΉ выдСляСтся ΠΈΠ· эндотСлия ΠΏΠΎΠ΄ дСйствиСм Π°Ρ†Π΅Ρ‚ΠΈΠ»Ρ…ΠΎΠ»ΠΈΠ½Π°. Как ΠΆΠ΅ Π·Π° Ρ‚Π°ΠΊΠΎΠ΅ ΠΊΠΎΡ€ΠΎΡ‚ΠΊΠΎΠ΅ врСмя (всСго ΡˆΠ΅ΡΡ‚ΡŒ Π»Π΅Ρ‚) ΡƒΠ΄Π°Π»ΠΎΡΡŒ Π²Ρ‹Ρ‡Π»Π΅Π½ΠΈΡ‚ΡŒ NO ΠΈΠ· Π΄Π»ΠΈΠ½Π½ΠΎΠ³ΠΎ ряда Π΄Ρ€ΡƒΠ³ΠΈΡ… ΠΏΡ€Π΅Ρ‚Π΅Π½Π΄Π΅Π½Ρ‚ΠΎΠ² Π½Π° Ρ€ΠΎΠ»ΡŒ посрСдника ΠΌΠ΅ΠΆΠ΄Ρƒ эндотСлиСм ΠΈ Π³Π»Π°Π΄ΠΊΠΎΠΉ ΠΌΡ‹ΡˆΡ†Π΅ΠΉ сосудов? Π”Π΅Π»ΠΎ Π² Ρ‚ΠΎΠΌ, Ρ‡Ρ‚ΠΎ Π΅Ρ‰Π΅ Π·Π° 10 Π»Π΅Ρ‚ Π΄ΠΎ Π·Π½Π°ΠΌΠ΅Π½ΠΈΡ‚ΠΎΠΉ Ρ€Π°Π±ΠΎΡ‚Ρ‹ Π€Π°Ρ€Ρ‡Π³ΠΎΡ‚Ρ‚Π° ΠΈ Завадски Π±Ρ‹Π»ΠΎ ΠΈΠ·ΡƒΡ‡Π΅Π½ΠΎ ΡΠΎΡΡƒΠ΄ΠΎΡ€Π°ΡΡˆΠΈΡ€ΡΡŽΡ‰Π΅Π΅ дСйствиС NO. Π’Π΅Π΄ΡŒ ΠΊ Ρ‚ΠΎΠΌΡƒ Π²Ρ€Π΅ΠΌΠ΅Π½ΠΈ ΡƒΠΆΠ΅ 100 Π»Π΅Ρ‚ Π½ΠΈΡ‚Ρ€ΠΎΠ³Π»ΠΈΡ†Π΅Ρ€ΠΈΠ½ΠΎΠΌ (ΠΎΠ½ слуТит источником ΠΌΠΎΠ»Π΅ΠΊΡƒΠ» NO) Π»Π΅Ρ‡ΠΈΠ»ΠΈ ΡΡ‚Π΅Π½ΠΎΠΊΠ°Ρ€Π΄ΠΈΡŽ, Π²ΠΎΠ·Π½ΠΈΠΊΠ°ΡŽΡ‰ΡƒΡŽ ΠΈΠ·-Π·Π° спазмов сосудов сСрдца. Π˜Π΄Π΅Π½Ρ‚ΠΈΡ‡Π½ΠΎΡΡ‚ΡŒ ΡΠ½Π΄ΠΎΡ‚Π΅Π»ΠΈΠ°Π»ΡŒΠ½ΠΎΠ³ΠΎ Ρ€Π°ΡΡΠ»Π°Π±Π»ΡΡŽΡ‰Π΅Π³ΠΎ Ρ„Π°ΠΊΡ‚ΠΎΡ€Π° ΠΈ NO установили ΠΈ ΠΏΠΎ Ρ‚Π°ΠΊΠΈΠΌ показатСлям, ΠΊΠ°ΠΊ чрСзвычайная Π½Π΅ΡΡ‚Π°Π±ΠΈΠ»ΡŒΠ½ΠΎΡΡ‚ΡŒ (особСнно Π² присутствии Π°ΠΊΡ‚ΠΈΠ²Π½Ρ‹Ρ… Ρ„ΠΎΡ€ΠΌ кислорода), инактивация ΠΏΡ€ΠΈ взаимодСйствии с Π³Π΅ΠΌΠΎΠ³Π»ΠΎΠ±ΠΈΠ½ΠΎΠΌ ΠΈ родствСнными Π±Π΅Π»ΠΊΠ°ΠΌΠΈ, Π° Ρ‚Π°ΠΊΠΆΠ΅ ΡΠΏΠΎΡΠΎΠ±Π½ΠΎΡΡ‚ΡŒ Π²Ρ‹Π·Ρ‹Π²Π°Ρ‚ΡŒ сходныС биохимичСскиС измСнСния Π² Π³Π»Π°Π΄ΠΊΠΎΠΌΡ‹ΡˆΠ΅Ρ‡Π½Ρ‹Ρ… ΠΊΠ»Π΅Ρ‚ΠΊΠ°Ρ… сосудов.

Π’ ΠΎΡ€Π³Π°Π½ΠΈΠ·ΠΌΠ΅ Ρ‡Π΅Π»ΠΎΠ²Π΅ΠΊΠ° ΠΈ ΠΆΠΈΠ²ΠΎΡ‚Π½Ρ‹Ρ… оксид Π°Π·ΠΎΡ‚Π° - ΠΎΠ΄ΠΈΠ½ ΠΈΠ· ΠΊΠ»ΡŽΡ‡Π΅Π²Ρ‹Ρ… эндогСнных рСгуляторов сСрдСчно-сосудистой ΠΈ Π΄Ρ€ΡƒΠ³ΠΈΡ… систСм. Π’ 1992 Π³. Π΅Π³ΠΎ Π½Π°Π·Π²Π°Π»ΠΈ ΠΌΠΎΠ»Π΅ΠΊΡƒΠ»ΠΎΠΉ Π³ΠΎΠ΄Π°, Π° Π΅ΠΆΠ΅Π³ΠΎΠ΄Π½ΠΎΠ΅ число ΠΏΡƒΠ±Π»ΠΈΠΊΠ°Ρ†ΠΈΠΉ ΠΎ Π΅Π³ΠΎ функциях Π² ΠΎΡ€Π³Π°Π½ΠΈΠ·ΠΌΠ΅ сСгодня составляСт нСсколько тысяч. Π­Π½Π΄ΠΎΡ‚Π΅Π»ΠΈΠΉ ΠΌΠΎΠΆΠ½ΠΎ Π½Π°Π·Π²Π°Ρ‚ΡŒ гигантским эндокринным ΠΎΡ€Π³Π°Π½ΠΎΠΌ, Π² ΠΊΠΎΡ‚ΠΎΡ€ΠΎΠΌ ΠΊΠ»Π΅Ρ‚ΠΊΠΈ Π½Π΅ собраны Π²ΠΎΠ΅Π΄ΠΈΠ½ΠΎ, ΠΊΠ°ΠΊ Π² ΠΆΠ΅Π»Π΅Π·Π°Ρ… Π²Π½ΡƒΡ‚Ρ€Π΅Π½Π½Π΅ΠΉ сСкрСции, Π° рассрСдоточСны Π² сосудах, ΠΏΡ€ΠΎΠ½ΠΈΠ·Ρ‹Π²Π°ΡŽΡ‰ΠΈΡ… всС ΠΎΡ€Π³Π°Π½Ρ‹ ΠΈ Ρ‚ΠΊΠ°Π½ΠΈ нашСго Ρ‚Π΅Π»Π°. Π’ Π½ΠΎΡ€ΠΌΠ°Π»ΡŒΠ½Ρ‹Ρ… физиологичСских условиях эндотСлий активируСтся Π³Π»Π°Π²Π½Ρ‹ΠΌ ΠΎΠ±Ρ€Π°Π·ΠΎΠΌ мСханичСски: напряТСниСм сдвига, создаваСмым ΠΏΠΎΡ‚ΠΎΠΊΠΎΠΌ ΠΊΡ€ΠΎΠ²ΠΈ , ΠΈΠ»ΠΈ растяТСниСм сосуда ΠΏΠΎΠ΄ Π΅Π΅ Π΄Π°Π²Π»Π΅Π½ΠΈΠ΅ΠΌ. ΠšΡ€ΠΎΠΌΠ΅ Ρ‚ΠΎΠ³ΠΎ, ΡΠ½Π΄ΠΎΡ‚Π΅Π»ΠΈΠ°Π»ΡŒΠ½Ρ‹Π΅ ΠΊΠ»Π΅Ρ‚ΠΊΠΈ ΠΌΠΎΠ³ΡƒΡ‚ Π°ΠΊΡ‚ΠΈΠ²ΠΈΡ€ΠΎΠ²Π°Ρ‚ΡŒΡΡ рСгуляторными ΠΌΠΎΠ»Π΅ΠΊΡƒΠ»Π°ΠΌΠΈ, Π½Π°ΠΏΡ€ΠΈΠΌΠ΅Ρ€ ΠΏΡƒΡ€ΠΈΠ½ΠΎΠ²Ρ‹ΠΌΠΈ соСдинСниями (АВЀ ΠΈ АДЀ), ΠΏΠ΅ΠΏΡ‚ΠΈΠ΄Π°ΠΌΠΈ (Π±Ρ€Π°Π΄ΠΈΠΊΠΈΠ½ΠΈΠ½ΠΎΠΌ, ΠΊΠ°Π»ΡŒΡ†ΠΈΡ‚ΠΎΠ½ΠΈΠ½-Π³Π΅Π½-родствСнным ΠΏΠ΅ΠΏΡ‚ΠΈΠ΄ΠΎΠΌ , субстанциСй Π  ΠΈ Π΄Ρ€.).

ΠšΡ€ΠΎΠΌΠ΅ оксида Π°Π·ΠΎΡ‚Π° Π² ΠΊΠ»Π΅Ρ‚ΠΊΠ°Ρ… эндотСлия ΡΠΈΠ½Ρ‚Π΅Π·ΠΈΡ€ΡƒΡŽΡ‚ΡΡ ΠΈ Π΄Ρ€ΡƒΠ³ΠΈΠ΅ вСщСства, Π²Π»ΠΈΡΡŽΡ‰ΠΈΠ΅ Π½Π° тонус сосудов, кровоснабТСниС Ρ‚ΠΊΠ°Π½Π΅ΠΉ ΠΈ Π°Ρ€Ρ‚Π΅Ρ€ΠΈΠ°Π»ΡŒΠ½ΠΎΠ΅ Π΄Π°Π²Π»Π΅Π½ΠΈΠ΅ . Π’Π°ΠΊ, ΠΏΠΎΠΌΠΎΡ‰Π½ΠΈΠΊΠ°ΠΌΠΈ NO Π² расслаблСнии сосудов ΠΌΠΎΠ³ΡƒΡ‚ Π±Ρ‹Ρ‚ΡŒ простациклин (простагландин I 2) ΠΈ ΡΠ½Π΄ΠΎΡ‚Π΅Π»ΠΈΠ°Π»ΡŒΠ½Ρ‹ΠΉ Π³ΠΈΠΏΠ΅Ρ€ΠΏΠΎΠ»ΡΡ€ΠΈΠ·ΡƒΡŽΡ‰ΠΈΠΉ Ρ„Π°ΠΊΡ‚ΠΎΡ€. Доля ΠΈΡ… участия зависит ΠΎΡ‚ ΠΏΠΎΠ»Π° ΠΈ Π²ΠΈΠ΄Π° ΠΆΠΈΠ²ΠΎΡ‚Π½ΠΎΠ³ΠΎ, Ρ‚ΠΈΠΏΠ° сосудистого русла ΠΈ Ρ€Π°Π·ΠΌΠ΅Ρ€ΠΎΠ² сосуда. НапримСр, дСйствиС NO сильнСС проявляСтся Π² ΡΡ€Π°Π²Π½ΠΈΡ‚Π΅Π»ΡŒΠ½ΠΎ ΠΊΡ€ΡƒΠΏΠ½Ρ‹Ρ… сосудах, Π° Π³ΠΈΠΏΠ΅Ρ€ΠΏΠΎΠ»ΡΡ€ΠΈΠ·ΡƒΡŽΡ‰Π΅Π³ΠΎ Ρ„Π°ΠΊΡ‚ΠΎΡ€Π° - Π² Π±ΠΎΠ»Π΅Π΅ ΠΌΠ΅Π»ΠΊΠΈΡ….

Π’ эндотСлии ΠΎΠ±Ρ€Π°Π·ΡƒΡŽΡ‚ΡΡ Π½Π΅ Ρ‚ΠΎΠ»ΡŒΠΊΠΎ ΡΠΎΡΡƒΠ΄ΠΎΡ€Π°ΡΡˆΠΈΡ€ΡΡŽΡ‰ΠΈΠ΅ вСщСства, Π½ΠΎ ΠΈ ΡΠΎΡΡƒΠ΄ΠΎΡΡƒΠΆΠΈΠ²Π°ΡŽΡ‰ΠΈΠ΅: Π½Π΅ΠΊΠΎΡ‚ΠΎΡ€Ρ‹Π΅ простагландины, тромбоксан, ΠΏΠ΅ΠΏΡ‚ΠΈΠ΄Ρ‹ эндотСлин-1 ΠΈ Π°Π½Π³ΠΈΠΎΡ‚Π΅Π½Π·ΠΈΠ½ II, супСроксиданион. Π’ Π·Π΄ΠΎΡ€ΠΎΠ²ΠΎΠΌ ΠΎΡ€Π³Π°Π½ΠΈΠ·ΠΌΠ΅ сСкрСторная Π°ΠΊΡ‚ΠΈΠ²Π½ΠΎΡΡ‚ΡŒ эндотСлия Π½Π°ΠΏΡ€Π°Π²Π»Π΅Π½Π° Π½Π° ΠΏΡ€ΠΎΠ΄ΡƒΠΊΡ†ΠΈΡŽ ΡΠΎΡΡƒΠ΄ΠΎΡ€Π°ΡΡˆΠΈΡ€ΡΡŽΡ‰ΠΈΡ… Ρ„Π°ΠΊΡ‚ΠΎΡ€ΠΎΠ². Но ΠΏΡ€ΠΈ Ρ€Π°Π·Π»ΠΈΡ‡Π½Ρ‹Ρ… заболСваниях (систСмной ΠΈΠ»ΠΈ Π»Π΅Π³ΠΎΡ‡Π½ΠΎΠΉ Π³ΠΈΠΏΠ΅Ρ€Ρ‚Π΅Π½Π·ΠΈΠΈ, ишСмии ΠΌΠΈΠΎΠΊΠ°Ρ€Π΄Π°, сахарном Π΄ΠΈΠ°Π±Π΅Ρ‚Π΅ ΠΈ Ρ‚. Π΄.) ΠΈΠ»ΠΈ Π² Π·Π΄ΠΎΡ€ΠΎΠ²ΠΎΠΌ ΠΎΡ€Π³Π°Π½ΠΈΠ·ΠΌΠ΅ ΠΏΡ€ΠΈ старСнии сСкрСторный Ρ„Π΅Π½ΠΎΡ‚ΠΈΠΏ эндотСлия ΠΌΠΎΠΆΠ΅Ρ‚ ΠΌΠ΅Π½ΡΡ‚ΡŒΡΡ Π² сторону ΡΠΎΡΡƒΠ΄ΠΎΡΡƒΠΆΠΈΠ²Π°ΡŽΡ‰ΠΈΡ… влияний .

НСсмотря Π½Π° ΠΌΠ½ΠΎΠ³ΠΎΠΎΠ±Ρ€Π°Π·ΠΈΠ΅ рСгуляторных ΠΌΠ΅Ρ…Π°Π½ΠΈΠ·ΠΌΠΎΠ², зависимых ΠΎΡ‚ эндотСлия, Π΅Π³ΠΎ Π½ΠΎΡ€ΠΌΠ°Π»ΡŒΠ½ΡƒΡŽ Ρ„ΡƒΠ½ΠΊΡ†ΠΈΡŽ Ρ‡Π°Ρ‰Π΅ всСго ΡΠ²ΡΠ·Ρ‹Π²Π°ΡŽΡ‚ со ΡΠΏΠΎΡΠΎΠ±Π½ΠΎΡΡ‚ΡŒΡŽ ΡΠ΅ΠΊΡ€Π΅Ρ‚ΠΈΡ€ΠΎΠ²Π°Ρ‚ΡŒ NO. Когда ΠΏΡ€ΠΈ заболСваниях эндотСлий измСняСт свои свойства, Π²Ρ€Π°Ρ‡ΠΈ Π½Π°Π·Ρ‹Π²Π°ΡŽΡ‚ Ρ‚Π°ΠΊΠΎΠ΅ состояниС дисфункциСй эндотСлия, подразумСвая ΠΏΡ€ΠΈ этом ΡƒΠΌΠ΅Π½ΡŒΡˆΠ΅Π½ΠΈΠ΅ ΠΏΡ€ΠΎΠ΄ΡƒΠΊΡ†ΠΈΠΈ NO. Π’ связи с Ρ‚Π°ΠΊΠΎΠΉ Π²Π°ΠΆΠ½ΠΎΡΡ‚ΡŒΡŽ NO ΠΌΡ‹ рассмотрим соврСмСнныС прСдставлСния ΠΎ Π΅Π³ΠΎ рСгуляторной Ρ€ΠΎΠ»ΠΈ, сначала Π² Π½ΠΎΡ€ΠΌΠ΅, Π° Π·Π°Ρ‚Π΅ΠΌ ΠΏΡ€ΠΈ Π½Π΅ΠΊΠΎΡ‚ΠΎΡ€Ρ‹Ρ… Ρ„ΠΎΡ€ΠΌΠ°Ρ… сосудистой ΠΏΠ°Ρ‚ΠΎΠ»ΠΎΠ³ΠΈΠΈ.

Π‘ΠΈΠ½Ρ‚Π΅Π· ΠΈ рСгуляция NO Π² эндотСлии

Π’ ΠΏΡ€ΠΈΡ€ΠΎΠ΄Π΅ синтСз оксида Π°Π·ΠΎΡ‚Π° ΠΌΠΎΠΆΠ΅Ρ‚ ΠΏΡ€ΠΎΡ‚Π΅ΠΊΠ°Ρ‚ΡŒ ΠΏΠΎ Ρ€Π°Π·Π»ΠΈΡ‡Π½Ρ‹ΠΌ путям. Π’Π°ΠΊ, Π² тропосфСрС ΠΎΠ½ образуСтся ΠΈΠ· O 2 ΠΈ N 2 ΠΏΠΎΠ΄ дСйствиСм Π³Ρ€ΠΎΠ·ΠΎΠ²Ρ‹Ρ… разрядов, Π² растСниях - благодаря фотохимичСской Ρ€Π΅Π°ΠΊΡ†ΠΈΠΈ ΠΌΠ΅ΠΆΠ΄Ρƒ NO 2 ΠΈ ΠΊΠ°Ρ€ΠΎΡ‚ΠΈΠ½ΠΎΠΈΠ΄Π°ΠΌΠΈ, Π° Π² ΠΎΡ€Π³Π°Π½ΠΈΠ·ΠΌΠ΅ ΠΆΠΈΠ²ΠΎΡ‚Π½Ρ‹Ρ… - ΠΏΡ€ΠΈ взаимодСйствии Π½ΠΈΡ‚Ρ€ΠΈΡ‚ΠΎΠ² ΠΈ Π½ΠΈΡ‚Ρ€Π°Ρ‚ΠΎΠ² с Π±Π΅Π»ΠΊΠ°ΠΌΠΈ, содСрТащими Π°Ρ‚ΠΎΠΌΡ‹ ΠΌΠ΅Ρ‚Π°Π»Π»ΠΎΠ² (Π½Π°ΠΏΡ€ΠΈΠΌΠ΅Ρ€, с Π³Π΅ΠΌΠΎΠ³Π»ΠΎΠ±ΠΈΠ½ΠΎΠΌ). ВсС пСрСчислСнныС Ρ€Π΅Π°ΠΊΡ†ΠΈΠΈ ΠΈΠ΄ΡƒΡ‚ Π±Π΅Π· участия биологичСских ΠΊΠ°Ρ‚Π°Π»ΠΈΠ·Π°Ρ‚ΠΎΡ€ΠΎΠ² - Π±Π΅Π»ΠΊΠΎΠ²-Ρ„Π΅Ρ€ΠΌΠ΅Π½Ρ‚ΠΎΠ², поэтому ΠΊΠΎΠ½Ρ‚Ρ€ΠΎΠ»ΠΈΡ€ΠΎΠ²Π°Ρ‚ΡŒ ΡΠΊΠΎΡ€ΠΎΡΡ‚ΡŒ ΡΡ€Π°Π²Π½ΠΈΡ‚Π΅Π»ΡŒΠ½ΠΎ Ρ‚Ρ€ΡƒΠ΄Π½ΠΎ. Однако Π² ΠΎΡ€Π³Π°Π½ΠΈΠ·ΠΌΠ΅ ΠΆΠΈΠ²ΠΎΡ‚Π½Ρ‹Ρ… основноС количСство NO ΠΊΠ°ΠΊ рСгулятора физиологичСских процСссов образуСтся ΠΏΠΎΠ΄ дСйствиСм ΡΠΏΠ΅Ρ†ΠΈΠ°Π»ΡŒΠ½Ρ‹Ρ… Ρ„Π΅Ρ€ΠΌΠ΅Π½Ρ‚ΠΎΠ² NO-синтаз (NOS), Π° источником Π°Ρ‚ΠΎΠΌΠ° Π°Π·ΠΎΡ‚Π° слуТит аминокислота L-Π°Ρ€Π³ΠΈΠ½ΠΈΠ½ [ , ].

БущСствуСт нСсколько разновидностСй (ΠΈΠ·ΠΎΡ„ΠΎΡ€ΠΌ) NO-синтаз, ΠΊΠΎΡ‚ΠΎΡ€Ρ‹Π΅ ΠΊΠΎΠ΄ΠΈΡ€ΡƒΡŽΡ‚ΡΡ Ρ€Π°Π·Π½Ρ‹ΠΌΠΈ Π³Π΅Π½Π°ΠΌΠΈ. Π’ 1990 Π³. ΠΈΠ· ΠΌΠΎΠ·Π³Π° крысы Π²Ρ‹Π΄Π΅Π»ΠΈΠ»ΠΈ Π½Π΅ΠΉΡ€ΠΎΠ½Π°Π»ΡŒΠ½ΡƒΡŽ Ρ„ΠΎΡ€ΠΌΡƒ Ρ„Π΅Ρ€ΠΌΠ΅Π½Ρ‚Π° (nNOS). Π§ΡƒΡ‚ΡŒ ΠΏΠΎΠ·ΠΆΠ΅ Π² ΠΊΠ»Π΅Ρ‚ΠΊΠ°Ρ… ΠΈΠΌΠΌΡƒΠ½Π½ΠΎΠΉ систСмы (ΠΌΠ°ΠΊΡ€ΠΎΡ„Π°Π³Π°Ρ…) ΠΎΠ±Π½Π°Ρ€ΡƒΠΆΠΈΠ»ΠΈ ΠΈΠ½Π΄ΡƒΡ†ΠΈΠ±Π΅Π»ΡŒΠ½ΡƒΡŽ NOS (iNOS), Π° Π² эндотСлии - ΡΠ½Π΄ΠΎΡ‚Π΅Π»ΠΈΠ°Π»ΡŒΠ½ΡƒΡŽ NOS (eNOS). Π•Ρ‰Π΅ ΠΎΠ΄Π½Π° ΠΈΠ·ΠΎΡ„ΠΎΡ€ΠΌΠ° NOS локализуСтся Π² митохондриях, ΠΎΠ½Π° Ρ€Π΅Π³ΡƒΠ»ΠΈΡ€ΡƒΠ΅Ρ‚ процСссы ΠΊΠ»Π΅Ρ‚ΠΎΡ‡Π½ΠΎΠ³ΠΎ дыхания. ΠŸΠΎΡΠΊΠΎΠ»ΡŒΠΊΡƒ Π² синтСзС NO участвуСт большоС число ΠΊΠΎΡ„Π°ΠΊΡ‚ΠΎΡ€ΠΎΠ², всС ΠΈΠ·ΠΎΡ„ΠΎΡ€ΠΌΡ‹ Ρ„Π΅Ρ€ΠΌΠ΅Π½Ρ‚Π° ΠΈΠΌΠ΅ΡŽΡ‚ для Π½ΠΈΡ… спСцифичСскиС участки связывания. КаТдая ΠΌΠΎΠ»Π΅ΠΊΡƒΠ»Π° NOS состоит ΠΈΠ· Π΄Π²ΡƒΡ… ΠΎΠ΄ΠΈΠ½Π°ΠΊΠΎΠ²Ρ‹Ρ… ΠΏΠΎΠ»ΠΎΠ²ΠΈΠ½ΠΎΠΊ. Для ΠΈΡ… объСдинСния Π² Π΄ΠΈΠΌΠ΅Ρ€ Π½Π΅ΠΎΠ±Ρ…ΠΎΠ΄ΠΈΠΌ ΠΊΠΎΡ„Π°ΠΊΡ‚ΠΎΡ€ Ρ‚Π΅Ρ‚Ρ€Π°Π³ΠΈΠ΄Ρ€ΠΎΠ±ΠΈΠΎΠΏΡ‚Π΅Ρ€ΠΈΠ½. ΠŸΡ€ΠΈ Π΅Π³ΠΎ нСдостаткС eNOS ΠΏΠ΅Ρ€Π΅ΠΊΠ»ΡŽΡ‡Π°Π΅Ρ‚ΡΡ Π½Π° ΠΏΡ€ΠΎΠ΄ΡƒΠΊΡ†ΠΈΡŽ Π°ΠΊΡ‚ΠΈΠ²Π½Ρ‹Ρ… Ρ„ΠΎΡ€ΠΌ кислорода (супСроксид-Π°Π½ΠΈΠΎΠ½Π° ΠΈ Н 2 О 2), Ρ‡Ρ‚ΠΎ ΠΌΠΎΠΆΠ΅Ρ‚ ΠΏΡ€ΠΈΠ²ΠΎΠ΄ΠΈΡ‚ΡŒ ΠΊ ΠΏΠΎΠ²Ρ€Π΅ΠΆΠ΄Π΅Π½ΠΈΡŽ эндотСлия ΠΈ Π΄Ρ€ΡƒΠ³ΠΈΡ… ΠΊΠ»Π΅Ρ‚ΠΎΠΊ сосудистой стСнки.

Π”Π²Π΅ ΠΈΠ·ΠΎΡ„ΠΎΡ€ΠΌΡ‹ Ρ„Π΅Ρ€ΠΌΠ΅Π½Ρ‚Π° - eNOS ΠΈ nNOS - Π½Π°Π·Ρ‹Π²Π°ΡŽΡ‚ конститутивными, ΠΏΠΎΡ‚ΠΎΠΌΡƒ Ρ‡Ρ‚ΠΎ ΠΎΠ½ΠΈ всСгда ΠΏΡ€ΠΈΡΡƒΡ‚ΡΡ‚Π²ΡƒΡŽΡ‚ Π² ΠΊΠ»Π΅Ρ‚ΠΊΠ°Ρ… ΠΈ ΡΠΈΠ½Ρ‚Π΅Π·ΠΈΡ€ΡƒΡŽΡ‚ NO Π² ΠΎΡ‚Π½ΠΎΡΠΈΡ‚Π΅Π»ΡŒΠ½ΠΎ Π½Π΅Π±ΠΎΠ»ΡŒΡˆΠΈΡ… (ΠΏΠΎ ΡΡ€Π°Π²Π½Π΅Π½ΠΈΡŽ с iNOS) количСствах, ΠΏΡ€ΠΈΡ‡Π΅ΠΌ Π°ΠΊΡ‚ΠΈΠ²Π½ΠΎΡΡ‚ΡŒ этих ΠΈΠ·ΠΎΡ„ΠΎΡ€ΠΌ рСгулируСтся физиологичСскими стимулами. Π’ ΠΎΡ‚Π»ΠΈΡ‡ΠΈΠ΅ ΠΎΡ‚ Π½ΠΈΡ…, iNOS постоянно синтСзируСтся лишь Π² Π½Π΅ΠΊΠΎΡ‚ΠΎΡ€Ρ‹Ρ… ΠΊΠ»Π΅Ρ‚ΠΊΠ°Ρ…, Π½Π°ΠΏΡ€ΠΈΠΌΠ΅Ρ€ Π² ΠΌΠ°ΠΊΡ€ΠΎΡ„Π°Π³Π°Ρ…, Π° Π² ΡΠ½Π΄ΠΎΡ‚Π΅Π»ΠΈΠ°Π»ΡŒΠ½Ρ‹Ρ…, Π½Π΅Ρ€Π²Π½Ρ‹Ρ… ΠΈ ΠΌΠ½ΠΎΠ³ΠΈΡ… Π΄Ρ€ΡƒΠ³ΠΈΡ… появляСтся лишь Π² ΠΎΡ‚Π²Π΅Ρ‚ Π½Π° внСшниС, Π² основном Π²ΠΎΡΠΏΠ°Π»ΠΈΡ‚Π΅Π»ΡŒΠ½Ρ‹Π΅, стимулы (Π½Π°ΠΏΡ€ΠΈΠΌΠ΅Ρ€, элСмСнты ΠΊΠ»Π΅Ρ‚ΠΎΡ‡Π½Ρ‹Ρ… стСнок Π±Π°ΠΊΡ‚Π΅Ρ€ΠΈΠΉ - Π±Π°ΠΊΡ‚Π΅Ρ€ΠΈΠ°Π»ΡŒΠ½Ρ‹Π΅ липополисахариды). Активная iNOS ΠΏΡ€ΠΎΠ΄ΡƒΡ†ΠΈΡ€ΡƒΠ΅Ρ‚ NO Π² 1000 Ρ€Π°Π· быстрСС, Ρ‡Π΅ΠΌ eNOS ΠΈ nNOS. ΠœΠ°ΠΊΡ€ΠΎΡ„Π°Π³ΠΈ ΠΈΡΠΏΠΎΠ»ΡŒΠ·ΡƒΡŽΡ‚ Ρ‚Π°ΠΊΠΈΠ΅ большиС количСства NO для умСрщвлСния Π²ΠΎΠ·Π±ΡƒΠ΄ΠΈΡ‚Π΅Π»Π΅ΠΉ ΠΏΠ΅Ρ€Π΅Π΄ Ρ‚Π΅ΠΌ, ΠΊΠ°ΠΊ ΠΈΡ… ΡƒΠ½ΠΈΡ‡Ρ‚ΠΎΠΆΠΈΡ‚ΡŒ.

Π’Π°ΠΊΠΈΠΌ ΠΎΠ±Ρ€Π°Π·ΠΎΠΌ, основная NO-синтаза Π² сосудистой стСнкС - eNOS, ΠΈ содСрТится ΠΎΠ½Π° прСимущСствСнно Π² эндотСлии. Π’Ρ€Π°Π½ΡΠΊΡ€ΠΈΠΏΡ†ΠΈΡŽ Π³Π΅Π½Π° eNOS Π² Π³Π»Π°Π΄ΠΊΠΎΠΌΡ‹ΡˆΠ΅Ρ‡Π½Ρ‹Ρ… ΠΊΠ»Π΅Ρ‚ΠΊΠ°Ρ… ΠΏΡ€Π΅Π΄ΠΎΡ‚Π²Ρ€Π°Ρ‰Π°ΡŽΡ‚ ΡΠΏΠ΅Ρ†ΠΈΠ°Π»ΡŒΠ½Ρ‹Π΅ ΠΌΠ΅Ρ…Π°Π½ΠΈΠ·ΠΌΡ‹, Π½Π°ΠΏΡ€ΠΈΠΌΠ΅Ρ€ ΠΌΠ΅Ρ‚ΠΈΠ»ΠΈΡ€ΠΎΠ²Π°Π½ΠΈΠ΅ «стартового» участка. Π‘ΠΈΠ½Ρ‚Π°Π·Π° связываСтся с Π½Π°Ρ€ΡƒΠΆΠ½ΠΎΠΉ ΠΌΠ΅ΠΌΠ±Ρ€Π°Π½ΠΎΠΉ ΡΠ½Π΄ΠΎΡ‚Π΅Π»ΠΈΠ°Π»ΡŒΠ½ΠΎΠΉ ΠΊΠ»Π΅Ρ‚ΠΊΠΈ Π² особых впячиваниях, ΠΊΠ°Π²Π΅ΠΎΠ»Π°Ρ…, Π³Π΄Π΅ сосрСдоточСно большоС количСство рСгуляторных ΠΌΠΎΠ»Π΅ΠΊΡƒΠ» (Ρ€Π°Π·Π»ΠΈΡ‡Π½Ρ‹Ρ… ΠΈΠΎΠ½Π½Ρ‹Ρ… ΠΊΠ°Π½Π°Π»ΠΎΠ² ΠΈ Ρ€Π΅Ρ†Π΅ΠΏΡ‚ΠΎΡ€ΠΎΠ²). Вакая «фиксация» Ρ„Π΅Ρ€ΠΌΠ΅Π½Ρ‚Π° обСспСчиваСт Π΅Π³ΠΎ Ρ„ΡƒΠ½ΠΊΡ†ΠΈΠΎΠ½Π°Π»ΡŒΠ½ΡƒΡŽ связь с Ρ€Π΅Ρ†Π΅ΠΏΡ‚ΠΎΡ€Π°ΠΌΠΈ ΠΈ ΠΊΠ°Π½Π°Π»Π°ΠΌΠΈ, Ρ‡Ρ‚ΠΎ ΠΎΠ±Π»Π΅Π³Ρ‡Π°Π΅Ρ‚ Ρ€Π΅Π³ΡƒΠ»ΡΡ†ΠΈΡŽ активности eNOS. Π’ ΠΊΠ°Π²Π΅ΠΎΠ»Π°Ρ… локализуСтся Π±Π΅Π»ΠΎΠΊ ΠΊΠ°Π²Π΅ΠΎΠ»ΠΈΠ½, ΠΊΠΎΡ‚ΠΎΡ€Ρ‹ΠΉ Ρ‚ΠΎΡ€ΠΌΠΎΠ·ΠΈΡ‚ Π°ΠΊΡ‚ΠΈΠ²Π½ΠΎΡΡ‚ΡŒ Ρ„Π΅Ρ€ΠΌΠ΅Π½Ρ‚Π° Π² отсутствиС ΠΏΠΎΠ±ΡƒΠΆΠ΄Π°ΡŽΡ‰ΠΈΡ… стимулов.

Π€ΡƒΠ½ΠΊΡ†ΠΈΠΎΠ½Π°Π»ΡŒΠ½Π°Ρ Ρ€ΠΎΠ»ΡŒ ΡΠ½Π΄ΠΎΡ‚Π΅Π»ΠΈΠ°Π»ΡŒΠ½ΠΎΠΉ NO-синтазы зависит ΠΎΡ‚ количСства ΠΌΠΎΠ»Π΅ΠΊΡƒΠ» Π² ΠΊΠ»Π΅Ρ‚ΠΊΠ΅ (уровня экспрСссии Π³Π΅Π½Π° eNOS) ΠΈ ΠΎΡ‚ Π΅Π΅ активности. Π‘Π»Π΅Π΄ΡƒΠ΅Ρ‚ ΠΎΡ‚ΠΌΠ΅Ρ‚ΠΈΡ‚ΡŒ, Ρ‡Ρ‚ΠΎ синтСз Π½ΠΎΠ²Ρ‹Ρ… ΠΌΠΎΠ»Π΅ΠΊΡƒΠ» Π±Π΅Π»ΠΊΠ° ΡΡ€Π°Π²Π½ΠΈΡ‚Π΅Π»ΡŒΠ½ΠΎ Π΄Π»ΠΈΡ‚Π΅Π»ΡŒΠ½Ρ‹ΠΉ, поэтому ΠΎΠ½ ΠΈΡΠΏΠΎΠ»ΡŒΠ·ΡƒΠ΅Ρ‚ΡΡ для обСспСчСния Π΄ΠΎΠ»Π³ΠΎΠ²Ρ€Π΅ΠΌΠ΅Π½Π½Ρ‹Ρ… ΠΈΠ·ΠΌΠ΅Π½Π΅Π½ΠΈΠΉ ΠΏΡ€ΠΎΠ΄ΡƒΠΊΡ†ΠΈΠΈ NO, Π½Π°ΠΏΡ€ΠΈΠΌΠ΅Ρ€, ΠΏΡ€ΠΈ Π°Π΄Π°ΠΏΡ‚Π°Ρ†ΠΈΠΈ сосудистой систСмы ΠΊ физичСской Π½Π°Π³Ρ€ΡƒΠ·ΠΊΠ΅ ΠΈΠ»ΠΈ ΠΆΠ΅ ΠΊ высокогорной гипоксии. Для быстрого контроля синтСза NO ΠΈΡΠΏΠΎΠ»ΡŒΠ·ΡƒΡŽΡ‚ΡΡ Π΄Ρ€ΡƒΠ³ΠΈΠ΅ ΠΌΠ΅Ρ…Π°Π½ΠΈΠ·ΠΌΡ‹, ΠΏΡ€Π΅ΠΆΠ΄Π΅ всСго ΠΈΠ·ΠΌΠ΅Π½Π΅Π½ΠΈΠ΅ Π²Π½ΡƒΡ‚Ρ€ΠΈΠΊΠ»Π΅Ρ‚ΠΎΡ‡Π½ΠΎΠΉ ΠΊΠΎΠ½Ρ†Π΅Π½Ρ‚Ρ€Π°Ρ†ΠΈΠΈ Π‘Π° 2+ , ΡƒΠ½ΠΈΠ²Π΅Ρ€ΡΠ°Π»ΡŒΠ½ΠΎΠ³ΠΎ рСгулятора ΠΊΠ»Π΅Ρ‚ΠΎΡ‡Π½Ρ‹Ρ… Ρ„ΡƒΠ½ΠΊΡ†ΠΈΠΉ . Π‘Ρ€Π°Π·Ρƒ ΠΎΡ‚ΠΌΠ΅Ρ‚ΠΈΠΌ, Ρ‡Ρ‚ΠΎ такая физиологичСская рСгуляция свойствСнна лишь eNOS ΠΈ nNOS, Π² Ρ‚ΠΎ врСмя ΠΊΠ°ΠΊ для iNOS (нСзависимого ΠΎΡ‚ Π‘Π° 2+ Ρ„Π΅Ρ€ΠΌΠ΅Π½Ρ‚Π°) ΠΎΠ½Π° происходит Π² основном Π½Π° ΡƒΡ€ΠΎΠ²Π½Π΅ экспрСссии Π³Π΅Π½Π°.

ΠŸΠΎΠ²Ρ‹ΡˆΠ΅Π½ΠΈΠ΅ ΠΊΠΎΠ½Ρ†Π΅Π½Ρ‚Ρ€Π°Ρ†ΠΈΠΈ Π‘Π° 2+ Π΄ΠΎ ΠΎΠΏΡ€Π΅Π΄Π΅Π»Π΅Π½Π½ΠΎΠ³ΠΎ ΠΏΠΎΡ€ΠΎΠ³ΠΎΠ²ΠΎΠ³ΠΎ уровня слуТит Π½Π΅ΠΏΡ€Π΅ΠΌΠ΅Π½Π½Ρ‹ΠΌ условиСм отщСплСния ΡΠ½Π΄ΠΎΡ‚Π΅Π»ΠΈΠ°Π»ΡŒΠ½ΠΎΠΉ NO-синтазы ΠΎΡ‚ ΠΊΠ°Π²Π΅ΠΎΠ»ΠΈΠ½Π° ΠΈ Π΅Π΅ ΠΏΠ΅Ρ€Π΅Ρ…ΠΎΠ΄Π° Π² Π°ΠΊΡ‚ΠΈΠ²Π½ΠΎΠ΅ состояниС. Помимо Π‘Π° 2+ большоС Π·Π½Π°Ρ‡Π΅Π½ΠΈΠ΅ для рСгуляции активности eNOS ΠΈΠΌΠ΅Π΅Ρ‚ фосфорилированиС, Ρ‚. Π΅. ΠΊΠΎΠ²Π°Π»Π΅Π½Ρ‚Π½ΠΎΠ΅ присоСдинСниС остатка фосфорной кислоты, осущСствляСмоС Π²Π½ΡƒΡ‚Ρ€ΠΈΠΊΠ»Π΅Ρ‚ΠΎΡ‡Π½Ρ‹ΠΌΠΈ Ρ„Π΅Ρ€ΠΌΠ΅Π½Ρ‚Π°ΠΌΠΈ - ΠΏΡ€ΠΎΡ‚Π΅ΠΈΠ½ΠΊΠΈΠ½Π°Π·Π°ΠΌΠΈ. ЀосфорилированиС измСняСт ΡΠΏΠΎΡΠΎΠ±Π½ΠΎΡΡ‚ΡŒ eNOS Π°ΠΊΡ‚ΠΈΠ²ΠΈΡ€ΠΎΠ²Π°Ρ‚ΡŒΡΡ ΠΏΠΎΠ΄ дСйствиСм ΠΊΠ°Π»ΡŒΡ†ΠΈΡ (рис. 2). ΠŸΡ€ΠΎΡ‚Π΅ΠΈΠ½ΠΊΠΈΠ½Π°Π·Ρ‹ ΠΏΡ€ΠΈΡΠΎΠ΅Π΄ΠΈΠ½ΡΡŽΡ‚ остатки фосфорной кислоты ΠΊ строго ΠΎΠΏΡ€Π΅Π΄Π΅Π»Π΅Π½Π½Ρ‹ΠΌ аминокислотным остаткам ΠΌΠΎΠ»Π΅ΠΊΡƒΠ»Ρ‹ eNOS, срСди ΠΊΠΎΡ‚ΠΎΡ€Ρ‹Ρ… Π½Π°ΠΈΠ±ΠΎΠ»Π΅Π΅ Π²Π°ΠΆΠ½Ρ‹ сСрин Π² ΠΏΠΎΠ»ΠΎΠΆΠ΅Π½ΠΈΠΈ 1177 (Ser1177) ΠΈ Ρ‚Ρ€Π΅ΠΎΠ½ΠΈΠ½ Π² ΠΏΠΎΠ»ΠΎΠΆΠ΅Π½ΠΈΠΈ 495 (Thr495) . Π‘Π°ΠΉΡ‚ Ser1177 считаСтся основным мСстом Π°ΠΊΡ‚ΠΈΠ²Π°Ρ†ΠΈΠΈ eNOS. Π˜Π·Π²Π΅ΡΡ‚Π½ΠΎ, Ρ‡Ρ‚ΠΎ ΡΡ‚Π΅ΠΏΠ΅Π½ΡŒ Π΅Π³ΠΎ фосфорилирования быстро растСт ΠΏΠΎΠ΄ дСйствиСм Π²Π°ΠΆΠ½Ρ‹Ρ… рСгуляторных Ρ„Π°ΠΊΡ‚ΠΎΡ€ΠΎΠ²: напряТСния сдвига, Π±Ρ€Π°Π΄ΠΈΠΊΠΈΠ½ΠΈΠ½Π°, Ρ„Π°ΠΊΡ‚ΠΎΡ€Π° роста сосудистого эндотСлия ΠΈ эстрадиола. Основной Ρ„Π΅Ρ€ΠΌΠ΅Π½Ρ‚, ΠΎΡΡƒΡ‰Π΅ΡΡ‚Π²Π»ΡΡŽΡ‰ΠΈΠΉ Ρ‚Π°ΠΊΠΎΠΉ процСсс, - Akt (Π΄Ρ€ΡƒΠ³ΠΎΠ΅ Π½Π°Π·Π²Π°Π½ΠΈΠ΅ - ΠΏΡ€ΠΎΡ‚Π΅ΠΈΠ½ΠΊΠΈΠ½Π°Π·Π° Π’), ΠΎΠ΄Π½Π°ΠΊΠΎ извСстны ΠΈ Π΄Ρ€ΡƒΠ³ΠΈΠ΅ ΠΊΠΈΠ½Π°Π·Ρ‹, способныС Π°ΠΊΡ‚ΠΈΠ²ΠΈΡ€ΠΎΠ²Π°Ρ‚ΡŒ eNOS (ΠΎ Π½ΠΈΡ… ΠΌΡ‹ Π΅Ρ‰Π΅ расскаТСм).

ЀосфорилированиС ΠΏΠΎ сайту Thr495 ΡƒΠΌΠ΅Π½ΡŒΡˆΠ°Π΅Ρ‚ Π°ΠΊΡ‚ΠΈΠ²Π½ΠΎΡΡ‚ΡŒ Ρ„Π΅Ρ€ΠΌΠ΅Π½Ρ‚Π°. Π’Π°ΠΊΠΎΠ΅ Π½Π΅Π³Π°Ρ‚ΠΈΠ²Π½ΠΎΠ΅ влияниС ΠΌΠΎΠΆΠ΅Ρ‚ ΡƒΡΠΈΠ»ΠΈΠ²Π°Ρ‚ΡŒΡΡ ΠΏΡ€ΠΈ Π½Π΅ΠΊΠΎΡ‚ΠΎΡ€Ρ‹Ρ… патологичСских состояниях - ΠΎΠΊΠΈΡΠ»ΠΈΡ‚Π΅Π»ΡŒΠ½ΠΎΠΌ стрСссС, сахарном Π΄ΠΈΠ°Π±Π΅Ρ‚Π΅ ΠΈ Π΄Ρ€. Напротив, ΠΏΡ€ΠΈ Π½Π΅ΠΊΠΎΡ‚ΠΎΡ€Ρ‹Ρ… Π½ΠΎΡ€ΠΌΠ°Π»ΡŒΠ½Ρ‹Ρ… физиологичСских воздСйствиях фосфат удаляСтся (Ρ‚. Π΅. происходит дСфосфорилированиС Thr495), благодаря Ρ‡Π΅ΠΌΡƒ ΠΏΠΎΠ²Ρ‹ΡˆΠ°Π΅Ρ‚ΡΡ сродство eNOS ΠΊ Π‘Π° 2+ ΠΈ, ΡΠ»Π΅Π΄ΠΎΠ²Π°Ρ‚Π΅Π»ΡŒΠ½ΠΎ, увСличиваСтся Π΅Π΅ Π°ΠΊΡ‚ΠΈΠ²Π½ΠΎΡΡ‚ΡŒ. Π’Π°ΠΊΠΈΠΌ ΠΎΠ±Ρ€Π°Π·ΠΎΠΌ, ΠΈΠ½Ρ‚Π΅Π½ΡΠΈΠ²Π½ΠΎΡΡ‚ΡŒ Ρ€Π°Π±ΠΎΡ‚Ρ‹ eNOS Π² ΡΠ½Π΄ΠΎΡ‚Π΅Π»ΠΈΠ°Π»ΡŒΠ½Ρ‹Ρ… ΠΊΠ»Π΅Ρ‚ΠΊΠ°Ρ… ΠΌΠΎΠΆΠ΅Ρ‚ Π΄ΠΈΠ½Π°ΠΌΠΈΡ‡Π½ΠΎ Ρ€Π΅Π³ΡƒΠ»ΠΈΡ€ΠΎΠ²Π°Ρ‚ΡŒΡΡ ΡƒΡ€ΠΎΠ²Π½Π΅ΠΌ Π‘Π° 2+ ΠΈ фосфорилированиСм / дСфосфорилированиСм ΠΏΠΎΠ΄ дСйствиСм Ρ€Π°Π·Π½Ρ‹Ρ… ΠΏΡ€ΠΎΡ‚Π΅ΠΈΠ½ΠΊΠΈΠ½Π°Π·. Π­Ρ‚ΠΎ Π² ΠΊΠΎΠ½Π΅Ρ‡Π½ΠΎΠΌ счСтС обСспСчиваСт Ρ‚ΠΎΠ½ΠΊΡƒΡŽ Ρ€Π΅Π³ΡƒΠ»ΡΡ†ΠΈΡŽ синтСза оксида Π°Π·ΠΎΡ‚Π° ΠΈ, соотвСтствСнно, Π΅Π³ΠΎ физиологичСских эффСктов Π½Π° сСрдСчно-ΡΠΎΡΡƒΠ΄ΠΈΡΡ‚ΡƒΡŽ систСму.

ΠœΠ΅Ρ…Π°Π½ΠΈΠ·ΠΌΡ‹ расслаблСния Π³Π»Π°Π΄ΠΊΠΎΠΌΡ‹ΡˆΠ΅Ρ‡Π½Ρ‹Ρ… ΠΊΠ»Π΅Ρ‚ΠΎΠΊ

Каким ΠΆΠ΅ ΠΎΠ±Ρ€Π°Π·ΠΎΠΌ NO, сСкрСтируСмый ΡΠ½Π΄ΠΎΡ‚Π΅Π»ΠΈΠ°Π»ΡŒΠ½Ρ‹ΠΌΠΈ ΠΊΠ»Π΅Ρ‚ΠΊΠ°ΠΌΠΈ, Π²Ρ‹Π·Ρ‹Π²Π°Π΅Ρ‚ Ρ€Π°ΡΡˆΠΈΡ€Π΅Π½ΠΈΠ΅ сосудов? Π‘ΠΎΠΊΡ€Π°Ρ‰Π΅Π½ΠΈΠ΅ всСх Ρ‚ΠΈΠΏΠΎΠ² ΠΌΡ‹ΡˆΠ΅Ρ‡Π½Ρ‹Ρ… ΠΊΠ»Π΅Ρ‚ΠΎΠΊ обСспСчиваСтся взаимодСйствиСм Π΄Π²ΡƒΡ… Π±Π΅Π»ΠΊΠΎΠ² - Π°ΠΊΡ‚ΠΈΠ½Π° ΠΈ ΠΌΠΈΠΎΠ·ΠΈΠ½Π°, ΠΏΡ€ΠΈΡ‡Π΅ΠΌ моторная Π°ΠΊΡ‚ΠΈΠ²Π½ΠΎΡΡ‚ΡŒ послСднСго Π² Π³Π»Π°Π΄ΠΊΠΎΠΌΡ‹ΡˆΠ΅Ρ‡Π½Ρ‹Ρ… ΠΊΠ»Π΅Ρ‚ΠΊΠ°Ρ… проявляСтся Ρ‚ΠΎΠ»ΡŒΠΊΠΎ послС Π΅Π³ΠΎ фосфорилирования. Π­Ρ‚ΠΎ ΠΏΠΎΠ΄Ρ€Π°Π·ΡƒΠΌΠ΅Π²Π°Π΅Ρ‚ Π½Π°Π»ΠΈΡ‡ΠΈΠ΅ большого числа рСгуляторных ΠΌΠ΅Ρ…Π°Π½ΠΈΠ·ΠΌΠΎΠ², Π²Π»ΠΈΡΡŽΡ‰ΠΈΡ… Π½Π° ΡΠΎΠΊΡ€Π°Ρ‚ΠΈΡ‚Π΅Π»ΡŒΠ½ΡƒΡŽ Π°ΠΊΡ‚ΠΈΠ²Π½ΠΎΡΡ‚ΡŒ Π³Π»Π°Π΄ΠΊΠΎΠΌΡ‹ΡˆΠ΅Ρ‡Π½ΠΎΠΉ ΠΊΠ»Π΅Ρ‚ΠΊΠΈ, ΠΊ числу ΠΊΠΎΡ‚ΠΎΡ€Ρ‹Ρ… относится ΠΈ оксид Π°Π·ΠΎΡ‚Π° .

ΠœΠΎΠ»Π΅ΠΊΡƒΠ»Ρ‹ NO Π»ΠΈΠΏΠΎΡ„ΠΈΠ»ΡŒΠ½Ρ‹, поэтому ΠΎΠ½ΠΈ свободно ΠΏΡ€ΠΎΠ½ΠΈΠΊΠ°ΡŽΡ‚ ΠΈΠ· ΡΠ½Π΄ΠΎΡ‚Π΅Π»ΠΈΠ°Π»ΡŒΠ½Ρ‹Ρ… ΠΊΠ»Π΅Ρ‚ΠΎΠΊ Π² Π³Π»Π°Π΄ΠΊΠΎΠΌΡ‹ΡˆΠ΅Ρ‡Π½Ρ‹Π΅. Π’ Π½ΠΈΡ… основной Π°ΠΊΡ†Π΅ΠΏΡ‚ΠΎΡ€ NО - Ρ„Π΅Ρ€ΠΌΠ΅Π½Ρ‚ Π³ΡƒΠ°Π½ΠΈΠ»Π°Ρ‚Ρ†ΠΈΠΊΠ»Π°Π·Π°, располоТСнный Π² Ρ†ΠΈΡ‚ΠΎΠ·ΠΎΠ»Π΅ ΠΈ ΠΏΠΎΡ‚ΠΎΠΌΡƒ Π½Π°Π·Ρ‹Π²Π°Π΅ΠΌΡ‹ΠΉ растворимым (Ρ‚. Π΅. Π½Π΅ связанным с ΠΊΠ»Π΅Ρ‚ΠΎΡ‡Π½Ρ‹ΠΌΠΈ ΠΌΠ΅ΠΌΠ±Ρ€Π°Π½Π°ΠΌΠΈ). Π“ΡƒΠ°Π½ΠΈΠ»Π°Ρ‚Ρ†ΠΈΠΊΠ»Π°Π·Π°, активированная оксидом Π°Π·ΠΎΡ‚Π°, синтСзируСт цикличСский гуанозинмонофосфат (Ρ†Π“ΠœΠ€), ΠΊΠΎΡ‚ΠΎΡ€Ρ‹ΠΉ слуТит ΠΌΠΎΡ‰Π½Ρ‹ΠΌ Π°ΠΊΡ‚ΠΈΠ²Π°Ρ‚ΠΎΡ€ΠΎΠΌ Π΄Ρ€ΡƒΠ³ΠΎΠ³ΠΎ Ρ„Π΅Ρ€ΠΌΠ΅Π½Ρ‚Π°, ΠΏΡ€ΠΎΡ‚Π΅ΠΈΠ½ΠΊΠΈΠ½Π°Π·Ρ‹ G. Π•Π΅ мишСнями Π² Π³Π»Π°Π΄ΠΊΠΎΠΌΡ‹ΡˆΠ΅Ρ‡Π½Ρ‹Ρ… ΠΊΠ»Π΅Ρ‚ΠΊΠ°Ρ… слуТат многочислСнныС Π±Π΅Π»ΠΊΠΈ, ΡƒΡ‡Π°ΡΡ‚Π²ΡƒΡŽΡ‰ΠΈΠ΅ Π² рСгуляции цитоплазматичСской ΠΊΠΎΠ½Ρ†Π΅Π½Ρ‚Ρ€Π°Ρ†ΠΈΠΈ Ca 2+ .

ΠŸΡ€ΠΎΡ‚Π΅ΠΈΠ½ΠΊΠΈΠ½Π°Π·Π° G Π°ΠΊΡ‚ΠΈΠ²ΠΈΡ€ΡƒΠ΅Ρ‚ Π½Π΅ΠΊΠΎΡ‚ΠΎΡ€Ρ‹Π΅ Ρ‚ΠΈΠΏΡ‹ ΠΊΠ°Π»ΠΈΠ΅Π²Ρ‹Ρ… ΠΊΠ°Π½Π°Π»ΠΎΠ², Ρ‡Ρ‚ΠΎ Π²Ρ‹Π·Ρ‹Π²Π°Π΅Ρ‚ Π³ΠΈΠΏΠ΅Ρ€ΠΏΠΎΠ»ΡΡ€ΠΈΠ·Π°Ρ†ΠΈΡŽ (сдвиг ΠΌΠ΅ΠΌΠ±Ρ€Π°Π½Π½ΠΎΠ³ΠΎ ΠΏΠΎΡ‚Π΅Π½Ρ†ΠΈΠ°Π»Π° Π² сторону Π±ΠΎΠ»Π΅Π΅ ΠΎΡ‚Ρ€ΠΈΡ†Π°Ρ‚Π΅Π»ΡŒΠ½Ρ‹Ρ… Π·Π½Π°Ρ‡Π΅Π½ΠΈΠΉ) Π³Π»Π°Π΄ΠΊΠΎΠΌΡ‹ΡˆΠ΅Ρ‡Π½Ρ‹Ρ… ΠΊΠ»Π΅Ρ‚ΠΎΠΊ, Π·Π°ΠΊΡ€Ρ‹Π²Π°Π΅Ρ‚ управляСмыС ΠΏΠΎΡ‚Π΅Π½Ρ†ΠΈΠ°Π»ΠΎΠΌ ΠΊΠ°Π»ΡŒΡ†ΠΈΠ΅Π²Ρ‹Π΅ ΠΊΠ°Π½Π°Π»Ρ‹ Π½Π°Ρ€ΡƒΠΆΠ½ΠΎΠΉ ΠΌΠ΅ΠΌΠ±Ρ€Π°Π½Ρ‹ ΠΈ Ρ‚Π΅ΠΌ ΡƒΠΌΠ΅Π½ΡŒΡˆΠ°Π΅Ρ‚ Π²Ρ…ΠΎΠ΄ Ca 2+ Π² ΠΊΠ»Π΅Ρ‚ΠΊΡƒ. ΠšΡ€ΠΎΠΌΠ΅ Ρ‚ΠΎΠ³ΠΎ, этот Ρ„Π΅Ρ€ΠΌΠ΅Π½Ρ‚ Π² Π°ΠΊΡ‚ΠΈΠ²Π½ΠΎΠΌ состоянии подавляСт выброс Π‘Π° 2+ ΠΈΠ· Π²Π½ΡƒΡ‚Ρ€ΠΈΠΊΠ»Π΅Ρ‚ΠΎΡ‡Π½Ρ‹Ρ… Π΄Π΅ΠΏΠΎ, Π° Ρ‚Π°ΠΊΠΆΠ΅ способствуСт Π΅Π³ΠΎ ΡƒΠ΄Π°Π»Π΅Π½ΠΈΡŽ ΠΈΠ· Ρ†ΠΈΡ‚ΠΎΠΏΠ»Π°Π·ΠΌΡ‹. Π­Ρ‚ΠΎ Ρ‚ΠΎΠΆΠ΅ ΡƒΠΌΠ΅Π½ΡŒΡˆΠ°Π΅Ρ‚ ΠΊΠΎΠ½Ρ†Π΅Π½Ρ‚Ρ€Π°Ρ†ΠΈΡŽ Π‘Π° 2+ ΠΈ расслабляСт Π³Π»Π°Π΄ΠΊΠΈΠ΅ ΠΌΡ‹ΡˆΡ†Ρ‹.

Помимо влияния Π½Π° Π‘Π° 2+ -гомСостаз, ΠΏΡ€ΠΎΡ‚Π΅ΠΈΠ½ΠΊΠΈΠ½Π°Π·Π° G Ρ€Π΅Π³ΡƒΠ»ΠΈΡ€ΡƒΠ΅Ρ‚ Π‘Π° 2+ -Ρ‡ΡƒΠ²ΡΡ‚Π²ΠΈΡ‚Π΅Π»ΡŒΠ½ΠΎΡΡ‚ΡŒ ΡΠΎΠΊΡ€Π°Ρ‚ΠΈΡ‚Π΅Π»ΡŒΠ½ΠΎΠ³ΠΎ Π°ΠΏΠΏΠ°Ρ€Π°Ρ‚Π° Π³Π»Π°Π΄ΠΊΠΎΠΌΡ‹ΡˆΠ΅Ρ‡Π½Ρ‹Ρ… ΠΊΠ»Π΅Ρ‚ΠΎΠΊ, Ρ‚. Π΅. ΡƒΠΌΠ΅Π½ΡŒΡˆΠ°Π΅Ρ‚ Π΅Π³ΠΎ ΡΠΏΠΎΡΠΎΠ±Π½ΠΎΡΡ‚ΡŒ Π°ΠΊΡ‚ΠΈΠ²ΠΈΡ€ΠΎΠ²Π°Ρ‚ΡŒΡΡ ΠΏΡ€ΠΈ ΠΏΠΎΠ²Ρ‹ΡˆΠ΅Π½ΠΈΠΈ Π‘Π° 2+ . Π˜Π·Π²Π΅ΡΡ‚Π½ΠΎ, Ρ‡Ρ‚ΠΎ активация ΠΏΡ€ΠΎΡ‚Π΅ΠΈΠ½ΠΊΠΈΠ½Π°Π·Ρ‹ G (с участиСм посрСдников) сниТаСт ΡƒΡ€ΠΎΠ²Π΅Π½ΡŒ фосфорилирования Π³Π»Π°Π΄ΠΊΠΎΠΌΡ‹ΡˆΠ΅Ρ‡Π½ΠΎΠ³ΠΎ ΠΌΠΈΠΎΠ·ΠΈΠ½Π°, Π² Ρ€Π΅Π·ΡƒΠ»ΡŒΡ‚Π°Ρ‚Π΅ ΠΎΠ½ Ρ…ΡƒΠΆΠ΅ взаимодСйствуСт с Π°ΠΊΡ‚ΠΈΠ½ΠΎΠΌ, Ρ‡Ρ‚ΠΎ способствуСт Ρ€Π°ΡΡΠ»Π°Π±Π»Π΅Π½ΠΈΡŽ. Π‘ΠΎΠ²ΠΎΠΊΡƒΠΏΠ½ΠΎΡΡ‚ΡŒ описанных событий ΠΏΡ€ΠΈΠ²ΠΎΠ΄ΠΈΡ‚ ΠΊ Ρ€Π°ΡΡˆΠΈΡ€Π΅Π½ΠΈΡŽ сосудов, ΡƒΠ²Π΅Π»ΠΈΡ‡Π΅Π½ΠΈΡŽ ΠΊΡ€ΠΎΠ²ΠΎΡ‚ΠΎΠΊΠ° Π² ΠΎΡ€Π³Π°Π½Π°Ρ… ΠΈ сниТСнию уровня Π°Ρ€Ρ‚Π΅Ρ€ΠΈΠ°Π»ΡŒΠ½ΠΎΠ³ΠΎ давлСния.

ЀизиологичСская рСгуляция ΠΏΡ€ΠΎΠ΄ΡƒΠΊΡ†ΠΈΠΈ NO

Π‘ΠΏΠΎΡΠΎΠ±Π½ΠΎΡΡ‚ΡŒ ΠΊ ΠΏΡ€ΠΎΠ΄ΡƒΠΊΡ†ΠΈΠΈ NO слуТит ΠΌΠ°Ρ€ΠΊΠ΅Ρ€ΠΎΠΌ Π½ΠΎΡ€ΠΌΠ°Π»ΡŒΠ½ΠΎΠ³ΠΎ Ρ„ΡƒΠ½ΠΊΡ†ΠΈΠΎΠ½Π°Π»ΡŒΠ½ΠΎΠ³ΠΎ состояния эндотСлия: устранСниС эффСктов NO Π² Π·Π΄ΠΎΡ€ΠΎΠ²ΠΎΠΌ ΠΎΡ€Π³Π°Π½ΠΈΠ·ΠΌΠ΅ (Π½Π°ΠΏΡ€ΠΈΠΌΠ΅Ρ€, ΠΏΡƒΡ‚Π΅ΠΌ фармакологичСской Π±Π»ΠΎΠΊΠ°Π΄Ρ‹ eNOS) Π²Π΅Π΄Π΅Ρ‚ ΠΊ ΡΡƒΠΆΠ΅Π½ΠΈΡŽ сосудов ΠΈ ΠΏΠΎΠ²Ρ‹ΡˆΠ΅Π½ΠΈΡŽ уровня систСмного Π°Ρ€Ρ‚Π΅Ρ€ΠΈΠ°Π»ΡŒΠ½ΠΎΠ³ΠΎ давлСния. Π’ Ρ€Π΅Π·ΡƒΠ»ΡŒΡ‚Π°Ρ‚Π΅ дСйствия ΠΏΠΎΡ‡Ρ‚ΠΈ всСх Π½ΠΎΡ€ΠΌΠ°Π»ΡŒΠ½Ρ‹Ρ… физиологичСских стимулов содСрТаниС NO-синтазы Π² эндотСлии (ΠΈ/ΠΈΠ»ΠΈ Π΅Π΅ Π°ΠΊΡ‚ΠΈΠ²Π½ΠΎΡΡ‚ΡŒ) ΠΏΠΎΠ²Ρ‹ΡˆΠ°Π΅Ρ‚ΡΡ. ΠšΠ»ΡŽΡ‡Π΅Π²ΠΎΠΉ Ρ„Π°ΠΊΡ‚ΠΎΡ€, Ρ€Π΅Π³ΡƒΠ»ΠΈΡ€ΡƒΡŽΡ‰ΠΈΠΉ ΠΏΡ€ΠΎΠ΄ΡƒΠΊΡ†ΠΈΡŽ NO, - ΠΏΠΎΡ‚ΠΎΠΊ ΠΊΡ€ΠΎΠ²ΠΈ . ΠŸΡ€ΠΈ Π΅Π΅ Π΄Π²ΠΈΠΆΠ΅Π½ΠΈΠΈ ΠΏΠΎ сосуду Π½Π° повСрхности эндотСлия Π²ΠΎΠ·Π½ΠΈΠΊΠ°Π΅Ρ‚ напряТСниС сдвига. Π­Ρ‚ΠΎΡ‚ стимул пСрСдаСтся ΠΊ Π»ΠΎΠΊΠ°Π»ΠΈΠ·ΠΎΠ²Π°Π½Π½ΠΎΠΉ Π²Π½ΡƒΡ‚Ρ€ΠΈ ΠΊΠ»Π΅Ρ‚ΠΎΠΊ ΡΠ½Π΄ΠΎΡ‚Π΅Π»ΠΈΠ°Π»ΡŒΠ½ΠΎΠΉ NO-синтазС Ρ‡Π΅Ρ€Π΅Π· Π°ΠΊΡ‚ΠΈΠ²Π°Ρ†ΠΈΡŽ ΠΌΠ΅Ρ…Π°Π½ΠΎΡ‡ΡƒΠ²ΡΡ‚Π²ΠΈΡ‚Π΅Π»ΡŒΠ½Ρ‹Ρ… ΠΊΠ°Π½Π°Π»ΠΎΠ² ΠΈ Π²Ρ…ΠΎΠ΄ Π‘Π° 2+ . Π”Ρ€ΡƒΠ³ΠΎΠΉ Π²Π°Ρ€ΠΈΠ°Π½Ρ‚ ΠΏΠ΅Ρ€Π΅Π΄Π°Ρ‡ΠΈ - посрСдством ΠΌΠ΅ΠΌΠ±Ρ€Π°Π½Π½Ρ‹Ρ… Ρ„Π΅Ρ€ΠΌΠ΅Π½Ρ‚ΠΎΠ², Ссли ΠΏΠΎΠ²Ρ‹ΡˆΠ°Π΅Ρ‚ΡΡ Π°ΠΊΡ‚ΠΈΠ²Π½ΠΎΡΡ‚ΡŒ ΠΏΡ€ΠΎΡ‚Π΅ΠΈΠ½ΠΊΠΈΠ½Π°Π·Ρ‹ Akt ΠΈ фосфорилируСтся eNOS (ΠΏΠΎ сайту Ser1177). ΠŸΠΎΡ‚ΠΎΠΊ ΠΊΡ€ΠΎΠ²ΠΈ обСспСчиваСт ΠΏΠΎΡΡ‚ΠΎΡΠ½Π½ΡƒΡŽ ΡΠ΅ΠΊΡ€Π΅Ρ†ΠΈΡŽ эндотСлиСм Π½Π΅Π±ΠΎΠ»ΡŒΡˆΠΈΡ… количСств NO (рис. 3).

Π’Π°ΠΆΠ½ΡƒΡŽ Ρ€ΠΎΠ»ΡŒ Π² Ρ‡ΡƒΠ²ΡΡ‚Π²ΠΈΡ‚Π΅Π»ΡŒΠ½ΠΎΡΡ‚ΠΈ эндотСлия ΠΊ Π½Π°ΠΏΡ€ΡΠΆΠ΅Π½ΠΈΡŽ сдвига ΠΈΠ³Ρ€Π°Π΅Ρ‚ гликокаликс. Π­Ρ‚ΠΎ ΠΏΠΎΠΊΡ€Ρ‹Π²Π°ΡŽΡ‰ΠΈΠΉ ΠΊΠ»Π΅Ρ‚ΠΊΠΈ слой ΠΏΠΎΠ»ΠΈΠΌΠ΅Ρ€Π½Ρ‹Ρ… ΠΌΠΎΠ»Π΅ΠΊΡƒΠ» ΡƒΠ³Π»Π΅Π²ΠΎΠ΄Π½ΠΎΠΉ ΠΏΡ€ΠΈΡ€ΠΎΠ΄Ρ‹, Ρ‚ΠΎΠ»Ρ‰ΠΈΠ½Π° ΠΊΠΎΡ‚ΠΎΡ€ΠΎΠ³ΠΎ ΠΌΠΎΠΆΠ΅Ρ‚ ΡΠΎΡΡ‚Π°Π²Π»ΡΡ‚ΡŒ нСсколько ΠΌΠΈΠΊΡ€ΠΎΠΌΠ΅Ρ‚Ρ€ΠΎΠ² ΠΈ Π΄Π°ΠΆΠ΅ ΠΏΡ€Π΅Π²Ρ‹ΡˆΠ°Ρ‚ΡŒ Ρ‚ΠΎΠ»Ρ‰ΠΈΠ½Ρƒ самого эндотСлия . ΠŸΠΎΡΠΊΠΎΠ»ΡŒΠΊΡƒ «кустики» Π³Π»ΠΈΠΊΠΎΠΏΡ€ΠΎΡ‚Π΅ΠΈΠ΄ΠΎΠ² растут Π²Π½ΡƒΡ‚Ρ€ΡŒ просвСта сосуда, ΠΈΠΌΠ΅Π½Π½ΠΎ ΠΎΠ½ΠΈ Π² ΠΏΠ΅Ρ€Π²ΡƒΡŽ ΠΎΡ‡Π΅Ρ€Π΅Π΄ΡŒ ΠΈΡΠΏΡ‹Ρ‚Ρ‹Π²Π°ΡŽΡ‚ дСйствиС ΠΏΠΎΡ‚ΠΎΠΊΠ° ΠΊΡ€ΠΎΠ²ΠΈ. Π”Π΅Ρ„ΠΎΡ€ΠΌΠΈΡ€ΡƒΡΡΡŒ, Π²ΠΎΠ»ΠΎΠΊΠ½Π° гликокаликса ΠΏΠ΅Ρ€Π΅Π΄Π°ΡŽΡ‚ сигнал ΠΌΠ΅ΠΌΠ±Ρ€Π°Π½Π½Ρ‹ΠΌ Π±Π΅Π»ΠΊΠ°ΠΌ ΠΈ Π΄Π°Π»Π΅Π΅ - eNOS. Π₯отя этот ΠΌΠ΅Ρ…Π°Π½ΠΈΠ·ΠΌ ΠΏΠΎΠΊΠ° ΠΈΠ·ΡƒΡ‡Π΅Π½ ΠΌΠ°Π»ΠΎ, ΠΎ Π΅Π³ΠΎ ваТности Π³ΠΎΠ²ΠΎΡ€ΠΈΡ‚ Ρ‚ΠΎΡ‚ Ρ„Π°ΠΊΡ‚, Ρ‡Ρ‚ΠΎ Π½Π°Ρ€ΡƒΡˆΠ΅Π½ΠΈΠ΅ Ρ€Π΅Π°ΠΊΡ†ΠΈΠΈ сосудов Π½Π° напряТСниС сдвига ΠΏΡ€ΠΈ Ρ€Π°Π·Π»ΠΈΡ‡Π½Ρ‹Ρ… заболСваниях (атСросклСрозС, сахарном Π΄ΠΈΠ°Π±Π΅Ρ‚Π΅ ΠΈ Π΄Ρ€.) сопряТСно с «облысСниСм» эндотСлия, Ρ‚. Π΅. с ΡƒΠΌΠ΅Π½ΡŒΡˆΠ΅Π½ΠΈΠ΅ΠΌ Ρ‚ΠΎΠ»Ρ‰ΠΈΠ½Ρ‹ ΠΈ ΠΈΠ·ΠΌΠ΅Π½Π΅Π½ΠΈΠ΅ΠΌ структуры гликокаликса.

ΠŸΠΎΠ²Ρ‹ΡˆΠ΅Π½ΠΈΠ΅ скорости ΠΊΡ€ΠΎΠ²ΠΎΡ‚ΠΎΠΊΠ° Π²Π΅Π΄Π΅Ρ‚ ΠΊ Π°ΠΊΡ‚ΠΈΠ²Π°Ρ†ΠΈΠΈ ΡΠ½Π΄ΠΎΡ‚Π΅Π»ΠΈΠ°Π»ΡŒΠ½ΠΎΠΉ NO-синтазы ΠΈ ΠΊ Ρ€Π°ΡΡˆΠΈΡ€Π΅Π½ΠΈΡŽ сосудов, Π° ΠΏΠΎΠ΄ΠΎΠ±Π½Ρ‹Π΅ Π΄Π»ΠΈΡ‚Π΅Π»ΡŒΠ½Ρ‹Π΅ ΠΈΠ»ΠΈ ΠΌΠ½ΠΎΠ³ΠΎΠΊΡ€Π°Ρ‚Π½Ρ‹Π΅ воздСйствия ΡƒΠ²Π΅Π»ΠΈΡ‡ΠΈΠ²Π°ΡŽΡ‚ содСрТаниС этого Ρ„Π΅Ρ€ΠΌΠ΅Π½Ρ‚Π° Π² эндотСлии. На этом основано Π±Π»Π°Π³ΠΎΡ‚Π²ΠΎΡ€Π½ΠΎΠ΅ влияниС физичСских ΡƒΠΏΡ€Π°ΠΆΠ½Π΅Π½ΠΈΠΉ: извСстно, Ρ‡Ρ‚ΠΎ с ΠΏΠΎΠΌΠΎΡ‰ΡŒΡŽ Ρ‚Ρ€Π΅Π½ΠΈΡ€ΠΎΠ²ΠΊΠΈ ΠΌΠΎΠΆΠ½ΠΎ Π·Π½Π°Ρ‡ΠΈΡ‚Π΅Π»ΡŒΠ½ΠΎ ΡƒΠ»ΡƒΡ‡ΡˆΠΈΡ‚ΡŒ Ρ€Π°Π±ΠΎΡ‚Ρƒ эндотСлия Π±Π΅Π· примСнСния лСкарств! Однако слСдуСт ΠΎΡ‚ΠΌΠ΅Ρ‚ΠΈΡ‚ΡŒ, Ρ‡Ρ‚ΠΎ Ρ‚Π°ΠΊΠΎΠ΅ ΠΏΠΎΠ»Π΅Π·Π½ΠΎΠ΅ влияниС ΠΎΠΊΠ°Π·Ρ‹Π²Π°ΡŽΡ‚ Π½Π΅ Π»ΡŽΠ±Ρ‹Π΅ упраТнСния. Π’ΠΎ-ΠΏΠ΅Ρ€Π²Ρ‹Ρ…, Π½Π°Π³Ρ€ΡƒΠ·ΠΊΠ° Π΄ΠΎΠ»ΠΆΠ½Π° ΡΠΎΠΏΡ€ΠΎΠ²ΠΎΠΆΠ΄Π°Ρ‚ΡŒΡΡ ΠΏΠΎΠ²Ρ‹ΡˆΠ΅Π½ΠΈΠ΅ΠΌ скорости ΠΊΡ€ΠΎΠ²ΠΎΡ‚ΠΎΠΊΠ° Π² Ρ€Π°Π±ΠΎΡ‚Π°ΡŽΡ‰ΠΈΡ… ΠΌΡ‹ΡˆΡ†Π°Ρ…, ΠΊΠ°ΠΊ это происходит ΠΏΡ€ΠΈ быстрой Ρ…ΠΎΠ΄ΡŒΠ±Π΅, Π±Π΅Π³Π΅ ΠΈΠ»ΠΈ Π΅Π·Π΄Π΅ Π½Π° вСлосипСдС, Π° силовыС упраТнСния с гирями Ρ‚Π°ΠΊΠΎΠ³ΠΎ влияния Π½Π΅ ΠΎΠΊΠ°Π·Ρ‹Π²Π°ΡŽΡ‚. Π’ΠΎ-Π²Ρ‚ΠΎΡ€Ρ‹Ρ…, нСльзя Ρ‚Ρ€Π΅Π½ΠΈΡ€ΠΎΠ²Π°Ρ‚ΡŒΡΡ Ρ‡Π΅Ρ€Π΅Π· силу: ΠΏΡ€ΠΈ Ρ‡Ρ€Π΅Π·ΠΌΠ΅Ρ€Π½Ρ‹Ρ… Π½Π°Π³Ρ€ΡƒΠ·ΠΊΠ°Ρ… Ρ€Π΅Π·ΠΊΠΎ ΠΏΠΎΠ²Ρ‹ΡˆΠ°Π΅Ρ‚ΡΡ сСкрСция основного Π³ΠΎΡ€ΠΌΠΎΠ½Π° стрСсса, ΠΊΠΎΡ€Ρ‚ΠΈΠ·ΠΎΠ»Π°, ΠΊΠΎΡ‚ΠΎΡ€Ρ‹ΠΉ сниТаСт Π°ΠΊΡ‚ΠΈΠ²Π½ΠΎΡΡ‚ΡŒ eNOS.

Π”ΠΎΠΏΠΎΠ»Π½ΠΈΡ‚Π΅Π»ΡŒΠ½ΡƒΡŽ Π°ΠΊΡ‚ΠΈΠ²Π°Ρ†ΠΈΡŽ ΡΠ½Π΄ΠΎΡ‚Π΅Π»ΠΈΠ°Π»ΡŒΠ½ΠΎΠΉ NO-синтазы ΠΏΡ€ΠΈ физичСской Π½Π°Π³Ρ€ΡƒΠ·ΠΊΠ΅ обСспСчиваСт ΠΏΡ€ΠΎΡ‚Π΅ΠΈΠ½ΠΊΠΈΠ½Π°Π·Π°, активируСмая адСнозинмонофосфатом (АМЀ), которая содСрТится ΠΏΠΎΡ‡Ρ‚ΠΈ Π²ΠΎ всСх ΠΊΠ»Π΅Ρ‚ΠΊΠ°Ρ… нашСго ΠΎΡ€Π³Π°Π½ΠΈΠ·ΠΌΠ°, Π² Ρ‚ΠΎΠΌ числС ΠΈ Π² ΡΠ½Π΄ΠΎΡ‚Π΅Π»ΠΈΠ°Π»ΡŒΠ½Ρ‹Ρ…. Π­Ρ‚ΠΎΡ‚ Ρ„Π΅Ρ€ΠΌΠ΅Π½Ρ‚ Π½Π°Π·Ρ‹Π²Π°ΡŽΡ‚ «сСнсором энСргСтичСского статуса ΠΊΠ»Π΅Ρ‚ΠΎΠΊΒ», ΠΏΠΎΡ‚ΠΎΠΌΡƒ Ρ‡Ρ‚ΠΎ ΠΎΠ½ активируСтся, ΠΊΠΎΠ³Π΄Π° Π² Ρ†ΠΈΡ‚ΠΎΠΏΠ»Π°Π·ΠΌΠ΅ ΠΊΠ»Π΅Ρ‚ΠΊΠΈ ΠΏΠΎΠ²Ρ‹ΡˆΠ°Π΅Ρ‚ΡΡ ΠΎΡ‚Π½ΠΎΡˆΠ΅Π½ΠΈΠ΅ АМЀ / АВЀ, Ρ‚. Π΅. расход энСргии Π½Π°Ρ‡ΠΈΠ½Π°Π΅Ρ‚ ΠΏΡ€Π΅Π²Ρ‹ΡˆΠ°Ρ‚ΡŒ Π΅Π΅ ΠΎΠ±Ρ€Π°Π·ΠΎΠ²Π°Π½ΠΈΠ΅. Π’ эндотСлии Π°Ρ€Ρ‚Π΅Ρ€ΠΈΠΉ, располоТСнных Π²Π½ΡƒΡ‚Ρ€ΠΈ интСнсивно ΡΠΎΠΊΡ€Π°Ρ‰Π°ΡŽΡ‰ΠΈΡ…ΡΡ скСлСтных ΠΌΡ‹ΡˆΡ†, это ΠΌΠΎΠΆΠ΅Ρ‚ ΠΏΡ€ΠΎΠΈΡΡ…ΠΎΠ΄ΠΈΡ‚ΡŒ Π² Ρ€Π΅Π·ΡƒΠ»ΡŒΡ‚Π°Ρ‚Π΅ гипоксии - ΠΌΡ‹ΡˆΠ΅Ρ‡Π½Ρ‹Π΅ ΠΊΠ»Π΅Ρ‚ΠΊΠΈ ΠΏΠΎΡ‚Ρ€Π΅Π±Π»ΡΡŽΡ‚ ΠΌΠ½ΠΎΠ³ΠΎ О 2 , ΠΈ Π΅Π³ΠΎ Π½Π΅ Ρ…Π²Π°Ρ‚Π°Π΅Ρ‚ ΡΠ½Π΄ΠΎΡ‚Π΅Π»ΠΈΡŽ сосудов. ΠšΡ€ΠΎΠΌΠ΅ Ρ‚ΠΎΠ³ΠΎ, Π½Π΅Π΄Π°Π²Π½ΠΎ Π±Ρ‹Π»ΠΎ ΠΏΠΎΠΊΠ°Π·Π°Π½ΠΎ, Ρ‡Ρ‚ΠΎ активация этой ΠΏΡ€ΠΎΡ‚Π΅ΠΈΠ½ΠΊΠΈΠ½Π°Π·Ρ‹ Π² ΡΠ½Π΄ΠΎΡ‚Π΅Π»ΠΈΠ°Π»ΡŒΠ½Ρ‹Ρ… ΠΊΠ»Π΅Ρ‚ΠΊΠ°Ρ… Π²ΠΎΠ·ΠΌΠΎΠΆΠ½Π° ΠΏΡ€ΠΈ ΡƒΠ²Π΅Π»ΠΈΡ‡Π΅Π½ΠΈΠΈ напряТСния сдвига, Ρ‚. Π΅. ΠΏΡ€ΠΈ усилСнии ΠΏΡ€ΠΈΡ‚ΠΎΠΊΠ° ΠΊΡ€ΠΎΠ²ΠΈ ΠΊ Ρ€Π°Π±ΠΎΡ‚Π°ΡŽΡ‰ΠΈΠΌ ΠΌΡ‹ΡˆΡ†Π°ΠΌ. Активированная ΠΏΡ€ΠΎΡ‚Π΅ΠΈΠ½ΠΊΠΈΠ½Π°Π·Π° фосфорилируСт eNOS ΠΏΠΎ сайту Ser1177, продукция NO увСличиваСтся ΠΈ сосуды Ρ€Π°ΡΡˆΠΈΡ€ΡΡŽΡ‚ΡΡ.

Π’Ρ€Π°Ρ‡ΠΈ-ΠΊΠ°Ρ€Π΄ΠΈΠΎΠ»ΠΎΠ³ΠΈ Ρ…ΠΎΡ€ΠΎΡˆΠΎ Π·Π½Π°ΡŽΡ‚, Ρ‡Ρ‚ΠΎ ΠΏΡƒΡ‚Π΅ΠΌ рСгулярных физичСских Ρ‚Ρ€Π΅Π½ΠΈΡ€ΠΎΠ²ΠΎΠΊ ΠΌΠΎΠΆΠ½ΠΎ ΡƒΠ»ΡƒΡ‡ΡˆΠΈΡ‚ΡŒ Ρ„ΡƒΠ½ΠΊΡ†ΠΈΡŽ эндотСлия Π½Π΅ Ρ‚ΠΎΠ»ΡŒΠΊΠΎ Π² скСлСтных ΠΌΡ‹ΡˆΡ†Π°Ρ… ΠΈ сСрдцС, ΠΊΠΎΡ‚ΠΎΡ€Ρ‹Π΅ ΠΏΡ€ΠΈ Ρ€Π°Π±ΠΎΡ‚Π΅ интСнсивно ΡΠ½Π°Π±ΠΆΠ°ΡŽΡ‚ΡΡ ΠΊΡ€ΠΎΠ²ΡŒΡŽ, Π½ΠΎ ΠΈ Π² ΠΎΡ€Π³Π°Π½Π°Ρ…, Π½Π°ΠΏΡ€ΡΠΌΡƒΡŽ Π½Π΅ задСйствованных Π² Ρ‚Ρ€Π΅Π½ΠΈΡ€ΠΎΠ²ΠΊΠ΅, - Π² Π³ΠΎΠ»ΠΎΠ²Π½ΠΎΠΌ ΠΌΠΎΠ·Π³Π΅, ΠΊΠΎΠΆΠ΅ ΠΈ Ρ‚. Π΄. Π­Ρ‚ΠΎ Π³ΠΎΠ²ΠΎΡ€ΠΈΡ‚ ΠΎ Ρ‚ΠΎΠΌ, Ρ‡Ρ‚ΠΎ ΠΏΠΎΠΌΠΈΠΌΠΎ влияния ΠΏΠΎΡ‚ΠΎΠΊΠ° ΠΊΡ€ΠΎΠ²ΠΈ Π½Π° эндотСлий ΠΈΠΌΠ΅ΡŽΡ‚ΡΡ ΠΈ Π΄Ρ€ΡƒΠ³ΠΈΠ΅ ΠΌΠ΅Ρ…Π°Π½ΠΈΠ·ΠΌΡ‹ рСгуляции ΡΠ½Π΄ΠΎΡ‚Π΅Π»ΠΈΠ°Π»ΡŒΠ½ΠΎΠΉ NO-синтазы. Π‘Ρ€Π΅Π΄ΠΈ Π½ΠΈΡ… вСдущая Ρ€ΠΎΠ»ΡŒ ΠΏΡ€ΠΈΠ½Π°Π΄Π»Π΅ΠΆΠΈΡ‚ Π³ΠΎΡ€ΠΌΠΎΠ½Π°ΠΌ, ΠΊΠΎΡ‚ΠΎΡ€Ρ‹Π΅ ΠΏΡ€ΠΎΠ΄ΡƒΡ†ΠΈΡ€ΡƒΡŽΡ‚ΡΡ ΠΆΠ΅Π»Π΅Π·Π°ΠΌΠΈ Π²Π½ΡƒΡ‚Ρ€Π΅Π½Π½Π΅ΠΉ сСкрСции, Ρ‚Ρ€Π°Π½ΡΠΏΠΎΡ€Ρ‚ΠΈΡ€ΡƒΡŽΡ‚ΡΡ ΠΊΡ€ΠΎΠ²ΡŒΡŽ ΠΈ Ρ€Π°ΡΠΏΠΎΠ·Π½Π°ΡŽΡ‚ ΠΊΠ»Π΅Ρ‚ΠΊΠΈ-мишСни Π² Ρ€Π°Π·Π»ΠΈΡ‡Π½Ρ‹Ρ… ΠΎΡ€Π³Π°Π½Π°Ρ… ΠΏΠΎ Π½Π°Π»ΠΈΡ‡ΠΈΡŽ ΡΠΏΠ΅Ρ†ΠΈΠ°Π»ΡŒΠ½Ρ‹Ρ… Π±Π΅Π»ΠΊΠΎΠ²-Ρ€Π΅Ρ†Π΅ΠΏΡ‚ΠΎΡ€ΠΎΠ² .

Из Π³ΠΎΡ€ΠΌΠΎΠ½ΠΎΠ², ΠΊΠΎΡ‚ΠΎΡ€Ρ‹Π΅ ΠΌΠΎΠ³ΡƒΡ‚ Π²Π»ΠΈΡΡ‚ΡŒ Π½Π° Ρ„ΡƒΠ½ΠΊΡ†ΠΈΡŽ эндотСлия ΠΏΡ€ΠΈ физичСской Π½Π°Π³Ρ€ΡƒΠ·ΠΊΠ΅, ΠΎΡ‚ΠΌΠ΅Ρ‚ΠΈΠΌ Π³ΠΎΡ€ΠΌΠΎΠ½ роста (соматотропный Π³ΠΎΡ€ΠΌΠΎΠ½), ΠΊΠΎΡ‚ΠΎΡ€Ρ‹ΠΉ сСкрСтируСтся Π³ΠΈΠΏΠΎΡ„ΠΈΠ·ΠΎΠΌ. Как сам ΠΏΠΎ сСбС, Ρ‚Π°ΠΊ ΠΈ Ρ‡Π΅Ρ€Π΅Π· свои посрСдники, инсулин-ΠΏΠΎΠ΄ΠΎΠ±Π½Ρ‹Π΅ Ρ„Π°ΠΊΡ‚ΠΎΡ€Ρ‹ роста, Π³ΠΎΡ€ΠΌΠΎΠ½ роста ΡƒΠ²Π΅Π»ΠΈΡ‡ΠΈΠ²Π°Π΅Ρ‚ ΠΎΠ±Ρ€Π°Π·ΠΎΠ²Π°Π½ΠΈΠ΅ ΡΠ½Π΄ΠΎΡ‚Π΅Π»ΠΈΠ°Π»ΡŒΠ½ΠΎΠΉ NO-синтазы ΠΈ Π΅Π΅ Π°ΠΊΡ‚ΠΈΠ²Π½ΠΎΡΡ‚ΡŒ.

НаиболСС извСстный ΠΏΡ€ΠΈΠΌΠ΅Ρ€ Π³ΠΎΡ€ΠΌΠΎΠ½Π°Π»ΡŒΠ½ΠΎΠΉ рСгуляции Ρ„ΡƒΠ½ΠΊΡ†ΠΈΠΉ эндотСлия - это влияниС ТСнских ΠΏΠΎΠ»ΠΎΠ²Ρ‹Ρ… Π³ΠΎΡ€ΠΌΠΎΠ½ΠΎΠ², эстрогСнов. Π˜ΡΡ…ΠΎΠ΄Π½ΠΎ Ρ‚Π°ΠΊΠΎΠ΅ прСдставлСниС ΡΡ„ΠΎΡ€ΠΌΠΈΡ€ΠΎΠ²Π°Π»ΠΎΡΡŒ благодаря эпидСмиологичСским наблюдСниям, ΠΊΠΎΠ³Π΄Π° Π²Ρ‹ΡΡΠ½ΠΈΠ»ΠΎΡΡŒ, Ρ‡Ρ‚ΠΎ ΠΏΠΎ ΠΊΠ°ΠΊΠΎΠΉ-Ρ‚ΠΎ ΠΏΡ€ΠΈΡ‡ΠΈΠ½Π΅ ΠΆΠ΅Π½Ρ‰ΠΈΠ½Ρ‹ Π΄Π΅Ρ‚ΠΎΡ€ΠΎΠ΄Π½ΠΎΠ³ΠΎ возраста ΠΏΠΎ ΡΡ€Π°Π²Π½Π΅Π½ΠΈΡŽ с ΠΌΡƒΠΆΡ‡ΠΈΠ½Π°ΠΌΠΈ мСньшС ΡΡ‚Ρ€Π°Π΄Π°ΡŽΡ‚ ΠΎΡ‚ сосудистых Π½Π°Ρ€ΡƒΡˆΠ΅Π½ΠΈΠΉ, связанных с дисфункциСй эндотСлия. Π‘ΠΎΠ»Π΅Π΅ Ρ‚ΠΎΠ³ΠΎ, Ρƒ ΠΆΠ΅Π½Ρ‰ΠΈΠ½ Π΅Π³ΠΎ ΡΠΏΠΎΡΠΎΠ±Π½ΠΎΡΡ‚ΡŒ ΠΏΡ€ΠΎΠ΄ΡƒΡ†ΠΈΡ€ΠΎΠ²Π°Ρ‚ΡŒ NO мСняСтся Π² Ρ‚Π΅Ρ‡Π΅Π½ΠΈΠ΅ ΠΌΠ΅Π½ΡΡ‚Ρ€ΡƒΠ°Π»ΡŒΠ½ΠΎΠ³ΠΎ Ρ†ΠΈΠΊΠ»Π°, ΠΏΡ€ΠΈΡ‡Π΅ΠΌ Π² ΠΏΠ΅Ρ€Π²ΠΎΠΉ ΠΏΠΎΠ»ΠΎΠ²ΠΈΠ½Π΅, ΠΊΠΎΠ³Π΄Π° концСнтрация эстрогСнов Π² ΠΊΡ€ΠΎΠ²ΠΈ высокая, Ρ€Π°ΡΡˆΠΈΡ€Π΅Π½ΠΈΠ΅ сосудов, зависимоС ΠΎΡ‚ эндотСлия, Π±ΠΎΠ»Π΅Π΅ Π²Ρ‹Ρ€Π°ΠΆΠ΅Π½ΠΎ. Π­Ρ‚ΠΈ наблюдСния послуТили Ρ‚ΠΎΠ»Ρ‡ΠΊΠΎΠΌ ΠΊ ΠΏΡ€ΠΎΠ²Π΅Π΄Π΅Π½ΠΈΡŽ многочислСнных экспСримСнтов Π½Π° ΠΆΠΈΠ²ΠΎΡ‚Π½Ρ‹Ρ…. Π’Π°ΠΊ, ΡƒΠ΄Π°Π»Π΅Π½ΠΈΠ΅ яичников Ρƒ самок крыс ΡƒΠΌΠ΅Π½ΡŒΡˆΠ°Π»ΠΎ содСрТаниС ΠΈ Π°ΠΊΡ‚ΠΈΠ²Π½ΠΎΡΡ‚ΡŒ ΡΠ½Π΄ΠΎΡ‚Π΅Π»ΠΈΠ°Π»ΡŒΠ½ΠΎΠΉ NO-синтазы Π² артСриях Ρ€Π°Π·Π½Ρ‹Ρ… ΠΎΡ€Π³Π°Π½ΠΎΠ² (Π³ΠΎΠ»ΠΎΠ²Π½ΠΎΠ³ΠΎ ΠΌΠΎΠ·Π³Π°, сСрдца, скСлСтных ΠΌΡ‹ΡˆΡ†, ΠΏΠΎΡ‡Π΅ΠΊ, ΠΊΠΈΡˆΠ΅Ρ‡Π½ΠΈΠΊΠ° ΠΈ Π΄Ρ€.), Π° Π²Π²Π΅Π΄Π΅Π½ΠΈΠ΅ Ρ‚Π°ΠΊΠΈΠΌ самкам эстрогСнов способствовало Π½ΠΎΡ€ΠΌΠ°Π»ΠΈΠ·Π°Ρ†ΠΈΠΈ Π½Π°Ρ€ΡƒΡˆΠ΅Π½Π½ΠΎΠΉ Ρ„ΡƒΠ½ΠΊΡ†ΠΈΠΈ. ВлияниС эстрогСнов Π½Π° Π°ΠΊΡ‚ΠΈΠ²Π½ΠΎΡΡ‚ΡŒ eNOS связано с Π°ΠΊΡ‚ΠΈΠ²Π°Ρ†ΠΈΠ΅ΠΉ ΠΏΡ€ΠΎΡ‚Π΅ΠΈΠ½ΠΊΠΈΠ½Π°Π·Ρ‹ Akt, Π° ΠΏΠΎΠ²Ρ‹ΡˆΠ΅Π½ΠΈΠ΅ синтСза eNOS - с ΠΈΡ… воздСйствиСм Π½Π° Π³Π΅Π½ΠΎΠΌ ΡΠ½Π΄ΠΎΡ‚Π΅Π»ΠΈΠ°Π»ΡŒΠ½Ρ‹Ρ… ΠΊΠ»Π΅Ρ‚ΠΎΠΊ.

Π˜Π½Ρ‚Π΅Ρ€Π΅ΡΠ½ΠΎ, Ρ‡Ρ‚ΠΎ Π½Π°Ρ€ΡƒΡˆΠ΅Π½ΠΈΠ΅ Ρ€Π΅Π°ΠΊΡ†ΠΈΠΉ Π°Ρ€Ρ‚Π΅Ρ€ΠΈΠΉ ΠΌΠΎΠ·Π³Π° ΠΎΠ±Π½Π°Ρ€ΡƒΠΆΠΈΠ»ΠΈ ΠΈ Π² экспСримСнтах с ΡƒΠ΄Π°Π»Π΅Π½ΠΈΠ΅ΠΌ ΠΏΠΎΠ»ΠΎΠ²Ρ‹Ρ… ΠΆΠ΅Π»Π΅Π· Ρƒ самцов, хотя сСмСнники ΡΠ΅ΠΊΡ€Π΅Ρ‚ΠΈΡ€ΡƒΡŽΡ‚ Π½Π΅ эстрогСны, Π° Π°Π½Π΄Ρ€ΠΎΠ³Π΅Π½Ρ‹, муТскиС ΠΏΠΎΠ»ΠΎΠ²Ρ‹Π΅ Π³ΠΎΡ€ΠΌΠΎΠ½Ρ‹. Π­Ρ‚ΠΎΡ‚ парадокс стал понятным, ΠΊΠΎΠ³Π΄Π° Π² эндотСлии Π°Ρ€Ρ‚Π΅Ρ€ΠΈΠΉ ΠΌΠΎΠ·Π³Π° ΠΎΠ±Π½Π°Ρ€ΡƒΠΆΠΈΠ»ΠΈ Π°Ρ€ΠΎΠΌΠ°Ρ‚Π°Π·Ρƒ - Ρ„Π΅Ρ€ΠΌΠ΅Π½Ρ‚, ΠΏΡ€Π΅Π²Ρ€Π°Ρ‰Π°ΡŽΡ‰ΠΈΠΉ Π°Π½Π΄Ρ€ΠΎΠ³Π΅Π½Ρ‹ Π² эстрогСны. Π’Π°ΠΊΠΈΠΌ ΠΎΠ±Ρ€Π°Π·ΠΎΠΌ, Π·Π°Ρ‰ΠΈΡ‚Π½ΠΎΠ΅ влияниС эстрогСнов Π½Π° эндотСлий сосудов ΠΌΠΎΠΆΠ΅Ρ‚ ΠΈΠΌΠ΅Ρ‚ΡŒ мСсто ΠΈ Ρƒ особСй муТского ΠΏΠΎΠ»Π°. Однако Π² Π΄Π°Π½Π½ΠΎΠΌ случаС слСдуСт Π³ΠΎΠ²ΠΎΡ€ΠΈΡ‚ΡŒ ΠΎ локальной рСгуляции, которая обСспСчиваСтся эстрогСнами, ΠΎΠ±Ρ€Π°Π·ΡƒΡŽΡ‰ΠΈΠΌΠΈΡΡ нСпосрСдствСнно Π² сосудистой стСнкС.

Π’ Π·Π°ΠΊΠ»ΡŽΡ‡Π΅Π½ΠΈΠ΅ рассмотрим Ρ€Π΅Π³ΡƒΠ»ΡΡ†ΠΈΡŽ ΡΠ½Π΄ΠΎΡ‚Π΅Π»ΠΈΠ°Π»ΡŒΠ½ΠΎΠΉ NO-синтазы Π³ΠΎΡ€ΠΌΠΎΠ½Π°ΠΌΠΈ Ρ‰ΠΈΡ‚ΠΎΠ²ΠΈΠ΄Π½ΠΎΠΉ ΠΆΠ΅Π»Π΅Π·Ρ‹. Π˜Π·Π²Π΅ΡΡ‚Π½ΠΎ, Ρ‡Ρ‚ΠΎ ΠΏΡ€ΠΈ Π½Π°Ρ€ΡƒΡˆΠ΅Π½ΠΈΡΡ… Π΅Π΅ Ρ€Π°Π±ΠΎΡ‚Ρ‹ Π² эндотСлии сосудов мСняСтся ΠΈΠ½Ρ‚Π΅Π½ΡΠΈΠ²Π½ΠΎΡΡ‚ΡŒ синтСза NO: ΠΏΡ€ΠΈ Π³ΠΈΠΏΠ΅Ρ€Ρ‚ΠΈΡ€Π΅ΠΎΠ·Π΅ ΠΏΠΎΠ²Ρ‹ΡˆΠ°Π΅Ρ‚ΡΡ, Π° ΠΏΡ€ΠΈ Π³ΠΈΠΏΠΎΡ‚ΠΈΡ€Π΅ΠΎΠ·Π΅ сниТаСтся. Π’Π°ΠΊΠΎΠ΅ влияниС обусловлСно Π² основном ΠΈΠ·ΠΌΠ΅Π½Π΅Π½ΠΈΠ΅ΠΌ содСрТания NO-синтазы Π² ΠΊΠ»Π΅Ρ‚ΠΊΠ°Ρ… эндотСлия. Однако Π² послСднСС врСмя появились Π΄Π°Π½Π½Ρ‹Π΅ ΠΎ сущСствовании Π΅Ρ‰Π΅ ΠΎΠ΄Π½ΠΎΠ³ΠΎ ΠΌΠ΅Ρ…Π°Π½ΠΈΠ·ΠΌΠ° дСйствия этих Π³ΠΎΡ€ΠΌΠΎΠ½ΠΎΠ² Π½Π° ΡΠ½Π΄ΠΎΡ‚Π΅Π»ΠΈΠ°Π»ΡŒΠ½Ρ‹Π΅ ΠΊΠ»Π΅Ρ‚ΠΊΠΈ сосудов. Π’Π°ΠΊ, Ca 2+ -зависимая Π°ΠΊΡ‚ΠΈΠ²Π½ΠΎΡΡ‚ΡŒ eNOS ΠΈ ΡΡ‚Π΅ΠΏΠ΅Π½ΡŒ Π΅Π΅ фосфорилирования ΠΏΠΎ сайту Ser1177 Π² артСриях крыс c ΡΠΊΡΠΏΠ΅Ρ€ΠΈΠΌΠ΅Π½Ρ‚Π°Π»ΡŒΠ½Ρ‹ΠΌ Π³ΠΈΠΏΠ΅Ρ€Ρ‚ΠΈΡ€Π΅ΠΎΠ·ΠΎΠΌ оказалась Π·Π½Π°Ρ‡ΠΈΡ‚Π΅Π»ΡŒΠ½ΠΎ Π²Ρ‹ΡˆΠ΅, Ρ‡Π΅ΠΌ Ρƒ крыс с Π³ΠΈΠΏΠΎΡ‚ΠΈΡ€Π΅ΠΎΠ·ΠΎΠΌ.

Π˜Π·Π²Π΅ΡΡ‚Π½ΠΎ, Ρ‡Ρ‚ΠΎ Π³ΠΎΡ€ΠΌΠΎΠ½Ρ‹ Ρ‰ΠΈΡ‚ΠΎΠ²ΠΈΠ΄Π½ΠΎΠΉ ΠΆΠ΅Π»Π΅Π·Ρ‹ ΠΈΠ³Ρ€Π°ΡŽΡ‚ ΠΊΠ»ΡŽΡ‡Π΅Π²ΡƒΡŽ Ρ€ΠΎΠ»ΡŒ Π² Π΄ΠΈΡ„Ρ„Π΅Ρ€Π΅Π½Ρ†ΠΈΡ€ΠΎΠ²ΠΊΠ΅ Ρ‚ΠΊΠ°Π½Π΅ΠΉ Π² Ρ€Π°Π·Π²ΠΈΠ²Π°ΡŽΡ‰Π΅ΠΌΡΡ ΠΎΡ€Π³Π°Π½ΠΈΠ·ΠΌΠ΅. Но ΠΈΡ… влияниС Π½Π΅ сводится лишь ΠΊ ΡƒΡΠΊΠΎΡ€Π΅Π½ΠΈΡŽ ΠΈΠ»ΠΈ замСдлСнию ΠΏΡ€ΠΎΡ‚Π΅ΠΊΠ°ΡŽΡ‰ΠΈΡ… процСссов, Π° Π·Π°Ρ‡Π°ΡΡ‚ΡƒΡŽ ΠΈΠΌΠ΅Π΅Ρ‚ ΠΏΡ€ΠΎΠ³Ρ€Π°ΠΌΠΌΠΈΡ€ΡƒΡŽΡ‰ΠΈΠΉ Ρ…Π°Ρ€Π°ΠΊΡ‚Π΅Ρ€. Π­Ρ‚ΠΎ ΠΎΠ·Π½Π°Ρ‡Π°Π΅Ρ‚, Ρ‡Ρ‚ΠΎ ΠΏΡ€ΠΈ нСдостаткС Π³ΠΎΡ€ΠΌΠΎΠ½ΠΎΠ² Ρ‰ΠΈΡ‚ΠΎΠ²ΠΈΠ΄Π½ΠΎΠΉ ΠΆΠ΅Π»Π΅Π·Ρ‹ Π² ΠΎΠΏΡ€Π΅Π΄Π΅Π»Π΅Π½Π½ΠΎΠΌ критичСском возрастС ΠΊΠ»Π΅Ρ‚ΠΊΠΈ Π½Π΅ смогут ΠΏΡ€Π΅Π²Ρ€Π°Ρ‚ΠΈΡ‚ΡŒΡΡ Π² ΠΏΠΎΠ»Π½ΠΎΡ†Π΅Π½Π½ΠΎ Ρ„ΡƒΠ½ΠΊΡ†ΠΈΠΎΠ½ΠΈΡ€ΡƒΡŽΡ‰ΠΈΠ΅, Π΄Π°ΠΆΠ΅ Ссли Π²Π²ΠΎΠ΄ΠΈΡ‚ΡŒ Π³ΠΎΡ€ΠΌΠΎΠ½Ρ‹ Π½Π° ΠΏΠΎΡΠ»Π΅Π΄ΡƒΡŽΡ‰ΠΈΡ… этапах ΠΆΠΈΠ·Π½ΠΈ (Ρƒ Ρ‡Π΅Π»ΠΎΠ²Π΅ΠΊΠ° Π³ΠΎΡ€ΠΌΠΎΠ½Π°Π»ΡŒΠ½Π°Ρ тСрапия эффСктивна лишь Π² Ρ‚Π΅Ρ‡Π΅Π½ΠΈΠ΅ ΠΏΠ΅Ρ€Π²Ρ‹Ρ… мСсяцСв послС роТдСния). ΠœΠ΅Ρ…Π°Π½ΠΈΠ·ΠΌΡ‹ ΠΏΡ€ΠΎΠ³Ρ€Π°ΠΌΠΌΠΈΡ€ΡƒΡŽΡ‰Π΅Π³ΠΎ влияния Π³ΠΎΡ€ΠΌΠΎΠ½ΠΎΠ² Ρ‰ΠΈΡ‚ΠΎΠ²ΠΈΠ΄Π½ΠΎΠΉ ΠΆΠ΅Π»Π΅Π·Ρ‹ ΠΏΠΎΠ΄Ρ€ΠΎΠ±Π½ΠΎ ΠΈΠ·ΡƒΡ‡Π΅Π½Ρ‹ лишь для Π½Π΅Ρ€Π²Π½ΠΎΠΉ систСмы, Π° для ΠΎΡΡ‚Π°Π»ΡŒΠ½Ρ‹Ρ… систСм - Π·Π½Π°Ρ‡ΠΈΡ‚Π΅Π»ΡŒΠ½ΠΎ Ρ…ΡƒΠΆΠ΅. ВмСстС с Ρ‚Π΅ΠΌ Ρ…ΠΎΡ€ΠΎΡˆΠΎ извСстно, Ρ‡Ρ‚ΠΎ Π³ΠΈΠΏΠΎΡ‚ΠΈΡ€Π΅ΠΎΠ· Ρƒ ΠΌΠ°Ρ‚Π΅Ρ€ΠΈ Π²ΠΎ врСмя бСрСмСнности слуТит, ΠΏΠΎΠΌΠΈΠΌΠΎ ΠΏΡ€ΠΎΡ‡Π΅Π³ΠΎ, Ρ„Π°ΠΊΡ‚ΠΎΡ€ΠΎΠΌ риска развития Ρƒ Ρ€Π΅Π±Π΅Π½ΠΊΠ° сСрдСчно-сосудистых Π·Π°Π±ΠΎΠ»Π΅Π²Π°Π½ΠΈΠΉ. Π˜Π½Ρ‚Π΅Ρ€Π΅ΡΠ½ΠΎ, Ρ‡Ρ‚ΠΎ Π² артСриях Π΄Π΅Ρ‚Π΅Π½Ρ‹ΡˆΠ΅ΠΉ крыс Π² ΠΏΠ΅Ρ€Π²Ρ‹Π΅ Π½Π΅Π΄Π΅Π»ΠΈ послС роТдСния Π²Ρ‹ΡΠ²Π»ΡΡŽΡ‚ΡΡ ΠΏΠΎΠ²Ρ‹ΡˆΠ΅Π½Π½Ρ‹Π΅ ΡƒΡ€ΠΎΠ²Π½ΠΈ Ρ€Π΅Ρ†Π΅ΠΏΡ‚ΠΎΡ€ΠΎΠ² Ρ‚ΠΈΡ€Π΅ΠΎΠΈΠ΄Π½Ρ‹Ρ… Π³ΠΎΡ€ΠΌΠΎΠ½ΠΎΠ², Π° Ρ‚Π°ΠΊΠΆΠ΅ Ρ„Π΅Ρ€ΠΌΠ΅Π½Ρ‚Π° Π΄Π΅ΠΉΠΎΠ΄ΠΈΠ½Π°Π·Ρ‹, ΠΊΠΎΡ‚ΠΎΡ€Ρ‹ΠΉ ΠΏΡ€Π΅Π²Ρ€Π°Ρ‰Π°Π΅Ρ‚ тироксин (Ρ‚Π΅Ρ‚Ρ€Π°ΠΉΠΎΠ΄Ρ‚ΠΈΡ€ΠΎΠ½ΠΈΠ½) Π² Π±ΠΎΠ»Π΅Π΅ Π°ΠΊΡ‚ΠΈΠ²Π½Ρ‹ΠΉ Ρ‚Ρ€ΠΈΠΉΠΎΠ΄Ρ‚ΠΈΡ€ΠΎΠ½ΠΈΠ½. На основании этих наблюдСний Π·Π°ΠΌΠ°Π½Ρ‡ΠΈΠ²ΠΎ ΠΏΡ€Π΅Π΄ΠΏΠΎΠ»ΠΎΠΆΠΈΡ‚ΡŒ, Ρ‡Ρ‚ΠΎ Π³ΠΎΡ€ΠΌΠΎΠ½Ρ‹ Ρ‰ΠΈΡ‚ΠΎΠ²ΠΈΠ΄Π½ΠΎΠΉ ΠΆΠ΅Π»Π΅Π·Ρ‹ ΠΌΠΎΠ³ΡƒΡ‚ ΠΎΠΊΠ°Π·Ρ‹Π²Π°Ρ‚ΡŒ ΠΏΡ€ΠΎΠ³Ρ€Π°ΠΌΠΌΠΈΡ€ΡƒΡŽΡ‰Π΅Π΅ влияниС ΠΈ Π½Π° эндотСлий сосудов. Насколько это Π²Π΅Ρ€Π½ΠΎ, ΠΏΠΎΠΊΠ°ΠΆΡƒΡ‚ Π±ΡƒΠ΄ΡƒΡ‰ΠΈΠ΅ исслСдования.

ΠœΠ΅Ρ…Π°Π½ΠΈΠ·ΠΌΡ‹ Π½Π°Ρ€ΡƒΡˆΠ΅Π½ΠΈΡ сСкрСции NO эндотСлиСм

К соТалСнию, возмоТности эндотСлия Π½Π°ΡˆΠΈΡ… сосудов ΠΎΠ±Π΅ΡΠΏΠ΅Ρ‡ΠΈΠ²Π°Ρ‚ΡŒ ΠΏΡ€ΠΎΠ΄ΡƒΠΊΡ†ΠΈΡŽ NO Π½Π΅ Π±Π΅Π·Π³Ρ€Π°Π½ΠΈΡ‡Π½Ρ‹. ΠΠΊΡ‚ΠΈΠ²Π½ΠΎΡΡ‚ΡŒ рСгуляторных систСм ΠΎΡ€Π³Π°Π½ΠΈΠ·ΠΌΠ° высока Π² ΠΌΠΎΠ»ΠΎΠ΄ΠΎΠΌ ΠΈ Π·Ρ€Π΅Π»ΠΎΠΌ возрастС, Π½ΠΎ сниТаСтся ΠΏΡ€ΠΈ старСнии ΠΏΠΎΠ΄ влияниСм Ρ†Π΅Π»ΠΎΠ³ΠΎ ряда Ρ„Π°ΠΊΡ‚ΠΎΡ€ΠΎΠ². Π’ΠΎ-ΠΏΠ΅Ρ€Π²Ρ‹Ρ…, лишь Π½Π΅ΠΌΠ½ΠΎΠ³ΠΈΠ΅ ΠΏΠΎΠΆΠΈΠ»Ρ‹Π΅ люди ΠΌΠΎΠ³ΡƒΡ‚ ΠΏΡ€ΠΈΠΌΠ΅Ρ€ΠΈΡ‚ΡŒ ΠΊ сСбС высказываниС дрСвнСгрСчСского философа АристотСля: Β«Π–ΠΈΠ·Π½ΡŒ Ρ‚Ρ€Π΅Π±ΡƒΠ΅Ρ‚ двиТСния». Π’ΠΎ-Π²Ρ‚ΠΎΡ€Ρ‹Ρ…, с возрастом угасаСт Π΄Π΅ΡΡ‚Π΅Π»ΡŒΠ½ΠΎΡΡ‚ΡŒ ΠΌΠ½ΠΎΠ³ΠΈΡ… Π³ΠΎΡ€ΠΌΠΎΠ½Π°Π»ΡŒΠ½Ρ‹Ρ… систСм: сниТаСтся сСкрСция Π³ΠΎΡ€ΠΌΠΎΠ½Π° роста ΠΈ ΠΏΠΎΠ»ΠΎΠ²Ρ‹Ρ… Π³ΠΎΡ€ΠΌΠΎΠ½ΠΎΠ², «засыпаСт» щитовидная ΠΆΠ΅Π»Π΅Π·Π°. Π’-Ρ‚Ρ€Π΅Ρ‚ΡŒΠΈΡ…, происходят измСнСния Π² ΠΌΠ΅Ρ‚Π°Π±ΠΎΠ»ΠΈΠ·ΠΌΠ΅ всСх ΠΊΠ»Π΅Ρ‚ΠΎΠΊ. Π’ частности, энСргСтичСскиС станции ΠΊΠ»Π΅Ρ‚ΠΊΠΈ, ΠΌΠΈΡ‚ΠΎΡ…ΠΎΠ½Π΄Ρ€ΠΈΠΈ, Π½Π°Ρ‡ΠΈΠ½Π°ΡŽΡ‚ Π² большом количСствС ΠΏΡ€ΠΎΠ΄ΡƒΡ†ΠΈΡ€ΠΎΠ²Π°Ρ‚ΡŒ Π°ΠΊΡ‚ΠΈΠ²Π½Ρ‹Π΅ Ρ„ΠΎΡ€ΠΌΡ‹ кислорода, ΠΊΠΎΡ‚ΠΎΡ€Ρ‹Π΅ ΠΈΠ½Π°ΠΊΡ‚ΠΈΠ²ΠΈΡ€ΡƒΡŽΡ‚ NO, Π° Ρ‚Π°ΠΊΠΆΠ΅ ΠΏΠΎΠ΄Π°Π²Π»ΡΡŽΡ‚ Π°ΠΊΡ‚ΠΈΠ²Π½ΠΎΡΡ‚ΡŒ ΠΈ ΡΠ½ΠΈΠΆΠ°ΡŽΡ‚ содСрТаниС ΡΠ½Π΄ΠΎΡ‚Π΅Π»ΠΈΠ°Π»ΡŒΠ½ΠΎΠΉ NO-синтазы. Π’ΠΈΠ΄ΠΈΠΌΠΎ, возрастныС измСнСния эндотСлия нСльзя ΠΏΡ€Π΅Π΄ΠΎΡ‚Π²Ρ€Π°Ρ‚ΠΈΡ‚ΡŒ, Π½ΠΎ ΠΈΡ… ΠΌΠΎΠΆΠ½ΠΎ Π·Π°ΠΌΠ΅Π΄Π»ΠΈΡ‚ΡŒ, увСличивая ΠΏΠΎΠ΄Π²ΠΈΠΆΠ½ΠΎΡΡ‚ΡŒ, ΠΎΠ³Ρ€Π°Π½ΠΈΡ‡ΠΈΠ² ΠΏΠΎΡ‚Ρ€Π΅Π±Π»Π΅Π½ΠΈΠ΅ высококалорийной ΠΏΠΈΡ‰ΠΈ (это Ρ‚Π°ΠΊΠΆΠ΅ ΠΏΠΎΠ²Ρ‹ΡˆΠ°Π΅Ρ‚ Π°ΠΊΡ‚ΠΈΠ²Π½ΠΎΡΡ‚ΡŒ ΠΏΡ€ΠΎΡ‚Π΅ΠΈΠ½ΠΊΠΈΠ½Π°Π·Ρ‹, Π°ΠΊΡ‚ΠΈΠ²ΠΈΡ€ΡƒΠ΅ΠΌΠΎΠΉ АМЀ), ΠΈΡΠΏΠΎΠ»ΡŒΠ·ΡƒΡ Π·Π°ΠΌΠ΅ΡΡ‚ΠΈΡ‚Π΅Π»ΡŒΠ½ΡƒΡŽ Π³ΠΎΡ€ΠΌΠΎΠ½Π°Π»ΡŒΠ½ΡƒΡŽ Ρ‚Π΅Ρ€Π°ΠΏΠΈΡŽ (Π½Π°ΠΏΡ€ΠΈΠΌΠ΅Ρ€, Ρƒ ΠΆΠ΅Π½Ρ‰ΠΈΠ½ послС наступлСния ΠΌΠ΅Π½ΠΎΠΏΠ°ΡƒΠ·Ρ‹) ΠΈΠ»ΠΈ антиоксиданты, Ρ€Π°Π·Ρ€Π°Π±ΠΎΡ‚ΠΊΠ° ΠΊΠΎΡ‚ΠΎΡ€Ρ‹Ρ… Π±Ρ‹Π»Π° ΠΈ остаСтся ΠΏΡ€ΠΈΠΎΡ€ΠΈΡ‚Π΅Ρ‚Π½Ρ‹ΠΌ Π½Π°ΠΏΡ€Π°Π²Π»Π΅Π½ΠΈΠ΅ΠΌ Π³Π΅Ρ€ΠΎΠ½Ρ‚ΠΎΠ»ΠΎΠ³ΠΈΠΈ .

ΠŸΠΎΡ‡Π΅ΠΌΡƒ ΠΆΠ΅ синтСз NO Π² эндотСлии сосудов Π½Π°Ρ€ΡƒΡˆΠ°Π΅Ρ‚ΡΡ ΠΏΡ€ΠΈ Ρ€Π°Π·Π»ΠΈΡ‡Π½Ρ‹Ρ… патологиях? Π—Π΄Π΅ΡΡŒ Π²ΠΎΠ·ΠΌΠΎΠΆΠ½Ρ‹ Π΄Π²Π° Ρ‚ΠΈΠΏΠ° ΠΈΠ·ΠΌΠ΅Π½Π΅Π½ΠΈΠΉ: быстрыС (сниТСниС активности NO-синтазы Π² эндотСлии), ΠΈ Π΄ΠΎΠ»Π³ΠΎΠ²Ρ€Π΅ΠΌΠ΅Π½Π½Ρ‹Π΅ - ΡƒΠΌΠ΅Π½ΡŒΡˆΠ΅Π½ΠΈΠ΅ Π΅Π΅ содСрТания Π² ΠΊΠ»Π΅Ρ‚ΠΊΠ°Ρ…. ΠœΡ‹ Π½Π΅ Π±ΡƒΠ΄Π΅ΠΌ Ρ€Π°ΡΡΠΌΠ°Ρ‚Ρ€ΠΈΠ²Π°Ρ‚ΡŒ Ρ€Π°Π·Π»ΠΈΡ‡Π½Ρ‹Π΅ заболСвания Π² ΠΎΡ‚Π΄Π΅Π»ΡŒΠ½ΠΎΡΡ‚ΠΈ, Π° пСрСчислим ΠΎΠ±Ρ‰ΠΈΠ΅ для Π½ΠΈΡ… ΠΌΠ΅Ρ…Π°Π½ΠΈΠ·ΠΌΡ‹ Π·Π»ΠΎΠ²Ρ€Π΅Π΄Π½ΠΎΠ³ΠΎ влияния Π½Π° Ρ€Π°Π±ΠΎΡ‚Ρƒ eNOS. Π‘Π½ΠΈΠΆΠ΅Π½ΠΈΠ΅ активности этого Ρ„Π΅Ρ€ΠΌΠ΅Π½Ρ‚Π° ΠΏΡ€ΠΈ заболСваниях, ΠΊΠ°ΠΊ ΠΏΡ€Π°Π²ΠΈΠ»ΠΎ, связано с ΠΏΠΎΠ²Ρ‹ΡˆΠ΅Π½ΠΈΠ΅ΠΌ Π΅Π³ΠΎ фосфорилирования ΠΏΠΎ сайту Thr495, обусловлСнным ΡƒΠ²Π΅Π»ΠΈΡ‡Π΅Π½ΠΈΠ΅ΠΌ активности ΠΏΡ€ΠΎΡ‚Π΅ΠΈΠ½ΠΊΠΈΠ½Π°Π·Ρ‹ Π‘. Π•Π΅ ΠΌΠΎΡ‰Π½Ρ‹ΠΉ Π°ΠΊΡ‚ΠΈΠ²Π°Ρ‚ΠΎΡ€ - Π΄ΠΈΠ°Ρ†ΠΈΠ»Π³Π»ΠΈΡ†Π΅Ρ€ΠΎΠ». Π’ Π½ΠΎΡ€ΠΌΠ΅ ΠΎΠ½ - Π²Ρ‚ΠΎΡ€ΠΈΡ‡Π½Ρ‹ΠΉ посрСдник Π² ΠΏΠ΅Ρ€Π΅Π΄Π°Ρ‡Π΅ сигнала ΠΎΡ‚ ΠΌΠ½ΠΎΠ³ΠΈΡ… ΠΌΠ΅ΠΌΠ±Ρ€Π°Π½Π½Ρ‹Ρ… Ρ€Π΅Ρ†Π΅ΠΏΡ‚ΠΎΡ€ΠΎΠ² , Π½ΠΎ Π΅Π³ΠΎ Ρ‡Ρ€Π΅Π·ΠΌΠ΅Ρ€Π½ΠΎΠ΅ Π½Π°ΠΊΠΎΠΏΠ»Π΅Π½ΠΈΠ΅ Π² ΡΠ½Π΄ΠΎΡ‚Π΅Π»ΠΈΠ°Π»ΡŒΠ½Ρ‹Ρ… ΠΊΠ»Π΅Ρ‚ΠΊΠ°Ρ… Π²Π΅Π΄Π΅Ρ‚ ΠΊ ΠΏΠ°Ρ‚ΠΎΠ»ΠΎΠ³ΠΈΠΈ.

Π―Ρ€ΠΊΠΈΠΌ ΠΏΡ€ΠΈΠΌΠ΅Ρ€ΠΎΠΌ ΠΏΠΎΠ΄ΠΎΠ±Π½Ρ‹Ρ… ΠΈΠ·ΠΌΠ΅Π½Π΅Π½ΠΈΠΉ ΠΌΠΎΠΆΠ΅Ρ‚ Π±Ρ‹Ρ‚ΡŒ Ρ‚Π°ΠΊΠΎΠ΅ Π·Π°Π±ΠΎΠ»Π΅Π²Π°Π½ΠΈΠ΅, ΠΊΠ°ΠΊ сахарный Π΄ΠΈΠ°Π±Π΅Ρ‚, ΠΏΡ€ΠΈ ΠΊΠΎΡ‚ΠΎΡ€ΠΎΠΌ Π½Π°Ρ€ΡƒΡˆΠ΅Π½ΠΈΠ΅ синтСза ΠΈΠ»ΠΈ ΠΆΠ΅ дСйствия инсулина Π½Π° ΠΊΠ»Π΅Ρ‚ΠΊΠΈ ΠΏΡ€ΠΈΠ²ΠΎΠ΄ΠΈΡ‚ ΠΊ ΠΏΠΎΠ²Ρ‹ΡˆΠ΅Π½Π½ΠΎΠΉ ΠΊΠΎΠ½Ρ†Π΅Π½Ρ‚Ρ€Π°Ρ†ΠΈΠΈ Π³Π»ΡŽΠΊΠΎΠ·Ρ‹ Π² ΠΊΡ€ΠΎΠ²ΠΈ . ΠŸΠΎΡΠΊΠΎΠ»ΡŒΠΊΡƒ транспорт Π³Π»ΡŽΠΊΠΎΠ·Ρ‹ Π² эндотСлий Π½Π΅ рСгулируСтся инсулином (Π² ΠΎΡ‚Π»ΠΈΡ‡ΠΈΠ΅ ΠΎΡ‚ ΠΊΠ»Π΅Ρ‚ΠΎΠΊ скСлСтных ΠΌΡ‹ΡˆΡ†, сСрдца, ΠΆΠΈΡ€ΠΎΠ²ΠΎΠΉ Ρ‚ΠΊΠ°Π½ΠΈ ΠΈ Π½Π΅ΠΊΠΎΡ‚ΠΎΡ€Ρ‹Ρ… Π΄Ρ€ΡƒΠ³ΠΈΡ…), сахар Ρ‚Π°ΠΌ накапливаСтся ΠΈ становится субстратом для синтСза Π΄ΠΈΠ°Ρ†ΠΈΠ»Π³Π»ΠΈΡ†Π΅Ρ€ΠΎΠ»Π°, ΠΊΠΎΡ‚ΠΎΡ€Ρ‹ΠΉ ΠΈ Π°ΠΊΡ‚ΠΈΠ²ΠΈΡ€ΡƒΠ΅Ρ‚ ΠΏΡ€ΠΎΡ‚Π΅ΠΈΠ½ΠΊΠΈΠ½Π°Π·Ρƒ Π‘.

ΠœΠ°Ρ€ΠΊΠ΅Ρ€ΠΎΠΌ ΠΌΠ½ΠΎΠ³ΠΈΡ… сСрдСчно-сосудистых ΠΏΠ°Ρ‚ΠΎΠ»ΠΎΠ³ΠΈΠΉ слуТит ΡƒΠΆΠ΅ ΡƒΠΏΠΎΠΌΠΈΠ½Π°Π²ΡˆΠΈΠΉΡΡ ΠΎΠΊΠΈΡΠ»ΠΈΡ‚Π΅Π»ΡŒΠ½Ρ‹ΠΉ стрСсс. ΠŸΠΎΠ²Ρ‹ΡˆΠ΅Π½Π½ΠΎΠ΅ ΠΎΠ±Ρ€Π°Π·ΠΎΠ²Π°Π½ΠΈΠ΅ Π°ΠΊΡ‚ΠΈΠ²Π½Ρ‹Ρ… Ρ„ΠΎΡ€ΠΌ кислорода Ρ…Π°Ρ€Π°ΠΊΡ‚Π΅Ρ€Π½ΠΎ ΠΈ для сахарного Π΄ΠΈΠ°Π±Π΅Ρ‚Π°, ΠΈ для атСросклСроза, ΠΈ для ΠΌΠ½ΠΎΠ³ΠΈΡ… Ρ„ΠΎΡ€ΠΌ Π°Ρ€Ρ‚Π΅Ρ€ΠΈΠ°Π»ΡŒΠ½ΠΎΠΉ Π³ΠΈΠΏΠ΅Ρ€Ρ‚Π΅Π½Π·ΠΈΠΈ. ΠŸΡ€ΠΈ этих состояниях часто Π½Π°Π±Π»ΡŽΠ΄Π°Π΅Ρ‚ΡΡ высокая Π°ΠΊΡ‚ΠΈΠ²Π½ΠΎΡΡ‚ΡŒ Ρ€Π΅Π½ΠΈΠ½-Π°Π½Π³ΠΈΠΎΡ‚Π΅Π½Π·ΠΈΠ½ΠΎΠ²ΠΎΠΉ систСмы, Π° Π°Π½Π³ΠΈΠΎΡ‚Π΅Π½Π·ΠΈΠ½ II - ΠΌΠΎΡ‰Π½Ρ‹ΠΉ ΠΏΡ€ΠΎΠ²ΠΎΠΊΠ°Ρ‚ΠΎΡ€ ΠΎΠΊΠΈΡΠ»ΠΈΡ‚Π΅Π»ΡŒΠ½ΠΎΠ³ΠΎ стрСсса, ΠΊΠΎΡ‚ΠΎΡ€Ρ‹ΠΉ, с ΠΎΠ΄Π½ΠΎΠΉ стороны, сниТаСт Π°ΠΊΡ‚ΠΈΠ²Π½ΠΎΡΡ‚ΡŒ eNOS (Π½Π°ΠΏΡ€ΠΈΠΌΠ΅Ρ€, окислСнныС Π»ΠΈΠΏΠΎΠΏΡ€ΠΎΡ‚Π΅ΠΈΠ΄Ρ‹ Π½ΠΈΠ·ΠΊΠΎΠΉ плотности ΠΌΠΎΠ³ΡƒΡ‚ Π°ΠΊΡ‚ΠΈΠ²ΠΈΡ€ΠΎΠ²Π°Ρ‚ΡŒ ΠΏΡ€ΠΎΡ‚Π΅ΠΈΠ½ΠΊΠΈΠ½Π°Π·Ρƒ Π‘), Π° с Π΄Ρ€ΡƒΠ³ΠΎΠΉ, ΡƒΠΌΠ΅Π½ΡŒΡˆΠ°Π΅Ρ‚ ΡΠΊΡΠΏΡ€Π΅ΡΡΠΈΡŽ Π³Π΅Π½Π° eNOS, Ρ‡Ρ‚ΠΎ Ρ‚Π°ΠΊΠΆΠ΅ сокращаСт ΠΏΡ€ΠΎΠ΄ΡƒΠΊΡ†ΠΈΡŽ NO. ΠŸΡ€ΠΈΠΌΠ΅Π½Π΅Π½ΠΈΠ΅ антиоксидантов ΠΈΠ»ΠΈ вСщСств, ΠΏΡ€Π΅ΠΏΡΡ‚ΡΡ‚Π²ΡƒΡŽΡ‰ΠΈΡ… ΠΎΠ±Ρ€Π°Π·ΠΎΠ²Π°Π½ΠΈΡŽ ΠΈΠ»ΠΈ Π΄Π΅ΠΉΡΡ‚Π²ΠΈΡŽ Π°Π½Π³ΠΈΠΎΡ‚Π΅Π½Π·ΠΈΠ½Π° II (ΠΈΠ½Π³ΠΈΠ±ΠΈΡ‚ΠΎΡ€ΠΎΠ² Π°Π½Π³ΠΈΠΎΡ‚Π΅Π½Π·ΠΈΠ½-ΠΏΡ€Π΅Π²Ρ€Π°Ρ‰Π°ΡŽΡ‰Π΅Π³ΠΎ Ρ„Π΅Ρ€ΠΌΠ΅Π½Ρ‚Π° ΠΈΠ»ΠΈ Π±Π»ΠΎΠΊΠ°Ρ‚ΠΎΡ€ΠΎΠ² Π°Π½Π³ΠΈΠΎΡ‚Π΅Π½Π·ΠΈΠ½Π° II), ΠΏΠΎΡ‡Ρ‚ΠΈ всСгда усиливаСт ΠΎΠ±Ρ€Π°Π·ΠΎΠ²Π°Π½ΠΈΠ΅ NO. Надо ΡΠΊΠ°Π·Π°Ρ‚ΡŒ, Ρ‡Ρ‚ΠΎ ΡƒΠΌΠ΅Π½ΡŒΡˆΠ΅Π½ΠΈΠ΅ ΠΏΡ€ΠΎΠ΄ΡƒΠΊΡ†ΠΈΠΈ оксида Π°Π·ΠΎΡ‚Π° ΠΏΡ€ΠΈ заболСваниях ΠΌΠΎΠΆΠ΅Ρ‚ Π±Ρ‹Ρ‚ΡŒ связано Π½Π΅ Ρ‚ΠΎΠ»ΡŒΠΊΠΎ с прямым влияниСм Π½Π° eNOS. Π’Π°ΠΊ, дСйствиС Π³Π»ΡŽΠΊΠΎΠΊΠΎΡ€Ρ‚ΠΈΠΊΠΎΠΈΠ΄ΠΎΠ² Π½Π° эндотСлий сниТаСт содСрТаниС Π½Π΅ Ρ‚ΠΎΠ»ΡŒΠΊΠΎ самого Ρ„Π΅Ρ€ΠΌΠ΅Π½Ρ‚Π°, Π½ΠΎ ΠΈ Π΅Π³ΠΎ ΠΊΠΎΡ„Π°ΠΊΡ‚ΠΎΡ€Π°, Ρ‚Π΅Ρ‚Ρ€Π°Π³ΠΈΠ΄Ρ€ΠΎΠ±ΠΈΠΎΠΏΡ‚Π΅Ρ€ΠΈΠ½Π°.

ΠΠ°Ρ€ΡƒΡˆΠ΅Π½ΠΈΠ΅ Ρ€Π°Π±ΠΎΡ‚Ρ‹ ΡΠ½Π΄ΠΎΡ‚Π΅Π»ΠΈΠ°Π»ΡŒΠ½ΠΎΠΉ NO-синтазы ΠΌΠΎΠΆΠ΅Ρ‚ Π±Ρ‹Ρ‚ΡŒ связано с нСдостатком Π΅Π΅ основного субстрата - L-Π°Ρ€Π³ΠΈΠ½ΠΈΠ½Π°. Как ΠΏΡ€Π°Π²ΠΈΠ»ΠΎ, эта аминокислота поступаСт Π² ΠΎΡ€Π³Π°Π½ΠΈΠ·ΠΌ с ΠΏΠΈΡ‰Π΅ΠΉ Π² достаточном количСствС ΠΈ, ΠΊΡ€ΠΎΠΌΠ΅ Ρ‚ΠΎΠ³ΠΎ, ΠΌΠΎΠΆΠ΅Ρ‚ нСпосрСдствСнно ΡΠΈΠ½Ρ‚Π΅Π·ΠΈΡ€ΠΎΠ²Π°Ρ‚ΡŒΡΡ Π²ΠΎ взрослом ΠΎΡ€Π³Π°Π½ΠΈΠ·ΠΌΠ΅. Однако Π°Ρ€Π³ΠΈΠ½ΠΈΠ½ ΠΏΠΎΠΌΠΈΠΌΠΎ NO-синтаз слуТит субстратом для ΠΌΠ½ΠΎΠ³ΠΈΡ… Π΄Ρ€ΡƒΠ³ΠΈΡ… Ρ„Π΅Ρ€ΠΌΠ΅Π½Ρ‚ΠΎΠ², Π² частности Π°Ρ€Π³ΠΈΠ½Π°Π·Ρ‹, которая располагаСтся Π² Ρ€Π°Π·Π»ΠΈΡ‡Π½Ρ‹Ρ… Ρ‚ΠΈΠΏΠ°Ρ… ΠΊΠ»Π΅Ρ‚ΠΎΠΊ, Π² Ρ‚ΠΎΠΌ числС ΠΈ Π² эндотСлии сосудов. ΠŸΡ€ΠΈ сахарном Π΄ΠΈΠ°Π±Π΅Ρ‚Π΅, ΠΎΠΊΠΈΡΠ»ΠΈΡ‚Π΅Π»ΡŒΠ½ΠΎΠΌ стрСссС, Π° Ρ‚Π°ΠΊΠΆΠ΅ ΠΏΡ€ΠΈ Π²ΠΎΡΠΏΠ°Π»ΠΈΡ‚Π΅Π»ΡŒΠ½Ρ‹Ρ… процСссах ΠΏΠΎΠ΄ дСйствиСм сСкрСтируСмых ΠΊΠ»Π΅Ρ‚ΠΊΠ°ΠΌΠΈ ΠΈΠΌΠΌΡƒΠ½Π½ΠΎΠΉ систСмы Ρ†ΠΈΡ‚ΠΎΠΊΠΈΠ½ΠΎΠ² (Ρ„Π°ΠΊΡ‚ΠΎΡ€Π° Π½Π΅ΠΊΡ€ΠΎΠ·Π° ΠΎΠΏΡƒΡ…ΠΎΠ»ΠΈ ΠΈ Π΄Ρ€.) содСрТаниС Π°Ρ€Π³ΠΈΠ½Π°Π·Ρ‹ Π² эндотСлии ΠΏΠΎΠ²Ρ‹ΡˆΠ°Π΅Ρ‚ΡΡ.

НаконСц, Π² ΠΎΡ€Π³Π°Π½ΠΈΠ·ΠΌΠ΅ людСй ΠΈ Π΄Ρ€ΡƒΠ³ΠΈΡ… ΠΆΠΈΠ²ΠΎΡ‚Π½Ρ‹Ρ… ΠΌΠΎΠ³ΡƒΡ‚ ΠΏΠΎΡΠ²Π»ΡΡ‚ΡŒΡΡ ΠΈΠ½Π³ΠΈΠ±ΠΈΡ‚ΠΎΡ€Ρ‹ ΡΠ½Π΄ΠΎΡ‚Π΅Π»ΠΈΠ°Π»ΡŒΠ½ΠΎΠΉ NO-синтазы, Π½Π°ΠΏΡ€ΠΈΠΌΠ΅Ρ€ Π΄ΠΈΠΌΠ΅Ρ‚ΠΈΠ»Π°Ρ€Π³ΠΈΠ½ΠΈΠ½. Π­Ρ‚ΠΎΡ‚ Β«Π»ΠΎΠΆΠ½Ρ‹ΠΉ субстрат» ΡΠ½Π΄ΠΎΡ‚Π΅Π»ΠΈΠ°Π»ΡŒΠ½ΠΎΠΉ NO-синтазы ΠΊΠΎΠ½ΠΊΡƒΡ€ΠΈΡ€ΡƒΠ΅Ρ‚ с истинным субстратом, L-Π°Ρ€Π³ΠΈΠ½ΠΈΠ½ΠΎΠΌ, Π·Π° Π°ΠΊΡ‚ΠΈΠ²Π½Ρ‹ΠΉ Ρ†Π΅Π½Ρ‚Ρ€ Ρ„Π΅Ρ€ΠΌΠ΅Π½Ρ‚Π°. Π’ Π½ΠΎΡ€ΠΌΠ΅ Π΄ΠΈΠΌΠ΅Ρ‚ΠΈΠ»Π°Ρ€Π³ΠΈΠ½ΠΈΠ½ образуСтся Π² ΠΎΡ€Π³Π°Π½ΠΈΠ·ΠΌΠ΅ лишь Π² Π½Π΅Π±ΠΎΠ»ΡŒΡˆΠΈΡ… количСствах (Ρƒ взрослого Ρ‡Π΅Π»ΠΎΠ²Π΅ΠΊΠ° ~60 ΠΌΠ³/сут), ΠΎΠ΄Π½Π°ΠΊΠΎ ΠΏΡ€ΠΈ самых Ρ€Π°Π·Π»ΠΈΡ‡Π½Ρ‹Ρ… патологиях кровообращСния (Π°Ρ€Ρ‚Π΅Ρ€ΠΈΠ°Π»ΡŒΠ½ΠΎΠΉ Π³ΠΈΠΏΠ΅Ρ€Ρ‚Π΅Π½Π·ΠΈΠΈ, атСросклСрозС, ΠΊΠΎΡ€ΠΎΠ½Π°Ρ€Π½ΠΎΠΉ нСдостаточности ΠΈ Ρ‚. Π΄.) Π΅Π³ΠΎ продукция сущСствСнно растСт, Π° Π°ΠΊΡ‚ΠΈΠ²Π½ΠΎΡΡ‚ΡŒ ΡΠ½Π΄ΠΎΡ‚Π΅Π»ΠΈΠ°Π»ΡŒΠ½ΠΎΠΉ NO-синтазы, соотвСтствСнно, сниТаСтся.

Π˜Ρ‚Π°ΠΊ, оксид Π°Π·ΠΎΡ‚Π° - это Π²Π°ΠΆΠ½Ρ‹ΠΉ рСгуляторный Ρ„Π°ΠΊΡ‚ΠΎΡ€, посрСдством ΠΊΠΎΡ‚ΠΎΡ€ΠΎΠ³ΠΎ эндотСлий ΠΎΠΊΠ°Π·Ρ‹Π²Π°Π΅Ρ‚ Ρ€Π°ΡΡΠ»Π°Π±Π»ΡΡŽΡ‰Π΅Π΅ дСйствиС Π½Π° ΡΠΎΡΠ΅Π΄ΡΡ‚Π²ΡƒΡŽΡ‰ΠΈΠ΅ с Π½ΠΈΠΌ Π³Π»Π°Π΄ΠΊΠΎΠΌΡ‹ΡˆΠ΅Ρ‡Π½Ρ‹Π΅ ΠΊΠ»Π΅Ρ‚ΠΊΠΈ, вызывая Ρ€Π°ΡΡˆΠΈΡ€Π΅Π½ΠΈΠ΅ сосудов ΠΈ сглаТивая Π½Π΅ΠΆΠ΅Π»Π°Ρ‚Π΅Π»ΡŒΠ½Ρ‹Π΅ ΠΏΠΎΠ²Ρ‹ΡˆΠ΅Π½ΠΈΡ Π°Ρ€Ρ‚Π΅Ρ€ΠΈΠ°Π»ΡŒΠ½ΠΎΠ³ΠΎ давлСния Π½Π° систСмном ΡƒΡ€ΠΎΠ²Π½Π΅. Пока эндотСлий сохраняСт ΡΠΏΠΎΡΠΎΠ±Π½ΠΎΡΡ‚ΡŒ ΡΠ΅ΠΊΡ€Π΅Ρ‚ΠΈΡ€ΠΎΠ²Π°Ρ‚ΡŒ NO Π² достаточном для Ρ€Π΅ΡˆΠ΅Π½ΠΈΡ этих Π·Π°Π΄Π°Ρ‡ количСствС, ΠΎ состоянии сосудистой систСмы ΠΌΠΎΠΆΠ½ΠΎ Π½Π΅ Π±Π΅ΡΠΏΠΎΠΊΠΎΠΈΡ‚ΡŒΡΡ.

Π Π°Π±ΠΎΡ‚Π° Π²Ρ‹ΠΏΠΎΠ»Π½Π΅Π½Π° ΠΏΡ€ΠΈ ΠΏΠΎΠ΄Π΄Π΅Ρ€ΠΆΠΊΠ΅ Российского Ρ„ΠΎΠ½Π΄Π° Ρ„ΡƒΠ½Π΄Π°ΠΌΠ΅Π½Ρ‚Π°Π»ΡŒΠ½Ρ‹Ρ… исслСдований. ΠŸΡ€ΠΎΠ΅ΠΊΡ‚ НК 14-04-31377 ΠΌΠΎΠ»-Π°.

Π›ΠΈΡ‚Π΅Ρ€Π°Ρ‚ΡƒΡ€Π°
. Furchgott R. F., Zawadzki J. V. The obligatory role of endothelial cells in the relaxation of arterial smooth muscle by acetylcholine // Nature . 1980. V. 288. P. 373–376.
. ΠœΠ΅Π»ΡŒΠΊΡƒΠΌΡΠ½Ρ† А. М., Π‘Π°Π»Π°ΡˆΠΎΠ² Π‘. А. ΠœΠ΅Ρ…Π°Π½ΠΎΡ‡ΡƒΠ²ΡΡ‚Π²ΠΈΡ‚Π΅Π»ΡŒΠ½ΠΎΡΡ‚ΡŒ Π°Ρ€Ρ‚Π΅Ρ€ΠΈΠ°Π»ΡŒΠ½ΠΎΠ³ΠΎ эндотСлия. Π’Π²Π΅Ρ€ΡŒ, 2005.
Calcitonin Gene Related Peptide ) образуСтся ΠΈΠ· Ρ‚ΠΎΠ³ΠΎ ΠΆΠ΅ Π³Π΅Π½Π°, Ρ‡Ρ‚ΠΎ ΠΈ ΠΊΠ°Π»ΡŒΡ†ΠΈΡ‚ΠΎΠ½ΠΈΠ½, ΠΏΡ€ΠΈ Π°Π»ΡŒΡ‚Π΅Ρ€Π½Π°Ρ‚ΠΈΠ²Π½ΠΎΠΌ мРНК-сплайсингС Π² ΠΌΠΎΠ·Π³Π΅ ΠΈ пСрифСричСской Π½Π΅Ρ€Π²Π½ΠΎΠΉ систСмС.

НомСра остатков ΠΏΡ€ΠΈΠ²Π΅Π΄Π΅Π½Ρ‹ Π² соотвСтствии с располоТСниСм Π² ΠΌΠΎΠ»Π΅ΠΊΡƒΠ»Π΅ eNOS Ρ‡Π΅Π»ΠΎΠ²Π΅ΠΊΠ°.

ΠŸΡ€ΠΈ ΠΎΠ±Ρ‹Ρ‡Π½ΠΎΠΉ Ρ‚Π΅ΠΌΠΏΠ΅Ρ€Π°Ρ‚ΡƒΡ€Π΅ N 2 O - бСсцвСтный Π³Π°Π· со слабым приятным Π·Π°ΠΏΠ°Ρ…ΠΎΠΌ ΠΈ сладковатым вкусом; ΠΎΠ±Π»Π°Π΄Π°Π΅Ρ‚ наркотичСским дСйствиСм, вызывая сначала судороТный смСх, Π·Π°Ρ‚Π΅ΠΌ - ΠΏΠΎΡ‚Π΅Ρ€ΡŽ сознания.

Бпособы получСния

1. Π Π°Π·Π»ΠΎΠΆΠ΅Π½ΠΈΠ΅ Π½ΠΈΡ‚Ρ€Π°Ρ‚Π° аммония ΠΏΡ€ΠΈ нСбольшом Π½Π°Π³Ρ€Π΅Π²Π°Π½ΠΈΠΈ:


NH 4 NO 3 = N 2 O + 2Н 2 О


2. ДСйствиС HNO 3 Π½Π° Π°ΠΊΡ‚ΠΈΠ²Π½Ρ‹Π΅ ΠΌΠ΅Ρ‚Π°Π»Π»Ρ‹


10HNO 3 (ΠΊΠΎΠ½Ρ†.) + 4Π‘Π° = N 2 O + 4Ca(NO 3) 2 + 5Н 2 О

Π₯имичСскиС свойства

N 2 O Π½Π΅ проявляСт Π½ΠΈ кислотных, Π½ΠΈ основных свойств, Ρ‚. Π΅. Π½Π΅ взаимодСйствуСт с основаниями, с кислотами, с Π²ΠΎΠ΄ΠΎΠΉ (Π½Π΅ΡΠΎΠ»Π΅ΠΎΠ±Ρ€Π°Π·ΡƒΡŽΡ‰ΠΈΠΉ оксид).


ΠŸΡ€ΠΈ Π’ > 500"Π‘ разлагаСтся Π½Π° простыС вСщСства. N 2 O - ΠΎΡ‡Π΅Π½ΡŒ ΡΠΈΠ»ΡŒΠ½Ρ‹ΠΉ ΠΎΠΊΠΈΡΠ»ΠΈΡ‚Π΅Π»ΡŒ. НапримСр, способСн Π² Π²ΠΎΠ΄Π½ΠΎΠΌ растворС ΠΎΠΊΠΈΡΠ»ΠΈΡ‚ΡŒ диоксид сСры Π΄ΠΎ сСрной кислоты:


N 2 O + SO 2 + Н 2 О = N 2 + H 2 SO 4

NO - оксид Π°Π·ΠΎΡ‚Π° (II), монооксид Π°Π·ΠΎΡ‚Π°.

ΠŸΡ€ΠΈ ΠΎΠ±Ρ‹Ρ‡Π½ΠΎΠΉ Ρ‚Π΅ΠΌΠΏΠ΅Ρ€Π°Ρ‚ΡƒΡ€Π΅ NO - бСсцвСтный Π³Π°Π· Π±Π΅Π· Π·Π°ΠΏΠ°Ρ…Π°, малорастворимый Π² Π²ΠΎΠ΄Π΅, ΠΎΡ‡Π΅Π½ΡŒ токсичный (Π² Π±ΠΎΠ»ΡŒΡˆΠΈΡ… концСнтрациях измСняСт структуру Π³Π΅ΠΌΠΎΠ³Π»ΠΎΠ±ΠΈΠ½Π°).

Бпособы получСния

1. ΠŸΡ€ΡΠΌΠΎΠΉ синтСз ΠΈΠ· простых вСщСств ΠΌΠΎΠΆΠ΅Ρ‚ Π±Ρ‹Ρ‚ΡŒ осущСствлСн Ρ‚ΠΎΠ»ΡŒΠΊΠΎ ΠΏΡ€ΠΈ ΠΎΡ‡Π΅Π½ΡŒ высокой Π’:


N 2 + O 2 = 2NО - Q


2. ΠŸΠΎΠ»ΡƒΡ‡Π΅Π½ΠΈΠ΅ Π² ΠΏΡ€ΠΎΠΌΡ‹ΡˆΠ»Π΅Π½Π½ΠΎΡΡ‚ΠΈ (1-я стадия производства HNO 3).


4NH 3 + 5O 2 = 4NО + 6Н 2 О


3. Π›Π°Π±ΠΎΡ€Π°Ρ‚ΠΎΡ€Π½Ρ‹ΠΉ способ - дСйствиС Ρ€Π°Π·Π±. HNO 3 Π½Π° тяТСлыС ΠΌΠ΅Ρ‚Π°Π»Π»Ρ‹:


8HNO 3 + 3Cu = 2NO + 3Cu(NO 3) 2 + 4Н 2 О

Π₯имичСскиС свойства

NO - Π½Π΅ΡΠΎΠ»Π΅ΠΎΠ±Ρ€Π°Π·ΡƒΡŽΡ‰ΠΈΠΉ оксид (ΠΏΠΎΠ΄ΠΎΠ±Π½ΠΎ N 2 О). ΠžΠ±Π»Π°Π΄Π°Π΅Ρ‚ ΠΎΠΊΠΈΡΠ»ΠΈΡ‚Π΅Π»ΡŒΠ½ΠΎ-Π²ΠΎΡΡΡ‚Π°Π½ΠΎΠ²ΠΈΡ‚Π΅Π»ΡŒΠ½ΠΎΠΉ Π΄Π²ΠΎΠΉΡΡ‚Π²Π΅Π½Π½ΠΎΡΡ‚ΡŒΡŽ.

I. NO - ΠΎΠΊΠΈΡΠ»ΠΈΡ‚Π΅Π»ΡŒ

2NO + SO 2 + Н 2 О = N 2 O + H 2 SO 4


2NO + 2H 2 = N 2 + 2Н 2 О (со Π²Π·Ρ€Ρ‹Π²ΠΎΠΌ)

II. NO - Π²ΠΎΡΡΡ‚Π°Π½ΠΎΠ²ΠΈΡ‚Π΅Π»ΡŒ

2NO + O 2 = 2NO 2


10NO + 6KMnO 4 + 9H 2 SO 4 = 10HNO 3 + 3K 2 SO 4 + 6MnSO 4 + 4Н 2 О

NO 2 - оксид Π°Π·ΠΎΡ‚Π° (IV), диоксид Π°Π·ΠΎΡ‚Π°

ΠŸΡ€ΠΈ ΠΎΠ±Ρ‹Ρ‡Π½ΠΎΠΉ Ρ‚Π΅ΠΌΠΏΠ΅Ρ€Π°Ρ‚ΡƒΡ€Π΅ NO 2 - красно-Π±ΡƒΡ€Ρ‹ΠΉ ядовитый Π³Π°Π· с Ρ€Π΅Π·ΠΊΠΈΠΌ Π·Π°ΠΏΠ°Ρ…ΠΎΠΌ. ΠŸΡ€Π΅Π΄ΡΡ‚Π°Π²Π»ΡΠ΅Ρ‚ собой смСсь NO 2 ΠΈ Π΅Π³ΠΎ Π΄ΠΈΠΌΠ΅Ρ€Π° N 2 O 4 Π² ΡΠΎΠΎΡ‚Π½ΠΎΡˆΠ΅Π½ΠΈΠΈ -1:4. Диоксид Π°Π·ΠΎΡ‚Π° Ρ…ΠΎΡ€ΠΎΡˆΠΎ растворяСтся Π² Π²ΠΎΠ΄Π΅.

Бпособы получСния

I. ΠŸΡ€ΠΎΠΌΡ‹ΡˆΠ»Π΅Π½Π½Ρ‹ΠΉ - окислСниС NO: 2NO + O 2 = 2NO 2


II. Π›Π°Π±ΠΎΡ€Π°Ρ‚ΠΎΡ€Π½Ρ‹Π΅:


дСйствиС ΠΊΠΎΠ½Ρ†. HNO 3 Π½Π° тяТСлыС ΠΌΠ΅Ρ‚Π°Π»Π»Ρ‹: 4HNO 3 + Π‘u = 2NO 2 + Cu(NO 3) 2 + 2Н 2 О


Ρ€Π°Π·Π»ΠΎΠΆΠ΅Π½ΠΈΠ΅ Π½ΠΈΡ‚Ρ€Π°Ρ‚ΠΎΠ²: 2Pb(NO 3) 2 = 4NO 2 + O 2 + 2Π bО

Π₯имичСскиС свойства

NO 2 - кислотный оксид, ΡΠΌΠ΅ΡˆΠ°Π½Π½Ρ‹ΠΉ Π°Π½Π³ΠΈΠ΄Ρ€ΠΈΠ΄ 2-Ρ… кислот

NO 2 взаимодСйствуСт с Π²ΠΎΠ΄ΠΎΠΉ, основными оксидами ΠΈ Ρ‰Π΅Π»ΠΎΡ‡Π°ΠΌΠΈ. Но Ρ€Π΅Π°ΠΊΡ†ΠΈΠΈ ΠΏΡ€ΠΎΡ‚Π΅ΠΊΠ°ΡŽΡ‚ Π½Π΅ Ρ‚Π°ΠΊ, ΠΊΠ°ΠΊ с ΠΎΠ±Ρ‹Ρ‡Π½Ρ‹ΠΌΠΈ оксидами - ΠΎΠ½ΠΈ всСгда ΠΎΠΊΠΈΡΠ»ΠΈΡ‚Π΅Π»ΡŒΠ½ΠΎ - Π²ΠΎΡΡΡ‚Π°Π½ΠΎΠ²ΠΈΡ‚Π΅Π»ΡŒΠ½Ρ‹Π΅. ΠžΠ±ΡŠΡΡΠ½ΡΠ΅Ρ‚ΡΡ это Ρ‚Π΅ΠΌ, Ρ‡Ρ‚ΠΎ Π½Π΅ сущСствуСт кислоты со Π‘.О. (N) = +4, поэтому NO 2 ΠΏΡ€ΠΈ растворСнии Π² Π²ΠΎΠ΄Π΅ диспропорционируСт с ΠΎΠ±Ρ€Π°Π·ΠΎΠ²Π°Π½ΠΈΠ΅ΠΌ 2-Ρ… кислот - Π°Π·ΠΎΡ‚Π½ΠΎΠΉ ΠΈ азотистой:


2NO 2 + Н 2 О = HNO 3 + HNO 2


Если растворСниС происходит Π² присутствии O 2 , Ρ‚ΠΎ образуСтся ΠΎΠ΄Π½Π° кислота - азотная:


4NO 2 + 2Н 2 О + O 2 = 4HNO 3


Аналогичным ΠΎΠ±Ρ€Π°Π·ΠΎΠΌ происходит взаимодСйствиС NO 2 со Ρ‰Π΅Π»ΠΎΡ‡Π°ΠΌΠΈ:


Π² отсутствиС O 2: 2NO 2 + 2NaOH = NaNO 3 + NaNO 2 + Н 2 О


Π² присутствии O 2: 4NO 2 + 4NaOH + O 2 = 4NaNO 3 + 2Н 2 О

NO 2 - ΠΎΡ‡Π΅Π½ΡŒ ΡΠΈΠ»ΡŒΠ½Ρ‹ΠΉ ΠΎΠΊΠΈΡΠ»ΠΈΡ‚Π΅Π»ΡŒ

По ΠΎΠΊΠΈΡΠ»ΠΈΡ‚Π΅Π»ΡŒΠ½ΠΎΠΉ способности NO 2 прСвосходит Π°Π·ΠΎΡ‚Π½ΡƒΡŽ кислоту. Π’ Π΅Π³ΠΎ атмосфСрС горят Π‘, S, Π , ΠΌΠ΅Ρ‚Π°Π»Π»Ρ‹ ΠΈ Π½Π΅ΠΊΠΎΡ‚ΠΎΡ€Ρ‹Π΅ органичСскиС вСщСства. ΠŸΡ€ΠΈ этом NO 2 восстанавливаСтся Π΄ΠΎ свободного Π°Π·ΠΎΡ‚Π°:


10NO 2 + 8P = 5N 2 + 4P 2 O 5


2NO 2 + 8HI = N 2 + 4I 2 + 4Н 2 О (Π²ΠΎΠ·Π½ΠΈΠΊΠ°Π΅Ρ‚ Ρ„ΠΈΠΎΠ»Π΅Ρ‚ΠΎΠ²ΠΎΠ΅ пламя)


Π’ присутствии Pt ΠΈΠ»ΠΈ Ni диоксид Π°Π·ΠΎΡ‚Π° восстанавливаСтся Π²ΠΎΠ΄ΠΎΡ€ΠΎΠ΄ΠΎΠΌ Π΄ΠΎ Π°ΠΌΠΌΠΈΠ°ΠΊΠ°:


2NO 2 + 7Н 2 = 2NH 3 + 4Н 2 О


Как ΠΎΠΊΠΈΡΠ»ΠΈΡ‚Π΅Π»ΡŒ NO 2 ΠΈΡΠΏΠΎΠ»ΡŒΠ·ΡƒΠ΅Ρ‚ΡΡ Π² Ρ€Π°ΠΊΠ΅Ρ‚Π½Ρ‹Ρ… Ρ‚ΠΎΠΏΠ»ΠΈΠ²Π°Ρ…. ΠŸΡ€ΠΈ Π΅Π³ΠΎ взаимодСйствии с Π³ΠΈΠ΄Ρ€Π°Π·ΠΈΠ½ΠΎΠΌ ΠΈ Π΅Π³ΠΎ ΠΏΡ€ΠΎΠΈΠ·Π²ΠΎΠ΄Π½Ρ‹ΠΌΠΈ выдСляСтся большоС количСство энСргии:


2NO 2 + 2N 2 H 4 = 3N 2 + 4Н 2 О + Q

N 2 O 3 ΠΈ N 2 O 5 - нСустойчивыС вСщСства

Оба оксида ΠΈΠΌΠ΅ΡŽΡ‚ ярко Π²Ρ‹Ρ€Π°ΠΆΠ΅Π½Π½Ρ‹ΠΉ кислотный Ρ…Π°Ρ€Π°ΠΊΡ‚Π΅Ρ€, ΡΠ²Π»ΡΡŽΡ‚ΡΡ соотвСтствСнно Π°Π½Π³ΠΈΠ΄Ρ€ΠΈΠ΄Π°ΠΌΠΈ азотистой ΠΈ Π°Π·ΠΎΡ‚Π½ΠΎΠΉ кислот.


N 2 O 3 ΠΊΠ°ΠΊ ΠΈΠ½Π΄ΠΈΠ²ΠΈΠ΄ΡƒΠ°Π»ΡŒΠ½ΠΎΠ΅ вСщСство сущСствуСт Ρ‚ΠΎΠ»ΡŒΠΊΠΎ Π² Ρ‚Π²Π΅Ρ€Π΄ΠΎΠΌ состоянии Π½ΠΈΠΆΠ΅ Π’ ΠΏΠ». (-10 0 Π‘).


Π‘ ΠΏΠΎΠ²Ρ‹ΡˆΠ΅Π½ΠΈΠ΅ΠΌ Ρ‚Π΅ΠΌΠΏΠ΅Ρ€Π°Ρ‚ΡƒΡ€Ρ‹ разлагаСтся: N 2 O 3 β†’ NO + NO 2


N 2 O 5 ΠΏΡ€ΠΈ ΠΊΠΎΠΌΠ½Π°Ρ‚Π½ΠΎΠΉ Ρ‚Π΅ΠΌΠΏΠ΅Ρ€Π°Ρ‚ΡƒΡ€Π΅ ΠΈ особСнно Π½Π° свСту разлагаСтся Ρ‚Π°ΠΊ энСргично, Ρ‡Ρ‚ΠΎ ΠΈΠ½ΠΎΠ³Π΄Π° ΡΠ°ΠΌΠΎΠΏΡ€ΠΎΠΈΠ·Π²ΠΎΠ»ΡŒΠ½ΠΎ взрываСтся.


Азот ΠΎΠ±Ρ€Π°Π·ΡƒΠ΅Ρ‚ с кислородом ряд оксидов; всС ΠΎΠ½ΠΈ ΠΌΠΎΠ³ΡƒΡ‚ Π±Ρ‹Ρ‚ΡŒ ΠΏΠΎΠ»ΡƒΡ‡Π΅Π½Ρ‹ ΠΈΠ· Π°Π·ΠΎΡ‚Π½ΠΎΠΉ кислоты ΠΈΠ»ΠΈ Π΅Π΅ солСй.

Оксид Π°Π·ΠΎΡ‚Π° (I), ΠΈΠ»ΠΈ закись Π°Π·ΠΎΡ‚Π°, N 2 O получаСтся ΠΏΡ€ΠΈ Π½Π°Π³Ρ€Π΅Π²Π°Π½ΠΈΠΈ Π½ΠΈΡ‚Ρ€Π°Ρ‚Π° аммония:

Оксид Π°Π·ΠΎΡ‚Π° (1) прСдставляСт собой бСсцвСтный Π³Π°Π· со слабым Π·Π°ΠΏΠ°Ρ…ΠΎΠΌ ΠΈ сладковатым вкусом. Он малорастворим Π² Π²ΠΎΠ΄Π΅: ΠΎΠ΄ΠΈΠ½ объСм Π²ΠΎΠ΄Ρ‹ ΠΏΡ€ΠΈ 20 Β°Π‘ растворяСт 0,63 объСма N 2 O.

Оксид Π°Π·ΠΎΡ‚Π° (I) - тСрмодинамичСски нСустойчивоС соСдинСниС. Бтандартная энСргия Гиббса Π΅Π³ΠΎ образования ΠΏΠΎΠ»ΠΎΠΆΠΈΡ‚Π΅Π»ΡŒΠ½Π° (Π”Π‘Β°Π± Ρ€ =

104 ΠΊΠ”ΠΆ/моль). Однако вслСдствиС большой прочности связСй Π² ΠΌΠΎΠ»Π΅ΠΊΡƒΠ»Π΅ N 2 O энСргии Π°ΠΊΡ‚ΠΈΠ²Π°Ρ†ΠΈΠΈ Ρ€Π΅Π°ΠΊΡ†ΠΈΠΉ, ΠΏΡ€ΠΎΡ‚Π΅ΠΊΠ°ΡŽΡ‰ΠΈΡ… с участиСм этого вСщСства, высоки. Π’ частности, высока энСргия Π°ΠΊΡ‚ΠΈΠ²Π°Ρ†ΠΈΠΈ распада N 2 O. ΠŸΠΎΡΡ‚ΠΎΠΌΡƒ ΠΏΡ€ΠΈ ΠΊΠΎΠΌΠ½Π°Ρ‚Π½ΠΎΠΉ Ρ‚Π΅ΠΌΠΏΠ΅Ρ€Π°Ρ‚ΡƒΡ€Π΅ оксид Π°Π·ΠΎΡ‚Π° (I) устойчив. Однако ΠΏΡ€ΠΈ ΠΏΠΎΠ²Ρ‹ΡˆΠ΅Π½Π½Ρ‹Ρ… Ρ‚Π΅ΠΌΠΏΠ΅Ρ€Π°Ρ‚ΡƒΡ€Π°Ρ… ΠΎΠ½ разлагаСтся Π½Π° Π°Π·ΠΎΡ‚ ΠΈ кислород; Ρ€Π°Π·Π»ΠΎΠΆΠ΅Π½ΠΈΠ΅ ΠΈΠ΄Π΅Ρ‚ Ρ‚Π΅ΠΌ быстрСС, Ρ‡Π΅ΠΌ Π²Ρ‹ΡˆΠ΅ Ρ‚Π΅ΠΌΠΏΠ΅Ρ€Π°Ρ‚ΡƒΡ€Π°.

Ни с Π²ΠΎΠ΄ΠΎΠΉ, Π½ΠΈ с кислотами, Π½ΠΈ со Ρ‰Π΅Π»ΠΎΡ‡ΡŒΡŽ оксид Π°Π·ΠΎΡ‚Π° (1) Π½Π΅ Ρ€Π΅Π°Π³ΠΈΡ€ΡƒΠ΅Ρ‚.

ЭлСктронная структура ΠΌΠΎΠ»Π΅ΠΊΡƒΠ»Ρ‹ N 2 O рассмотрСна Π² Β§ 41.

Π’Π΄Ρ‹Ρ…Π°Π½ΠΈΠ΅ Π½Π΅Π±ΠΎΠ»ΡŒΡˆΠΈΡ… количСств оксида Π°Π·ΠΎΡ‚Π° (I) ΠΏΡ€ΠΈΠ²ΠΎΠ΄ΠΈΡ‚ ΠΊ ΠΏΡ€ΠΈΡ‚ΡƒΠΏΠ»Π΅Π½ΠΈΡŽ Π±ΠΎΠ»Π΅Π²ΠΎΠΉ Ρ‡ΡƒΠ²ΡΡ‚Π²ΠΈΡ‚Π΅Π»ΡŒΠ½ΠΎΡΡ‚ΠΈ, вслСдствиС Ρ‡Π΅Π³ΠΎ этот Π³Π°Π· ΠΈΠ½ΠΎΠ³Π΄Π° ΠΏΡ€ΠΈΠΌΠ΅Π½ΡΡŽΡ‚ Π² смСси с кислородом для Π½Π°Ρ€ΠΊΠΎΠ·Π°. Π‘ΠΎΠ»ΡŒΡˆΠΈΠ΅ количСства оксида Π°Π·ΠΎΡ‚Π° (I) Π΄Π΅ΠΉΡΡ‚Π²ΡƒΡŽΡ‚ Π½Π° Π½Π΅Ρ€Π²Π½ΡƒΡŽ систСму Π²ΠΎΠ·Π±ΡƒΠΆΠ΄Π°ΡŽΡ‰Π΅; поэтому Ρ€Π°Π½ΡŒΡˆΠ΅ Π΅Π³ΠΎ Π½Π°Π·Ρ‹Π²Π°Π»ΠΈ «вСсСлящим Π³Π°Π·ΠΎΠΌΒ».

Оксид Π°Π·ΠΎΡ‚Π° (II), ΠΈΠ»ΠΈ окись Π°Π·ΠΎΡ‚Π°, NO прСдставляСт собой бСсцвСтный Ρ‚Ρ€ΡƒΠ΄Π½ΠΎ сТиТаСмый Π³Π°Π·. Π–ΠΈΠ΄ΠΊΠΈΠΉ оксид Π°Π·ΠΎΡ‚Π° (II) ΠΊΠΈΠΏΠΈΡ‚ ΠΏΡ€ΠΈ -151,7Β°Π‘ ΠΈ Π·Π°Ρ‚Π²Π΅Ρ€Π΄Π΅Π²Π°Π΅Ρ‚ ΠΏΡ€ΠΈ -163,7Β°Π‘. Π’ Π²ΠΎΠ΄Π΅ ΠΎΠ½ малорастворим: 1 объСм Π²ΠΎΠ΄Ρ‹ растворяСт ΠΏΡ€ΠΈ 0Β°Π‘ всСго 0,07 объСма NO.

По химичСским свойствам оксид Π°Π·ΠΎΡ‚Π° (II) относится ΠΊ числу Π±Π΅Π·Ρ€Π°Π·Π»ΠΈΡ‡Π½Ρ‹Ρ… оксидов, Ρ‚Π°ΠΊ ΠΊΠ°ΠΊ Π½Π΅ ΠΎΠ±Ρ€Π°Π·ΡƒΠ΅Ρ‚ Π½ΠΈΠΊΠ°ΠΊΠΎΠΉ кислоты.

Подобно N 2 O, оксид Π°Π·ΠΎΡ‚Π° (II) тСрмодинамичСски нСустойчив - стандартная энСргия Гиббса Π΅Π³ΠΎ образования ΠΏΠΎΠ»ΠΎΠΆΠΈΡ‚Π΅Π»ΡŒΠ½Π° (AGo 6p = = 86,6 ΠΊΠ”ΠΆ/моль). Но, ΠΎΠΏΡΡ‚ΡŒ-Ρ‚Π°ΠΊΠΈ ΠΏΠΎΠ΄ΠΎΠ±Π½ΠΎ N 2 O, ΠΏΡ€ΠΈ ΠΊΠΎΠΌΠ½Π°Ρ‚Π½ΠΎΠΉ Ρ‚Π΅ΠΌΠΏΠ΅Ρ€Π°Ρ‚ΡƒΡ€Π΅ NO Π½Π΅ разлагаСтся, ΠΏΠΎΡ‚ΠΎΠΌΡƒ Ρ‡Ρ‚ΠΎ Π΅Π³ΠΎ ΠΌΠΎΠ»Π΅ΠΊΡƒΠ»Ρ‹ достаточно ΠΏΡ€ΠΎΡ‡Π½Ρ‹. Π›ΠΈΡˆΡŒ ΠΏΡ€ΠΈ Ρ‚Π΅ΠΌΠΏΠ΅Ρ€Π°Ρ‚ΡƒΡ€Π°Ρ… Π²Ρ‹ΡˆΠ΅ 1000 0 C Π΅Π³ΠΎ распад Π½Π° Π°Π·ΠΎΡ‚ ΠΈ кислород Π½Π°Ρ‡ΠΈΠ½Π°Π΅Ρ‚ ΠΏΡ€ΠΎΡ‚Π΅ΠΊΠ°Ρ‚ΡŒ с Π·Π°ΠΌΠ΅Ρ‚Π½ΠΎΠΉ ΡΠΊΠΎΡ€ΠΎΡΡ‚ΡŒΡŽ. ΠŸΡ€ΠΈ ΠΎΡ‡Π΅Π½ΡŒ высоких Ρ‚Π΅ΠΌΠΏΠ΅Ρ€Π°Ρ‚ΡƒΡ€Π°Ρ…, ΠΏΠΎ ΠΏΡ€ΠΈΡ‡ΠΈΠ½Π°ΠΌ, рассмотрСнным Π² Β§ 65, распад NO ΠΏΡ€ΠΎΡ…ΠΎΠ΄ΠΈΡ‚ Π½Π΅ Π΄ΠΎ ΠΊΠΎΠ½Ρ†Π° - Π² систСмС NO-N 2 -O 2 устанавливаСтся равновСсиС. Благодаря этому оксид Π°Π·ΠΎΡ‚Π° (II) ΠΌΠΎΠΆΠ½ΠΎ ΠΏΠΎΠ»ΡƒΡ‡ΠΈΡ‚ΡŒ ΠΈΠ· простых вСщСств ΠΏΡ€ΠΈ Ρ‚Π΅ΠΌΠΏΠ΅Ρ€Π°Ρ‚ΡƒΡ€Π°Ρ… элСктричСской Π΄ΡƒΠ³ΠΈ (3000-4000 Β°Π‘).

Π’ Π»Π°Π±ΠΎΡ€Π°Ρ‚ΠΎΡ€ΠΈΠΈ оксид Π°Π·ΠΎΡ‚Π° (II) ΠΎΠ±Ρ‹Ρ‡Π½ΠΎ ΠΏΠΎΠ»ΡƒΡ‡Π°ΡŽΡ‚ взаимодСйствиСм 30-35%-Π½ΠΎΠΉ Π°Π·ΠΎΡ‚Π½ΠΎΠΉ кислоты с мСдью:

Π’ ΠΏΡ€ΠΎΠΌΡ‹ΡˆΠ»Π΅Π½Π½ΠΎΡΡ‚ΠΈ ΠΎΠ½ являСтся ΠΏΡ€ΠΎΠΌΠ΅ΠΆΡƒΡ‚ΠΎΡ‡Π½Ρ‹ΠΌ ΠΏΡ€ΠΎΠ΄ΡƒΠΊΡ‚ΠΎΠΌ ΠΏΡ€ΠΈ производствС Π°Π·ΠΎΡ‚Π½ΠΎΠΉ кислоты (см. Β§ 143).

Для оксида Π°Π·ΠΎΡ‚Π° (II) Ρ…Π°Ρ€Π°ΠΊΡ‚Π΅Ρ€Π½Π° ΠΎΠΊΠΈΡΠ»ΠΈΡ‚Π΅Π»ΡŒΠ½ΠΎ-Π²ΠΎΡΡΡ‚Π°Π½ΠΎΠ²ΠΈΡ‚Π΅Π»ΡŒΠ½Π°Ρ Π΄Π²ΠΎΠΉΡΡ‚Π²Π΅Π½Π½ΠΎΡΡ‚ΡŒ. Под дСйствиСм ΡΠΈΠ»ΡŒΠ½Ρ‹Ρ… окислитСлСй ΠΎΠ½ окисляСтся, Π° Π² присутствии ΡΠΈΠ»ΡŒΠ½Ρ‹Ρ… восстановитСлСй - восстанавливаСтся. НапримСр, ΠΎΠ½ Π»Π΅Π³ΠΊΠΎ окисляСтся кислородом Π²ΠΎΠ·Π΄ΡƒΡ…Π° Π΄ΠΎ диоксида Π°Π·ΠΎΡ‚Π°:

Π’ Ρ‚ΠΎ ΠΆΠ΅ врСмя смСсь Ρ€Π°Π²Π½Ρ‹Ρ… объСмов NO ΠΈ H 2 ΠΏΡ€ΠΈ Π½Π°Π³Ρ€Π΅Π²Π°Π½ΠΈΠΈ взрываСтся:

ЭлСктронная структура ΠΌΠΎΠ»Π΅ΠΊΡƒΠ»Ρ‹ NO Π»ΡƒΡ‡ΡˆΠ΅ всСго описываСтся ΠΌΠ΅Ρ‚ΠΎΠ΄ΠΎΠΌ МО. На рис. 116 прСдставлСна схСма заполнСния MO Π² ΠΌΠΎΠ»Π΅ΠΊΡƒΠ»Π΅ NO (Π΅Ρ€. с Π°Π½Π°Π»ΠΎΠ³ΠΈΡ‡Π½Ρ‹ΠΌΠΈ схСмами для ΠΌΠΎΠ»Π΅ΠΊΡƒΠ» N 2 ΠΈ БО - см. рис. 51 ΠΈ 53). ΠœΠΎΠ»Π΅ΠΊΡƒΠ»Π° NO ΠΈΠΌΠ΅Π΅Ρ‚ Π½Π° ΠΎΠ΄ΠΈΠ½ элСктрон большС, Ρ‡Π΅ΠΌ ΠΌΠΎΠ»Π΅ΠΊΡƒΠ»Ρ‹ N 2 ΠΈ БО: этот элСктрон находится Π½Π° Ρ€Π°Π·Ρ€Ρ‹Ρ…Π»ΡΡŽΡ‰Π΅ΠΉ ΠΎΡ€Π±ΠΈΡ‚Π°Π»ΠΈ Π» Ρ€Π°Π·Ρ€ 2Ρ€. Π’Π°ΠΊΠΈΠΌ ΠΎΠ±Ρ€Π°Π·ΠΎΠΌ, число ΡΠ²ΡΠ·Ρ‹Π²Π°ΡŽΡ‰ΠΈΡ… элСктронов ΠΏΡ€Π΅Π²Ρ‹ΡˆΠ°Π΅Ρ‚ здСсь число Ρ€Π°Π·Ρ€Ρ‹Ρ…Π»ΡΡŽΡ‰ΠΈΡ… Π½Π° ΠΏΡΡ‚ΡŒ. Π­Ρ‚ΠΎ соотвСтствуСт кратности связи 2,5 (5:2 = 2,5). Π”Π΅ΠΉΡΡ‚Π²ΠΈΡ‚Π΅Π»ΡŒΠ½ΠΎ, энСргия диссоциации ΠΌΠΎΠ»Π΅ΠΊΡƒΠ»Ρ‹ NO Π½Π° Π°Ρ‚ΠΎΠΌΡ‹ (632 ΠΊΠ”ΠΆ/моль) ΠΈΠΌΠ΅Π΅Ρ‚ ΠΏΡ€ΠΎΠΌΠ΅ΠΆΡƒΡ‚ΠΎΡ‡Π½ΠΎΠ΅ Π·Π½Π°Ρ‡Π΅Π½ΠΈΠ΅ ΠΏΠΎ ΡΡ€Π°Π²Π½Π΅Π½ΠΈΡŽ с ΡΠΎΠΎΡ‚Π²Π΅Ρ‚ΡΡ‚Π²ΡƒΡŽΡ‰ΠΈΠΌΠΈ Π²Π΅Π»ΠΈΡ‡ΠΈΠ½Π°ΠΌΠΈ для ΠΌΠΎΠ»Π΅ΠΊΡƒΠ»Ρ‹ O 2 (498 ΠΊΠ”ΠΆ/моль), Π² ΠΊΠΎΡ‚ΠΎΡ€ΠΎΠΉ ΠΊΡ€Π°Ρ‚Π½ΠΎΡΡ‚ΡŒ связи Ρ€Π°Π²Π½Π° Π΄Π²ΡƒΠΌ, ΠΈ ΠΌΠΎΠ»Π΅ΠΊΡƒΠ»Ρ‹ N 2 (945 ΠΊΠ”ΠΆ/моль), Π³Π΄Π΅ связь тройная. ВмСстС с Ρ‚Π΅ΠΌ ΠΏΠΎ энСргии диссоциации ΠΌΠΎΠ»Π΅ΠΊΡƒΠ»Π° NO Π±Π»ΠΈΠ·ΠΊΠ° ΠΊ молСкулярному ΠΈΠΎΠ½Ρƒ кислорода O 2 (644 ΠΊΠ”ΠΆ/моль), Π² ΠΊΠΎΡ‚ΠΎΡ€ΠΎΠΌ ΠΊΡ€Π°Ρ‚Π½ΠΎΡΡ‚ΡŒ связи Ρ‚Π°ΠΊΠΆΠ΅ Ρ€Π°Π²Π½Π° 2,5.

ΠŸΡ€ΠΈ ΠΎΡ‚Ρ€Ρ‹Π²Π΅ ΠΎΡ‚ ΠΌΠΎΠ»Π΅ΠΊΡƒΠ»Ρ‹ NO ΠΎΠ΄Π½ΠΎΠ³ΠΎ элСктрона образуСтся ΠΈΠΎΠ½ NO + , Π½Π΅ содСрТащий Ρ€Π°Π·Ρ€Ρ‹Ρ…Π»ΡΡŽΡ‰ΠΈΡ… элСктронов; ΠΊΡ€Π°Ρ‚Π½ΠΎΡΡ‚ΡŒ связи ΠΌΠ΅ΠΆΠ΄Ρƒ Π°Ρ‚ΠΎΠΌΠ°ΠΌΠΈ возрастаСт ΠΏΡ€ΠΈ этом Π΄ΠΎ Ρ‚Ρ€Π΅Ρ… (ΡˆΠ΅ΡΡ‚ΡŒ ΡΠ²ΡΠ·Ρ‹Π²Π°ΡŽΡ‰ΠΈΡ… элСктронов). ΠŸΠΎΡΡ‚ΠΎΠΌΡƒ энСргия диссоциации ΠΈΠΎΠ½Π° NO + (1050 ΠΊΠ”ΠΆ/моль) Π²Ρ‹ΡˆΠ΅ энСргии диссоциации ΠΌΠΎΠ»Π΅ΠΊΡƒΠ»Ρ‹ NO ΠΈ Π±Π»ΠΈΠ·ΠΊΠ° ΠΊ ΡΠΎΠΎΡ‚Π²Π΅Ρ‚ΡΡ‚Π²ΡƒΡŽΡ‰Π΅ΠΉ Π²Π΅Π»ΠΈΡ‡ΠΈΠ½Π΅ для ΠΌΠΎΠ»Π΅ΠΊΡƒΠ»Ρ‹ БО (1076 ΠΊΠ”ΠΆ/моль), Π² ΠΊΠΎΡ‚ΠΎΡ€ΠΎΠΉ ΠΊΡ€Π°Ρ‚Π½ΠΎΡΡ‚ΡŒ связи Ρ€Π°Π²Π½Π° Ρ‚Ρ€Π΅ΠΌ.


Рис. 116.

Диоксид (ΠΈΠ»ΠΈ Π΄Π²ΡƒΠΎΠΊΠΈΡΡŒ) Π°Π·ΠΎΡ‚Π° NO 2 - Π±ΡƒΡ€Ρ‹ΠΉ ядовитый Π³Π°Π·, ΠΎΠ±Π»Π°Π΄Π°ΡŽΡ‰ΠΈΠΉ Ρ…Π°Ρ€Π°ΠΊΡ‚Π΅Ρ€Π½Ρ‹ΠΌ Π·Π°ΠΏΠ°Ρ…ΠΎΠΌ. Он Π»Π΅Π³ΠΊΠΎ сгущаСтся Π² ΠΊΡ€Π°ΡΠ½ΠΎΠ²Π°Ρ‚ΡƒΡŽ ΠΆΠΈΠ΄ΠΊΠΎΡΡ‚ΡŒ (Ρ‚Π΅ΠΌΠΏ. ΠΊΠΈΠΏ. 21 0 C), которая ΠΏΡ€ΠΈ ΠΎΡ…Π»Π°ΠΆΠ΄Π΅Π½ΠΈΠΈ постСпСнно свСтлССт ΠΈ ΠΏΡ€ΠΈ -11,2 Β°Π‘ Π·Π°ΠΌΠ΅Ρ€Π·Π°Π΅Ρ‚, образуя Π±Π΅ΡΡ†Π²Π΅Ρ‚Π½ΡƒΡŽ ΠΊΡ€ΠΈΡΡ‚Π°Π»Π»ΠΈΡ‡Π΅ΡΠΊΡƒΡŽ массу. ΠŸΡ€ΠΈ Π½Π°Π³Ρ€Π΅Π²Π°Π½ΠΈΠΈ Π³Π°Π·ΠΎΠΎΠ±Ρ€Π°Π·Π½ΠΎΠ³ΠΎ диоксида Π°Π·ΠΎΡ‚Π° Π΅Π³ΠΎ окраска, Π½Π°ΠΎΠ±ΠΎΡ€ΠΎΡ‚, усиливаСтся, Π° ΠΏΡ€ΠΈ 140 Β°Π‘ становится ΠΏΠΎΡ‡Ρ‚ΠΈ Ρ‡Π΅Ρ€Π½ΠΎΠΉ. ИзмСнСниС окраски диоксида Π°Π·ΠΎΡ‚Π° ΠΏΡ€ΠΈ ΠΏΠΎΠ²Ρ‹ΡˆΠ΅Π½ΠΈΠΈ Ρ‚Π΅ΠΌΠΏΠ΅Ρ€Π°Ρ‚ΡƒΡ€Ρ‹ сопровоТдаСтся ΠΈ ΠΈΠ·ΠΌΠ΅Π½Π΅Π½ΠΈΠ΅ΠΌ Π΅Π³ΠΎ молСкулярной массы. ΠŸΡ€ΠΈ Π½ΠΈΠ·ΠΊΠΎΠΉ Ρ‚Π΅ΠΌΠΏΠ΅Ρ€Π°Ρ‚ΡƒΡ€Π΅ ΠΏΠ»ΠΎΡ‚Π½ΠΎΡΡ‚ΡŒ ΠΏΠ°Ρ€Π° ΠΏΡ€ΠΈΠ±Π»ΠΈΠ·ΠΈΡ‚Π΅Π»ΡŒΠ½ΠΎ ΠΎΡ‚Π²Π΅Ρ‡Π°Π΅Ρ‚ ΡƒΠ΄Π²ΠΎΠ΅Π½Π½ΠΎΠΉ Ρ„ΠΎΡ€ΠΌΡƒΠ»Π΅ N 2 O 4 . C ΠΏΠΎΠ²Ρ‹ΡˆΠ΅Π½ΠΈΠ΅ΠΌ Ρ‚Π΅ΠΌΠΏΠ΅Ρ€Π°Ρ‚ΡƒΡ€Ρ‹ ΠΏΠ»ΠΎΡ‚Π½ΠΎΡΡ‚ΡŒ ΠΏΠ°Ρ€Π° ΡƒΠΌΠ΅Π½ΡŒΡˆΠ°Π΅Ρ‚ΡΡ ΠΈ ΠΏΡ€ΠΈ 140 Β°Π‘ соотвСтствуСт Ρ„ΠΎΡ€ΠΌΡƒΠ»Π΅ NO 2 . БСсцвСтныС кристаллы, ΡΡƒΡ‰Π΅ΡΡ‚Π²ΡƒΡŽΡ‰ΠΈΠ΅ ΠΏΡ€ΠΈ -11,2 0 C ΠΈ Π½ΠΈΠΆΠ΅, состоят ΠΈΠ· ΠΌΠΎΠ»Π΅ΠΊΡƒΠ» N 2 O 4 . По ΠΌΠ΅Ρ€Π΅ нагрСвания ΠΌΠΎΠ»Π΅ΠΊΡƒΠ»Ρ‹ N 2 O 4 Π΄ΠΈΡΡΠΎΡ†ΠΈΠΈΡ€ΡƒΡŽΡ‚ с ΠΎΠ±Ρ€Π°Π·ΠΎΠ²Π°Π½ΠΈΠ΅ΠΌ ΠΌΠΎΠ»Π΅ΠΊΡƒΠ» Ρ‚Π΅ΠΌΠ½ΠΎ-Π±ΡƒΡ€ΠΎΠ³ΠΎ диоксида Π°Π·ΠΎΡ‚Π°; полная диссоциация происходит ΠΏΡ€ΠΈ 140 0 C. Π’Π°ΠΊΠΈΠΌ ΠΎΠ±Ρ€Π°Π·ΠΎΠΌ, ΠΏΡ€ΠΈ Ρ‚Π΅ΠΌΠΏΠ΅Ρ€Π°Ρ‚ΡƒΡ€Π°Ρ… ΠΎΡ‚ -11,2 Π΄ΠΎ 140 Β°Π‘ ΠΌΠΎΠ»Π΅ΠΊΡƒΠ»Ρ‹ NO 2 ΠΈ N 2 O 4 находятся Π² равновСсии Π΄Ρ€ΡƒΠ³ с Π΄Ρ€ΡƒΠ³ΠΎΠΌ:

Π’Ρ‹ΡˆΠ΅ 140 Β°Π‘ начинаСтся диссоциация NO 2 Π½Π° NO ΠΈ кислород.

Диоксид Π°Π·ΠΎΡ‚Π° - ΠΎΡ‡Π΅Π½ΡŒ энСргичный ΠΎΠΊΠΈΡΠ»ΠΈΡ‚Π΅Π»ΡŒ. МногиС вСщСства ΠΌΠΎΠ³ΡƒΡ‚ Π³ΠΎΡ€Π΅Ρ‚ΡŒ Π² атмосфСрС NO 2 , отнимая ΠΎΡ‚ Π½Π΅Π³ΠΎ кислород. Диоксид сСры окисляСтся ΠΈΠΌ Π² триоксид, Π½Π° Ρ‡Π΅ΠΌ основан Π½ΠΈΡ‚Ρ€ΠΎΠ·Π½Ρ‹ΠΉ ΠΌΠ΅Ρ‚ΠΎΠ΄ получСния сСрной кислоты (см. Β§ 131).

ΠŸΠ°Ρ€Ρ‹ NO 2 ядовиты. Π’Π΄Ρ‹Ρ…Π°Π½ΠΈΠ΅ ΠΈΡ… Π²Ρ‹Π·Ρ‹Π²Π°Π΅Ρ‚ сильноС Ρ€Π°Π·Π΄Ρ€Π°ΠΆΠ΅Π½ΠΈΠ΅ Π΄Ρ‹Ρ…Π°Ρ‚Π΅Π»ΡŒΠ½Ρ‹Ρ… ΠΏΡƒΡ‚Π΅ΠΉ ΠΈ ΠΌΠΎΠΆΠ΅Ρ‚ привСсти ΠΊ ΡΠ΅Ρ€ΡŒΠ΅Π·Π½ΠΎΠΌΡƒ ΠΎΡ‚Ρ€Π°Π²Π»Π΅Π½ΠΈΡŽ.

ΠŸΡ€ΠΈ растворСнии Π² Π²ΠΎΠ΄Π΅ NO 2 вступаСт Π² Ρ€Π΅Π°ΠΊΡ†ΠΈΡŽ с Π²ΠΎΠ΄ΠΎΠΉ, образуя Π°Π·ΠΎΡ‚Π½ΡƒΡŽ ΠΈ Π°Π·ΠΎΡ‚ΠΈΡΡ‚ΡƒΡŽ кислоты:

Но азотистая кислота ΠΎΡ‡Π΅Π½ΡŒ нСстойка ΠΈ быстро разлагаСтся:

ΠŸΠΎΡΡ‚ΠΎΠΌΡƒ практичСски взаимодСйствиС диоксида Π°Π·ΠΎΡ‚Π° с Π²ΠΎΠ΄ΠΎΠΉ, особСнно с горячСй, ΠΈΠ΄Π΅Ρ‚ согласно ΡƒΡ€Π°Π²Π½Π΅Π½ΠΈΡŽ

ΠΊΠΎΡ‚ΠΎΡ€ΠΎΠ΅ ΠΌΠΎΠΆΠ½ΠΎ ΠΏΠΎΠ»ΡƒΡ‡ΠΈΡ‚ΡŒ слоТСниСм Π΄Π²ΡƒΡ… ΠΏΡ€Π΅Π΄Ρ‹Π΄ΡƒΡ‰ΠΈΡ… ΡƒΡ€Π°Π²Π½Π΅Π½ΠΈΠΉ, Ссли ΠΏΡ€Π΅Π΄Π²Π°Ρ€ΠΈΡ‚Π΅Π»ΡŒΠ½ΠΎ ΠΏΠ΅Ρ€Π²ΠΎΠ΅ ΠΈΠ· Π½ΠΈΡ… ΡƒΠΌΠ½ΠΎΠΆΠΈΡ‚ΡŒ Π½Π° Ρ‚Ρ€ΠΈ.

Π’ присутствии Π²ΠΎΠ·Π΄ΡƒΡ…Π° ΠΎΠ±Ρ€Π°Π·ΡƒΡŽΡ‰ΠΈΠΉΡΡ оксид Π°Π·ΠΎΡ‚Π° Π½Π΅ΠΌΠ΅Π΄Π»Π΅Π½Π½ΠΎ окисляСтся Π² диоксид Π°Π·ΠΎΡ‚Π°, Ρ‚Π°ΠΊ Ρ‡Ρ‚ΠΎ Π² этом случаС NO 2 Π² ΠΊΠΎΠ½Π΅Ρ‡Π½ΠΎΠΌ ΠΈΡ‚ΠΎΠ³Π΅ ΠΏΠΎΠ»Π½ΠΎΡΡ‚ΡŒΡŽ ΠΏΠ΅Ρ€Π΅Ρ…ΠΎΠ΄ΠΈΡ‚ Π² Π°Π·ΠΎΡ‚Π½ΡƒΡŽ кислоту:

Π­Ρ‚Π° рСакция ΠΈΡΠΏΠΎΠ»ΡŒΠ·ΡƒΠ΅Ρ‚ΡΡ Π² соврСмСнных способах получСния Π°Π·ΠΎΡ‚Π½ΠΎΠΉ кислоты.

Если Ρ€Π°ΡΡ‚Π²ΠΎΡ€ΡΡ‚ΡŒ диоксид Π°Π·ΠΎΡ‚Π° Π² Ρ‰Π΅Π»ΠΎΡ‡Π°Ρ…, Ρ‚ΠΎ образуСтся смСсь солСй Π°Π·ΠΎΡ‚Π½ΠΎΠΉ ΠΈ азотистой кислот, Π½Π°ΠΏΡ€ΠΈΠΌΠ΅Ρ€:

Оксид Π°Π·ΠΎΡ‚Π° (III), ΠΈΠ»ΠΈ азотистый Π°Π½Π³ΠΈΠ΄Ρ€ΠΈΠ΄, N 2 O 3 прСдставляСт собой Ρ‚Π΅ΠΌΠ½ΠΎ-синюю ΠΆΠΈΠ΄ΠΊΠΎΡΡ‚ΡŒ, ΡƒΠΆΠ΅ ΠΏΡ€ΠΈ Π½ΠΈΠ·ΠΊΠΈΡ… Ρ‚Π΅ΠΌΠΏΠ΅Ρ€Π°Ρ‚ΡƒΡ€Π°Ρ… Ρ€Π°Π·Π»Π°Π³Π°ΡŽΡ‰ΡƒΡŽΡΡ Π½Π° NO ΠΈ NO 2 . БмСсь Ρ€Π°Π²Π½Ρ‹Ρ… объСмов NO ΠΈ NO 2 ΠΏΡ€ΠΈ ΠΎΡ…Π»Π°ΠΆΠ΄Π΅Π½ΠΈΠΈ вновь ΠΎΠ±Ρ€Π°Π·ΡƒΠ΅Ρ‚ N 2 O 3:

ΠžΠΊΡΠΈΠ΄Ρƒ Π°Π·ΠΎΡ‚Π° (III) соотвСтствуСт азотистая кислота HNO 2 .

Оксид Π°Π·ΠΎΡ‚Π° (V), ΠΈΠ»ΠΈ Π°Π·ΠΎΡ‚Π½Ρ‹ΠΉ Π°Π½Π³ΠΈΠ΄Ρ€ΠΈΠ΄, N 2 O 5 - Π±Π΅Π»Ρ‹Π΅ кристаллы, ΡƒΠΆΠ΅ ΠΏΡ€ΠΈ ΠΊΠΎΠΌΠ½Π°Ρ‚Π½ΠΎΠΉ Ρ‚Π΅ΠΌΠΏΠ΅Ρ€Π°Ρ‚ΡƒΡ€Π΅ постСпСнно Ρ€Π°Π·Π»Π°Π³Π°ΡŽΡ‰ΠΈΠ΅ΡΡ Π½Π° NO 2 ΠΈ O 2 . Он ΠΌΠΎΠΆΠ΅Ρ‚ Π±Ρ‹Ρ‚ΡŒ ΠΏΠΎΠ»ΡƒΡ‡Π΅Π½ дСйствиСм фосфорного Π°Π½Π³ΠΈΠ΄Ρ€ΠΈΠ΄Π° Π½Π° Π°Π·ΠΎΡ‚Π½ΡƒΡŽ кислоту:

Оксид Π°Π·ΠΎΡ‚Π° (V) - ΠΎΡ‡Π΅Π½ΡŒ ΡΠΈΠ»ΡŒΠ½Ρ‹ΠΉ ΠΎΠΊΠΈΡΠ»ΠΈΡ‚Π΅Π»ΡŒ. МногиС органичСскиС вСщСства ΠΏΡ€ΠΈ соприкосновСнии с Π½ΠΈΠΌ Π²ΠΎΡΠΏΠ»Π°ΠΌΠ΅Π½ΡΡŽΡ‚ΡΡ. Π’ Π²ΠΎΠ΄Π΅ оксид Π°Π·ΠΎΡ‚Π° (V) Ρ…ΠΎΡ€ΠΎΡˆΠΎ растворяСтся с ΠΎΠ±Ρ€Π°Π·ΠΎΠ²Π°Π½ΠΈΠ΅ΠΌ Π°Π·ΠΎΡ‚Π½ΠΎΠΉ кислоты.

Π’ Ρ‚Π²Π΅Ρ€Π΄ΠΎΠΌ состоянии N 2 O 5 ΠΎΠ±Ρ€Π°Π·ΠΎΠ²Π°Π½ Π½ΠΈΡ‚Ρ€Π°Ρ‚Π½Ρ‹ΠΌ ΠΈΠΎΠ½ΠΎΠΌ NO 3 ΠΈ ΠΈΠΎΠ½ΠΎΠΌ

нитрония NO 2 . ПослСдний содСрТит Ρ‚Π°ΠΊΠΎΠ΅ ΠΆΠ΅ число элСктронов, Ρ‡Ρ‚ΠΎ ΠΈ ΠΌΠΎ-

Π»Π΅ΠΊΡƒΠ»Π° CO 2 ΠΈ, ΠΏΠΎΠ΄ΠΎΠ±Π½ΠΎ послСднСй, ΠΈΠΌΠ΅Π΅Ρ‚ Π»ΠΈΠ½Π΅ΠΉΠ½ΠΎΠ΅ строСниС: O=N=O.

Π’ ΠΏΠ°Ρ€Π°Ρ… ΠΌΠΎΠ»Π΅ΠΊΡƒΠ»Π° N 2 O 5 симмСтрична; Π΅Π΅ строСниС ΠΌΠΎΠΆΠ΅Ρ‚ Π±Ρ‹Ρ‚ΡŒ прСдставлСно ΡΠ»Π΅Π΄ΡƒΡŽΡ‰Π΅ΠΉ Π²Π°Π»Π΅Π½Ρ‚Π½ΠΎΠΉ схСмой, Π² ΠΊΠΎΡ‚ΠΎΡ€ΠΎΠΉ ΠΏΡƒΠ½ΠΊΡ‚ΠΈΡ€ΠΎΠΌ ΠΏΠΎΠΊΠ°Π·Π°Π½Ρ‹ Ρ‚Ρ€Π΅Ρ…Ρ†Π΅Π½Ρ‚Ρ€ΠΎΠ²Ρ‹Π΅ связи (ср. с Π²Π°Π»Π΅Π½Ρ‚Π½ΠΎΠΉ схСмой ΠΌΠΎΠ»Π΅ΠΊΡƒΠ»Ρ‹ Π°Π·ΠΎΡ‚Π½ΠΎΠΉ кислоты).

Π’ связи с Ρ‚Π΅ΠΌ, Ρ‡Ρ‚ΠΎ Π² своих соСдинСниях Π°Π·ΠΎΡ‚ проявляСт Ρ€Π°Π·Π»ΠΈΡ‡Π½Ρ‹Π΅ валСнтности, для этого элСмСнта Ρ…Π°Ρ€Π°ΠΊΡ‚Π΅Ρ€Π½ΠΎ нСсколько оксидов: оксид Π΄ΠΈΠ°Π·ΠΎΡ‚Π°, ΠΌΠΎΠ½ΠΎ-, Ρ‚Ρ€ΠΈ-, Π΄ΠΈ- ΠΈ пСнтаоксиды Π°Π·ΠΎΡ‚Π°. Рассмотрим ΠΊΠ°ΠΆΠ΄Ρ‹ΠΉ ΠΈΠ· Π½ΠΈΡ… Π±ΠΎΠ»Π΅Π΅ ΠΏΠΎΠ΄Ρ€ΠΎΠ±Π½ΠΎ.

ΠžΠŸΠ Π•Π”Π•Π›Π•ΠΠ˜Π•

Оксид Π΄ΠΈΠ°Π·ΠΎΡ‚Π° (вСсСлящий Π³Π°Π·, закись Π°Π·ΠΎΡ‚Π°) прСдставляСт собой бСсцвСтный Π³Π°Π·, тСрмичСски устойчивый.

ΠŸΠ»ΠΎΡ…ΠΎ растворяСтся Π² Π²ΠΎΠ΄Π΅. ΠŸΡ€ΠΈ сильном ΠΎΡ…Π»Π°ΠΆΠ΄Π΅Π½ΠΈΠΈ ΠΈΠ· раствора кристаллизуСтся ΠΊΠ»Π°Ρ€Π°Ρ‚ N 2 OΓ—5,75H 2 O.

ΠžΠŸΠ Π•Π”Π•Π›Π•ΠΠ˜Π•

Монооксид Π°Π·ΠΎΡ‚Π° ΠΌΠΎΠΆΠ΅Ρ‚ ΡΡƒΡ‰Π΅ΡΡ‚Π²ΠΎΠ²Π°Ρ‚ΡŒ ΠΊΠ°ΠΊ Π² Π²ΠΈΠ΄Π΅ бСсцвСтного Π³Π°Π·Π°, Ρ‚Π°ΠΊ ΠΈ Тидкости Π³ΠΎΠ»ΡƒΠ±ΠΎΠ³ΠΎ Ρ†Π²Π΅Ρ‚Π°.

Π’ Ρ‚Π²Π΅Ρ€Π΄ΠΎΠΌ состоянии ΠΎΠ½ ΠΏΠΎΠ»Π½ΠΎΡΡ‚ΡŒΡŽ Π΄ΠΈΠΌΠ΅Ρ€ΠΈΠ·ΠΎΠ²Π°Π½ (N 2 O 2), Π² ΠΆΠΈΠ΄ΠΊΠΎΠΌ состоянии - частично (β‰ˆ 25% N 2 O 2), Π² Π³Π°Π·Π΅ - Π² ΠΎΡ‡Π΅Π½ΡŒ ΠΌΠ°Π»ΠΎΠΉ стСпСни. Π§Ρ€Π΅Π·Π²Ρ‹Ρ‡Π°ΠΉΠ½ΠΎ тСрмичСски устойчив. ΠŸΠ»ΠΎΡ…ΠΎ растворяСтся Π² Π²ΠΎΠ΄Π΅.

ΠžΠŸΠ Π•Π”Π•Π›Π•ΠΠ˜Π•

Вриоксид Π°Π·ΠΎΡ‚Π° прСдставляСт собой тСрмичСски Π½Π΅ΡƒΡΡ‚ΠΎΠΉΡ‡ΠΈΠ²ΡƒΡŽ ΠΆΠΈΠ΄ΠΊΠΎΡΡ‚ΡŒ синСго Ρ†Π²Π΅Ρ‚Π°.

ΠŸΡ€ΠΈ ΠΊΠΎΠΌΠ½Π°Ρ‚Π½ΠΎΠΉ Ρ‚Π΅ΠΌΠΏΠ΅Ρ€Π°Ρ‚ΡƒΡ€Π΅ Π½Π° 90% разлагаСтся Π½Π° NOΠΈ NO 2 ΠΈ ΠΎΠΊΡ€Π°ΡˆΠΈΠ²Π°Π΅Ρ‚ΡΡ Π² Π±ΡƒΡ€Ρ‹ΠΉ Ρ†Π²Π΅Ρ‚ (NO 2), Π½Π΅ ΠΈΠΌΠ΅Π΅Ρ‚ Ρ‚Π΅ΠΌΠΏΠ΅Ρ€Π°Ρ‚ΡƒΡ€Ρ‹ кипСния (NO испаряСтся ΠΏΠ΅Ρ€Π²Ρ‹ΠΌ). Π’ Ρ‚Π²Π΅Ρ€Π΄ΠΎΠΌ состоянии - это Π±Π΅Π»ΠΎΠ΅ ΠΈΠ»ΠΈ Π³ΠΎΠ»ΡƒΠ±ΠΎΠ²Π°Ρ‚ΠΎΠ΅ вСщСство с ΠΈΠΎΠ½Π½Ρ‹ΠΌ строСниСм - Π½ΠΈΡ‚Ρ€ΠΈΡ‚ Π½ΠΈΡ‚Ρ€ΠΎΠ·ΠΈΠ»Π° (NO +)(NO 2 β€”). Π’ Π³Π°Π·Π΅ ΠΈΠΌΠ΅Π΅Ρ‚ молСкулярноС строСниС ON-NO 2 .

ΠžΠŸΠ Π•Π”Π•Π›Π•ΠΠ˜Π•

Диоксид Π°Π·ΠΎΡ‚Π° (лисий хвост) прСдставляСт собой Π±ΡƒΡ€Ρ‹ΠΉ Π³Π°Π·.

ΠŸΡ€ΠΈ Ρ‚Π΅ΠΌΠΏΠ΅Ρ€Π°Ρ‚ΡƒΡ€Π΅ Π²Ρ‹ΡˆΠ΅ 135 o Π‘ - это ΠΌΠΎΠ½ΠΎΠΌΠ΅Ρ€, ΠΏΡ€ΠΈ ΠΊΠΎΠΌΠ½Π°Ρ‚Π½ΠΎΠΉ Ρ‚Π΅ΠΌΠΏΠ΅Ρ€Π°Ρ‚ΡƒΡ€Π΅ - красно-бурая смСсь NO 2 ΠΈ Π΅Π³ΠΎ Π΄ΠΈΠΌΠ΅Ρ€Π° (тСтраоксида Π΄ΠΈΠ°Π·ΠΎΡ‚Π°) N 2 O 4 . Π’ ΠΆΠΈΠ΄ΠΊΠΎΠΌ состоянии Π΄ΠΈΠΌΠ΅Ρ€ бСсцвСтСн, Π² Ρ‚Π²Π΅Ρ€Π΄ΠΎΠΌ состоянии Π±Π΅Π»Ρ‹ΠΉ. Π₯ΠΎΡ€ΠΎΡˆΠΎ растворяСтся Π² Ρ…ΠΎΠ»ΠΎΠ΄Π½ΠΎΠΉ Π²ΠΎΠ΄Π΅ (насыщСнный раствор - ярко-Π·Π΅Π»Π΅Π½Ρ‹ΠΉ), ΠΏΠΎΠ»Π½ΠΎΡΡ‚ΡŒΡŽ рСагируя с Π½Π΅ΠΉ.

ΠžΠŸΠ Π•Π”Π•Π›Π•ΠΠ˜Π•

ΠŸΠ΅Π½Ρ‚Π°ΠΎΠΊΡΠΈΠ΄ Π°Π·ΠΎΡ‚Π° (Π°Π·ΠΎΡ‚Π½Ρ‹ΠΉ Π°Π½Π³ΠΈΠ΄Ρ€ΠΈΠ΄) прСдставляСт собой Π±Π΅Π»ΠΎΠ΅ Ρ‚Π²Π΅Ρ€Π΄ΠΎΠ΅ вСщСство, бСсцвСтныС Π³Π°Π· ΠΈ ΠΆΠΈΠ΄ΠΊΠΎΡΡ‚ΡŒ.

ΠŸΡ€ΠΈ Π½Π°Π³Ρ€Π΅Π²Π°Π½ΠΈΠΈ возгоняСтся ΠΈ плавится, ΠΏΡ€ΠΈ ΠΊΠΎΠΌΠ½Π°Ρ‚Π½ΠΎΠΉ Ρ‚Π΅ΠΌΠΏΠ΅Ρ€Π°Ρ‚ΡƒΡ€Π΅ разлагаСтся Π·Π° 10 часов. Π’ Ρ‚Π²Π΅Ρ€Π΄ΠΎΠΌ состоянии ΠΈΠΌΠ΅Π΅Ρ‚ ΠΈΠΎΠ½Π½ΠΎΠ΅ строСниС (NO 2 +)(NO 3 β€”) - Π½ΠΈΡ‚Ρ€Π°Ρ‚ Π½ΠΈΡ‚Ρ€ΠΎΠΈΠ»Π°.

Π’Π°Π±Π»ΠΈΡ†Π° 1. ЀизичСскиС свойства оксидов Π°Π·ΠΎΡ‚Π°.

ΠŸΠΎΠ»ΡƒΡ‡Π΅Π½ΠΈΠ΅ оксида Π°Π·ΠΎΡ‚Π°

Π’ Π»Π°Π±ΠΎΡ€Π°Ρ‚ΠΎΡ€Π½Ρ‹Ρ… условиях оксид Π΄ΠΈΠ°Π·ΠΎΡ‚Π° ΠΏΠΎΠ»ΡƒΡ‡Π°ΡŽΡ‚ ΠΏΡƒΡ‚Π΅ΠΌ остороТного нагрСвания сухого Π½ΠΈΡ‚Ρ€Π°Ρ‚Π° аммония (1) ΠΈΠ»ΠΈ Π½Π°Π³Ρ€Π΅Π²Π°Π½ΠΈΠ΅ΠΌ смСси ΡΡƒΠ»ΡŒΡ„Π°ΠΌΠΈΠ½ΠΎΠ²ΠΎΠΉ ΠΈ Π°Π·ΠΎΡ‚Π½ΠΎΠΉ (73%-ная) кислот (2):

NH 4 NO 3 = N 2 O + 2H 2 O (1);

NH 2 SO 2 OH + HNO 3 = N 2 O + H 2 SO 4 + H 2 O (2).

Монооксид Π°Π·ΠΎΡ‚Π° ΠΏΠΎΠ»ΡƒΡ‡Π°ΡŽΡ‚ взаимодСйствиСм простых вСщСств Π°Π·ΠΎΡ‚Π° ΠΈ кислорода ΠΏΡ€ΠΈ высоких Ρ‚Π΅ΠΌΠΏΠ΅Ρ€Π°Ρ‚ΡƒΡ€Π°Ρ… (β‰ˆ1300 o Π‘):

N 2 + O 2 = 2NO.

ΠšΡ€ΠΎΠΌΠ΅ этого оксид Π°Π·ΠΎΡ‚Π° (II) являСтся ΠΎΠ΄Π½ΠΈΠΌ ΠΈΠ· ΠΏΡ€ΠΎΠ΄ΡƒΠΊΡ‚ΠΎΠ² Ρ€Π΅Π°ΠΊΡ†ΠΈΠΈ растворСния ΠΌΠ΅Π΄ΠΈ Π² Ρ€Π°Π·Π±Π°Π²Π»Π΅Π½Π½ΠΎΠΉ Π°Π·ΠΎΡ‚Π½ΠΎΠΉ кислотС:

3Cu + 8HNO 3 = 3Cu(NO 3) 2 + 2NO + 4H 2 O.

ΠŸΡ€ΠΈ ΠΎΡ…Π»Π°ΠΆΠ΄Π΅Π½ΠΈΠΈ смСси Π³Π°Π·ΠΎΠ², состоящСй ΠΈΠ· оксидов Π°Π·ΠΎΡ‚Π° (II) ΠΈ (IV) Π΄ΠΎ -36 o Π‘ образуСтся триоксид Π°Π·ΠΎΡ‚Π°:

NO + NO 2 = N 2 O 3 .

Π”Π°Π½Π½ΠΎΠ΅ соСдинСниС ΠΌΠΎΠΆΠ½ΠΎ ΠΏΠΎΠ»ΡƒΡ‡ΠΈΡ‚ΡŒ ΠΏΡ€ΠΈ дСйствии 50%-Π½ΠΎΠΉ Π°Π·ΠΎΡ‚Π½ΠΎΠΉ кислоты Π½Π° оксид ΠΌΡ‹ΡˆΡŒΡΠΊΠ° (III) (3) ΠΈΠ»ΠΈ ΠΊΡ€Π°Ρ…ΠΌΠ°Π» (4):

2HNO 3 + As 2 O 3 = NO 2 + NO + 2HAsO 3 (3);

HNO 3 + (C 6 H 10 O 5) n = 6nNO + 6nNO 2 + 6nCO 2 + 11nH 2 O (4).

ВСрмичСскоС Ρ€Π°Π·Π»ΠΎΠΆΠ΅Π½ΠΈΠ΅ Π½ΠΈΡ‚Ρ€Π°Ρ‚Π° свинца (II) ΠΏΡ€ΠΈΠ²ΠΎΠ΄ΠΈΡ‚ ΠΊ ΠΎΠ±Ρ€Π°Π·ΠΎΠ²Π°Π½ΠΈΡŽ диоксидазота:

2Pb(NO 3) 2 = 2PbO + 4NO 2 + O 2 .

Π­Ρ‚ΠΎ ΠΆΠ΅ соСдинСниС образуСтся ΠΏΡ€ΠΈ растворСнии ΠΌΠ΅Π΄ΠΈ Π² ΠΊΠΎΠ½Ρ†Π΅Π½Ρ‚Ρ€ΠΈΡ€ΠΎΠ²Π°Π½Π½ΠΎΠΉ Π°Π·ΠΎΡ‚Π½ΠΎΠΉ кислотС:

Cu + 4HNO 3 = Cu(NO 3) 2 + 2NO 2 + 2H 2 O.

ΠŸΠ΅Π½Ρ‚Π°ΠΎΠΊΡΠΈΠ΄ Π°Π·ΠΎΡ‚Π° ΠΏΠΎΠ»ΡƒΡ‡Π°ΡŽΡ‚ ΠΏΡƒΡ‚Π΅ΠΌ пропускания сухого Ρ…Π»ΠΎΡ€Π° Π½Π°Π΄ сухим Π½ΠΈΡ‚Ρ€Π°Ρ‚ΠΎΠΌ сСрСбра (5), Π° Ρ‚Π°ΠΊΠΆΠ΅ ΠΏΠΎ Ρ€Π΅Π°ΠΊΡ†ΠΈΠΈ взаимодСйствия ΠΌΠ΅ΠΆΠ΄Ρƒ оксидом Π°Π·ΠΎΡ‚Π° (IV) ΠΈ ΠΎΠ·ΠΎΠ½ΠΎΠΌ (6):

2Cl 2 + 4AgNO 3 = 2N 2 O 5 + 4AgCl + O 2 (5);

2NO 2 + O 3 = N 2 O 5 + O 2 (6).

Π₯имичСскиС свойства оксида Π°Π·ΠΎΡ‚Π°

Оксид Π΄ΠΈΠ°Π·ΠΎΡ‚Π° малорСакционноспособный, Π½Π΅ Ρ€Π΅Π°Π³ΠΈΡ€ΡƒΠ΅Ρ‚ с Ρ€Π°Π·Π±Π°Π²Π»Π΅Π½Π½Ρ‹ΠΌΠΈ кислотами, Ρ‰Π΅Π»ΠΎΡ‡Π°ΠΌΠΈ, Π³ΠΈΠ΄Ρ€Π°Ρ‚ΠΎΠΌ Π°ΠΌΠΌΠΈΠ°ΠΊΠ°, кислородом. ΠŸΡ€ΠΈ Π½Π°Π³Ρ€Π΅Π²Π°Π½ΠΈΠΈ Ρ€Π΅Π°Π³ΠΈΡ€ΡƒΠ΅Ρ‚ с ΠΊΠΎΠ»Π½Ρ†Π΅Π½Ρ‚Ρ€ΠΈΡ€ΠΎΠ²Π°Π½Π½ΠΎΠΉ сСрной кислотой, Π²ΠΎΠ΄ΠΎΡ€ΠΎΠ΄ΠΎΠΌ, ΠΌΠ΅Ρ‚Π°Π»Π»Π°ΠΌΠΈ, Π°ΠΌΠΌΠΈΠ°ΠΊΠΎΠΌ. ΠŸΠΎΠ΄Π΄Π΅Ρ€ΠΆΠΈΠ²Π°Π΅Ρ‚ Π³ΠΎΡ€Π΅Π½ΠΈΠ΅ ΡƒΠ³Π»Π΅Ρ€ΠΎΠ΄Π° ΠΈ фосфора. Π’ ΠžΠ’Π  ΠΌΠΎΠΆΠ΅Ρ‚ ΠΏΡ€ΠΎΡΠ²Π»ΡΡ‚ΡŒ свойства ΠΊΠ°ΠΊ слабого окислитСля, Ρ‚Π°ΠΊ ΠΈ слабого восстановитСля.

Монооксид Π°Π·ΠΎΡ‚Π° Π½Π΅ Ρ€Π΅Π°Π³ΠΈΡ€ΡƒΠ΅Ρ‚ с Π²ΠΎΠ΄ΠΎΠΉ, Ρ€Π°Π·Π±Π°Π²Π»Π΅Π½Π½Ρ‹ΠΌΠΈ кислотами, Ρ‰Π΅Π»ΠΎΡ‡Π°ΠΌΠΈ, Π³ΠΈΠ΄Ρ€Π°Ρ‚ΠΎΠΌ Π°ΠΌΠΌΠΈΠ°ΠΊΠ°. МгновСнно присоСдиняСт кислород. ΠŸΡ€ΠΈ Π½Π°Π³Ρ€Π΅Π²Π°Π½ΠΈΠΈ Ρ€Π΅Π°Π³ΠΈΡ€ΡƒΠ΅Ρ‚ с Π³Π°Π»ΠΎΠ³Π΅Π½Π°ΠΌΠΈ ΠΈ Π΄Ρ€ΡƒΠ³ΠΈΠΌΠΈ Π½Π΅ΠΌΠ΅Ρ‚Π°Π»Π»Π°ΠΌΠΈ, ΡΠΈΠ»ΡŒΠ½Ρ‹ΠΌΠΈ окислитСлями ΠΈ восстановитСлями. ВступаСт Π² Ρ€Π΅Π°ΠΊΡ†ΠΈΠΈ комплСксообразования.

Вриоксид Π°Π·ΠΎΡ‚Π° проявляСт кислотныС свойства, Ρ€Π΅Π°Π³ΠΈΡ€ΡƒΠ΅Ρ‚ с Π²ΠΎΠ΄ΠΎΠΉ, Ρ‰Π΅Π»ΠΎΡ‡Π°ΠΌΠΈ, Π³ΠΈΠ΄Ρ€Π°Ρ‚ΠΎΠΌ Π°ΠΌΠΌΠΈΠ°ΠΊΠ°. Π­Π½Π΅Ρ€Π³ΠΈΡ‡Π½ΠΎ Ρ€Π΅Π°Π³ΠΈΡ€ΡƒΠ΅Ρ‚ с кислородом ΠΈ ΠΎΠ·ΠΎΠ½ΠΎΠΌ, окисляСт ΠΌΠ΅Ρ‚Π°Π»Π»Ρ‹.

Диоксид Π°Π·ΠΎΡ‚Π° Ρ€Π΅Π°Π³ΠΈΡ€ΡƒΠ΅Ρ‚ с Π²ΠΎΠ΄ΠΎΠΉ ΠΈ Ρ‰Π΅Π»ΠΎΡ‡Π°ΠΌΠΈ. Π’ ΠžΠ’Π  проявляСт свойства сильного окислитСля. Π’Ρ‹Π·Ρ‹Π²Π°Π΅Ρ‚ ΠΊΠΎΡ€Ρ€ΠΎΠ·ΠΈΡŽ ΠΌΠ΅Ρ‚Π°Π»Π»ΠΎΠ².

ΠŸΠ΅Π½Ρ‚Π°ΠΎΠΊΡΠΈΠ΄ Π°Π·ΠΎΡ‚Π° проявляСт кислотныС свойства, Ρ€Π΅Π°Π³ΠΈΡ€ΡƒΠ΅Ρ‚ с Π²ΠΎΠ΄ΠΎΠΉ, Ρ‰Π΅Π»ΠΎΡ‡Π°ΠΌΠΈ, Π³ΠΈΠ΄Ρ€Π°Ρ‚ΠΎΠΌ Π°ΠΌΠΌΠΈΠ°ΠΊΠ°. ЯвляСтся ΠΎΡ‡Π΅Π½ΡŒ ΡΠΈΠ»ΡŒΠ½Ρ‹ΠΌ окислитСлСм.

ΠŸΡ€ΠΈΠΌΠ΅Π½Π΅Π½ΠΈΠ΅ оксида Π°Π·ΠΎΡ‚Π°

Оксид Π΄ΠΈΠ°Π·ΠΎΡ‚Π° ΠΈΡΠΏΠΎΠ»ΡŒΠ·ΡƒΡŽΡ‚ Π² ΠΏΠΈΡ‰Π΅Π²ΠΎΠΉ ΠΏΡ€ΠΎΠΌΡ‹ΡˆΠ»Π΅Π½Π½ΠΎΡΡ‚ΠΈ (ΠΏΡ€ΠΎΠΏΠ΅Π»Π»Π΅Π½Ρ‚ ΠΏΡ€ΠΈ ΠΈΠ·Π³ΠΎΡ‚ΠΎΠ²Π»Π΅Π½ΠΈΠΈ Π²Π·Π±ΠΈΡ‚Ρ‹Ρ… сливок), ΠΌΠ΅Π΄ΠΈΡ†ΠΈΠ½Π΅ (для ингаляционного Π½Π°Ρ€ΠΊΠΎΠ·Π°), Π° Ρ‚Π°ΠΊΠΆΠ΅ Π² качСствС основного ΠΊΠΎΠΌΠΏΠΎΠ½Π΅Π½Ρ‚Π° Ρ€Π°ΠΊΠ΅Ρ‚Π½ΠΎΠ³ΠΎ Ρ‚ΠΎΠΏΠ»ΠΈΠ²Π°.

Вриоксид ΠΈ диоксид Π°Π·ΠΎΡ‚Π° ΠΏΡ€ΠΈΠΌΠ΅Π½ΡΡŽΡ‚ΡΡ Π² нСорганичСском синтСзС для получСния Π°Π·ΠΎΡ‚Π½ΠΎΠΉ ΠΈ сСрной кислот. Оксид Π°Π·ΠΎΡ‚Π° (IV) Ρ‚Π°ΠΊΠΆΠ΅ нашСл ΠΏΡ€ΠΈΠΌΠ΅Π½Π΅Π½ΠΈΠ΅ Π² качСствС ΠΎΠ΄Π½ΠΎΠ³ΠΎ ΠΈΠ· ΠΊΠΎΠΌΠΏΠΎΠ½Π΅Π½Ρ‚ΠΎΠ² Ρ€Π°ΠΊΠ΅Ρ‚Π½ΠΎΠ³ΠΎ Ρ‚ΠΎΠΏΠ»ΠΈΠ²Π° ΠΈ смСсСвых Π²Π·Ρ€Ρ‹Π²Ρ‡Π°Ρ‚Ρ‹Ρ… вСщСств.

ΠŸΡ€ΠΈΠΌΠ΅Ρ€Ρ‹ Ρ€Π΅ΡˆΠ΅Π½ΠΈΡ Π·Π°Π΄Π°Ρ‡

ΠŸΠ Π˜ΠœΠ•Π  1

Π—Π°Π΄Π°Π½ΠΈΠ΅ Оксид Π°Π·ΠΎΡ‚Π° содСрТит 63,2% кислорода. Какова Ρ„ΠΎΡ€ΠΌΡƒΠ»Π° оксида.
РСшСниС Массовая доля элСмСнта Π₯ Π² ΠΌΠΎΠ»Π΅ΠΊΡƒΠ»Π΅ состава НΠ₯ рассчитываСтся ΠΏΠΎ ΡΠ»Π΅Π΄ΡƒΡŽΡ‰Π΅ΠΉ Ρ„ΠΎΡ€ΠΌΡƒΠ»Π΅:

Ο‰ (Π₯) = n Γ— Ar (X) / M (HX) Γ— 100%.

Вычислим ΠΌΠ°ΡΡΠΎΠ²ΡƒΡŽ долю Π°Π·ΠΎΡ‚Π° Π² оксидС:

Ο‰ (N) = 100% β€” Ο‰(O) = 100% β€” 63,2% = 36,8%.

ΠžΠ±ΠΎΠ·Π½Π°Ρ‡ΠΈΠΌ количСство моль элСмСнтов, входящих Π² состав соСдинСния Π·Π° Β«Ρ…Β» (Π°Π·ΠΎΡ‚) ΠΈ Β«ΡƒΒ» (кислород). Π’ΠΎΠ³Π΄Π°, мольноС ΠΎΡ‚Π½ΠΎΡˆΠ΅Π½ΠΈΠ΅ Π±ΡƒΠ΄Π΅Ρ‚ Π²Ρ‹Π³Π»ΡΠ΄Π΅Ρ‚ΡŒ ΡΠ»Π΅Π΄ΡƒΡŽΡ‰ΠΈΠΌ ΠΎΠ±Ρ€Π°Π·ΠΎΠΌ (значСния ΠΎΡ‚Π½ΠΎΡΠΈΡ‚Π΅Π»ΡŒΠ½Ρ‹Ρ… Π°Ρ‚ΠΎΠΌΠ½Ρ‹Ρ… масс, взятых ΠΈΠ· ΠŸΠ΅Ρ€ΠΈΠΎΠ΄ΠΈΡ‡Π΅ΡΠΊΠΎΠΉ Ρ‚Π°Π±Π»ΠΈΡ†Ρ‹ Π”.И. МСндСлССва, ΠΎΠΊΡ€ΡƒΠ³Π»ΠΈΠΌ Π΄ΠΎ Ρ†Π΅Π»Ρ‹Ρ… чисСл):

x:y = Ο‰(N)/Ar(N) : Ο‰(O)/Ar(O);

x:y= 36,8/14: 63,2/16;

x:y= 2,6: 3,95 = 1: 2.

Π—Π½Π°Ρ‡ΠΈΡ‚ Ρ„ΠΎΡ€ΠΌΡƒΠ»Π° соСдинСния Π°Π·ΠΎΡ‚Π° ΠΈ кислорода Π±ΡƒΠ΄Π΅Ρ‚ ΠΈΠΌΠ΅Ρ‚ΡŒ Π²ΠΈΠ΄ NO 2 . Π­Ρ‚ΠΎ оксид Π°Π·ΠΎΡ‚Π° (IV).

ΠžΡ‚Π²Π΅Ρ‚ NO 2

ΠŸΠ Π˜ΠœΠ•Π  2

Π—Π°Π΄Π°Π½ΠΈΠ΅ КакиС Π³Π°Π·Ρ‹ тяТСлСС, Π° ΠΊΠ°ΠΊΠΈΠ΅ Π»Π΅Π³Ρ‡Π΅ Π²ΠΎΠ·Π΄ΡƒΡ…Π° ΠΈ Π²ΠΎ сколько Ρ€Π°Π·: диоксид ΡƒΠ³Π»Π΅Ρ€ΠΎΠ΄Π°, диоксид Π°Π·ΠΎΡ‚Π°, монооксид ΡƒΠ³Π»Π΅Ρ€ΠΎΠ΄Π°, Ρ…Π»ΠΎΡ€, Π°ΠΌΠΌΠΈΠ°ΠΊ?
РСшСниС ΠžΡ‚Π½ΠΎΡˆΠ΅Π½ΠΈΠ΅ массы Π΄Π°Π½Π½ΠΎΠ³ΠΎ Π³Π°Π·Π° ΠΊ массС Π΄Ρ€ΡƒΠ³ΠΎΠ³ΠΎ Π³Π°Π·Π°, взятого Π² Ρ‚ΠΎΠΌ ΠΆΠ΅ объСмС, ΠΏΡ€ΠΈ Ρ‚ΠΎΠΉ ΠΆΠ΅ Ρ‚Π΅ΠΌΠΏΠ΅Ρ€Π°Ρ‚ΡƒΡ€Π΅ ΠΈ Ρ‚ΠΎΠΌ ΠΆΠ΅ Π΄Π°Π²Π»Π΅Π½ΠΈΠΈ, называСтся ΠΎΡ‚Π½ΠΎΡΠΈΡ‚Π΅Π»ΡŒΠ½ΠΎΠΉ ΠΏΠ»ΠΎΡ‚Π½ΠΎΡΡ‚ΡŒΡŽ ΠΏΠ΅Ρ€Π²ΠΎΠ³ΠΎ Π³Π°Π·Π° ΠΏΠΎ Π²Ρ‚ΠΎΡ€ΠΎΠΌΡƒ. Данная Π²Π΅Π»ΠΈΡ‡ΠΈΠ½Π° ΠΏΠΎΠΊΠ°Π·Ρ‹Π²Π°Π΅Ρ‚, Π²ΠΎ сколько Ρ€Π°Π· ΠΏΠ΅Ρ€Π²Ρ‹ΠΉ Π³Π°Π· тяТСлСС ΠΈΠ»ΠΈ Π»Π΅Π³Ρ‡Π΅ Π²Ρ‚ΠΎΡ€ΠΎΠ³ΠΎ Π³Π°Π·Π°.

ΠžΡ‚Π½ΠΎΡΠΈΡ‚Π΅Π»ΡŒΠ½ΡƒΡŽ ΠΌΠΎΠ»Π΅ΠΊΡƒΠ»ΡΡ€Π½ΡƒΡŽ массу Π²ΠΎΠ·Π΄ΡƒΡ…Π° ΠΏΡ€ΠΈΠ½ΠΈΠΌΠ°ΡŽΡ‚ Ρ€Π°Π²Π½ΠΎΠΉ 29 (с ΡƒΡ‡Π΅Ρ‚ΠΎΠΌ содСрТания Π² Π²ΠΎΠ·Π΄ΡƒΡ…Π΅ Π°Π·ΠΎΡ‚Π°, кислорода ΠΈ Π΄Ρ€ΡƒΠ³ΠΈΡ… Π³Π°Π·ΠΎΠ²). Π‘Π»Π΅Π΄ΡƒΠ΅Ρ‚ ΠΎΡ‚ΠΌΠ΅Ρ‚ΠΈΡ‚ΡŒ, Ρ‡Ρ‚ΠΎ понятиС Β«ΠΎΡ‚Π½ΠΎΡΠΈΡ‚Π΅Π»ΡŒΠ½Π°Ρ молСкулярная масса Π²ΠΎΠ·Π΄ΡƒΡ…Π°Β» употрСбляСтся условно, Ρ‚Π°ΠΊ ΠΊΠ°ΠΊ Π²ΠΎΠ·Π΄ΡƒΡ… - это смСсь Π³Π°Π·ΠΎΠ².

D air (CO 2) = M r (CO 2) / M r (air);

D air (CO 2) = 44 / 29 = 1,52.

M r (CO 2) = A r (C) + 2 Γ—A r (O) = 12 + 2 Γ— 16 = 12 + 32 = 44.

D air (NO 2) = M r (NO 2) / M r (air);

D air (NO 2) = 46 / 29 = 1,59.

M r (NO 2) = A r (N) + 2 Γ—A r (O) = 14 + 2 Γ— 16 = 14 + 32 = 46.

D air (CO) = M r (CO) / M r (air);

D air (CO) = 28 / 29 = 0,97.

M r (CO) = A r (C) + A r (O) = 12 + 16 = 28.

D air (Cl 2) = M r (Cl 2) / M r (air);

D air (Cl 2) = 71 / 29 = 2,45.

M r (Cl 2) = 2 Γ— A r (Cl) = 2 Γ— 35,5 = 71.

D air (NH 3) = M r (NH 3) / M r (air);

D air (NH 3) = 17 / 29 = 0,57.

M r (NH 3) = A r (N) + 3 Γ—A r (H) = 14 + 3 Γ—1 = 17.

ΠžΡ‚Π²Π΅Ρ‚ Диоксид ΡƒΠ³Π»Π΅Ρ€ΠΎΠ΄Π°, диоксид Π°Π·ΠΎΡ‚Π° ΠΈ Ρ…Π»ΠΎΡ€ тяТСлСС Π²ΠΎΠ·Π΄ΡƒΡ…Π° соотвСтствСнно Π² 1,52; 1,59 ΠΈ 2,45 Ρ€Π°Π·Π°, Π° монооксид ΡƒΠ³Π»Π΅Ρ€ΠΎΠ΄Π° ΠΈ Π°ΠΌΠΌΠΈΠ°ΠΊ - Π»Π΅Π³Ρ‡Π΅ Π² 0,97 ΠΈ 0,57 Ρ€Π°Π·.